Nurse 261 - exam 2 FA DAVIS

अब Quizwiz के साथ अपने होमवर्क और परीक्षाओं को एस करें!

The parents of a neonate with a patent ductus arteriosus ask why their child is receiving ibuprofen if the child is not in pain. What is the nurse's best response? "Ibuprofen will help facilitate closure of the patent ductus arteriosus." "Ibuprofen is used prophylactically to prevent infection in children with cardiac disorders." "Ibuprofen will help keep the ductus open in order to provide additional blood flow to the body." "Routine pain medication is given to children with cardiac disorders to keep them calm."

"Ibuprofen will help facilitate closure of the patent ductus arteriosus." Rationale: ibuprofen results in complete cessation of ductal flow by facilitating necrosis of the intima of the ductus arteriosus.

A 10-year-old child is admitted to the hospital with a suspected brain tumor and is scheduled for a magnetic resonance imaging (MRI) in the morning. What should the nurse include in the child's teaching? "The MRI will explain the type of abnormal growth." "The MRI will locate the exact place that is causing you to be sick." "The MRI room is hot and noisy during the study." "The next test will differentiate the cells and help with future treatments."

"The MRI will locate the exact place that is causing you to be sick."

A child with chronic otitis media is having PE tubes placed. Which statement would indicate that the parent understands education about PE tubes? "The tubes have to be surgically removed in 9 months or so." "The tubes were placed to equalize pressure." "The tubes will be visible outside the ear." "The tubes will be placed while the child is awake."

"The tubes were placed to equalize pressure."

The nurse is discussing with the parents of a child just diagnosed with a cardiac defect an upcoming noninvasive test that indicates structure, size, flow patterns, function, and the blood vessels attached to the heart. The parents ask the nurse what the test is called, as they would like to do more research. What is the appropriate nursing response? "This test is called an echocardiogram." "An electrocardiogram (EKG) is the test used to look at flow pattern functions." "This is called angiography." "This is referred to as a cardiac catheterization."

"This test is called an echocardiogram." Rationale: a noninvasive test that indicates structure, size, flow patterns, function, and the blood vessels attached to the heart is an echocardiogram.

Which child does the nurse anticipate to be most at risk for being hospitalized for respiratory syncytial virus (RSV)? 1. A three-month-old who was born at 30 weeks gestation 2. A 18-month-old with a tracheostomy 3. A four-year-old with a ventricular septal defect (VSD) 4. A five-year-old who was term but has never received any immunizations

1, Option 1: The younger the child, the greater the risk for the child. This is especially so if the child was a premature delivery. Option 2: The child has a tracheostomy but could potentially be monitored at home. Option 3: A four-year old with a VSD is not the highest risk in this group. Option 4: A term infant would not need to get an immunization for RSV. Having not received any other immunizations does not put the child at risk for developing RSV.

A 15-month-old is admitted to the pediatric unit with a history of a recent upper respiratory infection. Which symptom is consistent with the diagnosis of laryngotracheobronchitis (croup)? Select all that apply. 1. Reported inspiratory stridor which is worse at night. 2. Suprasternal retractions are present upon examination. 3. The toddler has a barking, seal-like, harsh cough. 4. Lung sounds have inspiratory wheezing. 5. Lung sounds with crackles in the bases bilaterally.

1,2,3 Option 1: Inspiratory stridor is present in croup and as a result of the inflammation of the larynx and trachea. Option 2: Retractions are a sign of respiratory distress and present in croup. Option 3: This is a manifestation of croup.

Which should the nurse expect to be included in the treatment of the client experiencing acute asthma symptoms? Select all that apply. 1. Bronchodilators 2. Corticosteroids 3. Oxygen 4. Montelukast (Singular) 5. Immediate Intubation

1,2,3 Option 1: Short acting beta 2 bronchodilators should be used in an acute asthma attack. Option 2: Steroids will decrease airway edema and are useful in status asthmaticus. Option 3: Oxygen should be used in an acute asthma attack to facilitate breathing. Option 4: This is a medication used in long-term maintenance for asthma. It is not used for an acute asthma attack.

The nurse should instruct the parent whose child is diagnosed with respiratory syncytial virus (RSV) to notify the healthcare provider for which issue? Select all that apply. 1. The child is not eating 2. There is a decrease in wet diapers 3. There is increased work of breathing 4. The child develops yellow drainage from the nose 5. Only when the child wheezes

1,2,3, Option 1: This could lead to dehydration and the child may need intravenous fluids. Option 2: This could lead to dehydration. Option 3: This is a sign of respiratory distress.

Which statements are true about apnea of prematurity? Select all that apply. 1. It occurs in most infants at less than 28 weeks gestation. 2. As premature births have expanded in the United States so has apnea of prematurity. 3. It occurs in 50% of infants born at 30-31 weeks gestation. 4. It is not affected by maternal drug use. 5. It usually resolves by 36 weeks post-conceptual age.

1,2,3,5 Option 1: Apnea of prematurity does occur in most infants at less than 28 weeks gestation. Option 2: As the ultra-preemie population has increased in the United States, so has the incidence of apnea of prematurity. Option 3: Apnea of prematurity does occur in 50 % of infants born at 30-31 weeks gestation. option 5: Apnea of prematurity does usually resolve by 36 weeks post conceptual age.

You are teaching a family with a child who has cystic fibrosis (CF) about chest physiotherapy treatment (CPT). Which of the following teaching points are correct to include? 1. It should be performed three to four times a day. 2. It may cause bronchospasm. 3. It is all right to percuss over the spine or internal organs. 4. When manually percussing you should use a cupped hand. 5. CPT can be done at any time including after eating.

1,2,4

Which care techniques would a nurse implement when caring for a child diagnosed with respiratory syncytial virus (RSV)? Select all that apply. 1. Antipyretics 2. IV and/or oral fluid therapy 3. IV antibiotics for infection 4. Cardiorespiratory monitoring 5. Immediate intubation to maintain the airway

1,2,4. Option 1: The administration of antipyretics for fever would be beneficial. Option 2: Intravenous or oral fluid therapy will maintain hydration. option 4: Monitoring heart rate and respiratory rate would be indicated.

Which are signs and symptoms of respiratory distress in a two-month-old? Select all that apply. 1. Nasal flaring 2. Intercostal retractions 3. Coughing 4. Bronchovesicular lung sounds 5. Grunting

1,2,5 Option 1: Nasal flaring is a sign of respiratory distress in an infant. Option 2: Retractions are a sign of respiratory distress in an infant Option 3: While a cough might indicate illness it, by itself, is not a sign of respiratory distress in an infant. Option 4: Bronchovesicular lung sounds would be normal in an infant. Option 5: Grunting is a sign of respiratory distress in an infant. It is their innate way of maintaining peep or keeping their alveoli open.

An infant is diagnosed with central apnea. What symptoms might the nurse expect to see for this infant? Select all that apply. 1. Observed periods of cessation of breathing for 20 seconds or more 2. Enlarged tonsils 3. Seizure activity 4. Desaturations on the pulse oximeter 5. Primarily nasal breathing and nasal flaring

1,3,4 Option 1: Central apnea has a period of severe episodic breathing, particularly if there are more than 10 episodes per night. Option 2: This is usually indicative of obstructive apnea versus central apnea. Option 3: Seizure activity can occur specifically if the infant has a diagnosis of intraventricular hemorrhage or meningitis. Option 4: Desaturations are common with central apnea and oxygen may be required to assist with periods of hypoxia. Option 5: Nasal breathing and nasal flaring would be a sign of respiratory distress.

The nurse understands that in a child with cystic fibrosis (CF) which vitamin absorption is impaired? Select all that apply. 1.A 2.B 3.K 4.D 5.E

1,3,4,5 Malabsorption of fat soluble vitamins is likely in most patients with CF, particularly those who are pancreatic insufficient. ADEK vitamins are usually prescribed.

A five-month old who was born at 27 weeks was in severe respiratory distress. The respiratory viral panel came back positive for rhinovirus, enterovirus, and metapneumovirus. The parents state that the infant has always been a "noisy breather." Upon initial assessment severe suprasternal retractions and stridor are present. What illness might the nurse expect the child to have? 1. Laryngotracheomalacia 2. Apnea of prematurity 3. Bronchiolitis 4. Severe gastroesophageal (GE) reflux

1. The severe suprasternal retractions and stridor are indicative of laryngotracheomalacia. This should be confirmed with a bronchoscopy.

A nurse is explaining what a tracheoesophageal (TE) fistula is to a parent. Which statement is correct? 1. "It is an abnormal opening between the trachea and the esophagus." 2. "The trachea and esophagus do not form in utero." 3. "Both the esophagus and trachea end in a pouch." 4. "The trachea connects to the stomach and the esophagus to the lungs."

1. A TE fistula is an abnormal opening between the trachea and the esophagus.

Which abnormal finding indicates to the nurse that a child likely has cystic fibrosis (CF)? 1. A positive sweat chloride test 2. A positive blood culture 3. Excessive mucus production 4. Edema and weight gain

1. A positive sweat chloride test is indicative of cystic fibrosis. This test must be repeated to confirm.

Which indicates the earliest sign of hemorrhage in a child who has just had a tonsillectomy? 1. Frequent swallowing 2. Labored respirations 3. Tachypnea stridor 4. Dark brown emesis

1. Most often if hemorrhage occurs, frequent swallowing of blood is the first thing an astute nurse will notice.

Which statement does the nurse understand to be true about clients admitted with Bordetella Pertussis (Whooping cough)? 1. A paroxysmal cough might be present. 2. The client should be maintained on contact precautions/isolation. 3. Peak occurrence is in the fall and winter. 4. It is most often caused by Haemophilus Influenzae.

1. Paroxysmal "whoop" type cough is very common in pertussis.

Which are the 4 heart defects that cause an Increase in Pulmonary blood flow?

1. ASD 2. VSD 3. Patent ductus arteriosus (PDA) 4. Atrioventricular canal

57. A nurse should be most concerned about which type cyanosis in a newborn infant? 1. Cyanosis of the hands 2. Cyanosis of the feet 3. Periorbital cyanosis 4. Circumoral cyanosis

1. Cyanosis of the hands is normal for a transitioning newborn. 2. Cyanosis of the feet is normal for a transitioning newborn. 3. While abnormal in a newborn, it may indicate a transitioning newborn. 4. Circumoral cyanosis of the mouth and mucus membranes indicates central cyanosis.

3 drugs that improve cardiac function in CHF?

1. Digoxin = improve contractibility 2. ACE inhibitor = decrease afterload 3. Beta-blocker= reduce HR, BP, reduce O2 demand

14. Otitis media is a common infection children have when an upper respiratory illness is present because: 1. The Eustachian tubes are short and immature. 2. The immune system is extremely compromised and more susceptible to infections. 3. Bottle feeding increases the risk in babies. 4. All of the above are correct.

1. Eustachian tubes are short and do not provide adequate draining for mucous during an upper respiratory infection in children. 2. Immunity and susceptibility to infections cause the primary illness. Otitis media is a secondary illness. 3. A child that is positioned correctly during bottle feedings is not at an increased risk for otitis media. 4. Eustachian tubes are short and do not provide adequate draining for mucous during an upper respiratory infection in children, causing only one answer to be correct.

What are the 3 clinical manifestation of CHF?

1. Impaired myocardial function: tachycardia, sweating, Decrease UO, fatigue 2. Pulmonary congestion: retraction, tachypnea, dyspnea 3. Systemic venous congestion: weight gain, hepatomegaly, ascites, JVD

what are the 2 heart defects that cause DECREASE in pulmonary blood flow?

1. Tetralogy of fallot 2. tricuspid atresia

What are the 3 heart defects that cause obstruction to blood flow from ventricles?

1. coarctation of aorta 2. Aortic stenosis 3. Pulmonic stenosis

What are the 3 causes of cardiomyopathy?

1. dilated= floppy muscle 2. hypertrophic= stiff & overgrown muscle 3. restrictive = diastolic dysfunction

What are the 4 defects of Tetralogy of Fallot?

1. pulmonary stenosis. 2. overriding Aorta 3. ventricular septal defect. 4. hypertrophy of right ventricle.

Which assessment finding would indicate to the nurse that the plan of care for an eight-year-old hospitalized for an acute asthma attack is effective? 1. Diminished breath sounds on auscultation 2. Decreased tachypnea 3. Prolonged expiratory phase of breathing 4. Lung sounds have expiratory wheezes

2 Decreased tachypnea and work of breathing is evidence that the interventions are working and the child is getting better.

The nurse is caring for an infant who has bronchopulmonary dysplasia (BPD). What issues would the nurse associate with this diagnosis? Select all that apply 1. Term infants can develop BPD. 2. Mild forms of BPD can heal as the lungs grow and remodel. 3. Children with BPD often are poor feeders. 4. All clients with BPD will develop heart failure. 5. Pulmonary hypertension frequently occurs with BPD.

2,3,5 Option 2: This is true. As the child grows and forms new lung tissue that will replace the fibrotic lung tissue with BPD. Option 3: Infants with BPD are often poor feeders and have failure to thrive. Option 5: Pulmonary hypertension, increased lung fluid, interstitial fibrosis and smooth muscle hypertrophy occur with BPD.

A child with chronic otitis media has bilateral myringotomy tubes placed. Which statement would indicate that the parent understands education about myringotomy tubes? 1. "The tubes have to be surgically removed in 9 months or so." 2. "The tubes were placed to equalize pressure." 3. "These tubes won't affect my child being able to go swimming in the summer." 4. "My child will still need to be on Amoxicillin prophylactically for six months."

2. Myringotomy tubes relieve pressure and drain fluid from the middle ear.

The nurse is taking care of a 12-year-old client with cystic fibrosis (CF). The breakfast tray has arrived. The client is ordered pancreatic enzymes. When should the nurse administer this medication for maximal effect? 1. One hour before breakfast 2. With breakfast 3. Two hours after breakfast 4. With antibiotics

2. Pancreatic enzymes are effective for only 45-60 minutes after administration. This medication must be given at the start of a meal or snack to promote digestions and absorption of vitamins.

The nurse in a pediatric clinic is assessing a toddler brought in by a parent who states, "I have noticed some increasing incidents of coughing and wheezing over the last few weeks." Auscultation by the nurse reveals some adventitious breath sounds in the upper right lobe. Which questions does the nurse ask the parent? Select all that apply. 1. "Have you noticed any missing small toys?" 2. "How often is the child allowed to self-feed?" 3. "Is there an older child who gives this child food?" 4. "Can you recall a specific time of gagging or cyanosis?" 5. "Have you noticed any foreign objects in the child's stool?"

20. ANS: 1, 3, 4 1. This is correct. The nurse needs to ascertain if and what type of foreign object the toddler may have aspirated. This is an appropriate question if it is suspected that the child aspirated a foreign object. 3. This is correct. The nurse is appropriate in asking if there is another child who may have given the toddler food that could be aspirated. The parent may not be aware but needs to consider the possibility. Of greatest concern is peanuts, tree nuts, hard candies, etc. 4. This is correct. At the time, if the parent noticed the toddler gagging or appearing cyanotic, the nurse may have a better timeline, and the parent's memory may be jogged about a cause. 2. This is incorrect. Asking how often the child is allowed to self-feed is not an important question if the nurse suspects aspiration of a foreign object. 5. This is incorrect. Questioning the parent about noticing a foreign object in the toddler's stool is not applicable to the possibility of aspirating a foreign body. Swallowing an object may cause passage in the stool.

Which statement does the nurse understand to be true about giving palivizumab (Synagis), the immunization for respiratory syncytial virus (RSV) to an infant? 1. Children don't need to get it every year. 2. It is 100% effective in preventing RSV. 3. The immunization must be given monthly. 4. Any child can receive the vaccination.

3. The immunization must be given every month throughout RSV season (October to May).

Which statement does the nurse understand to be true about the developmental differences in the respiratory system of a child versus an adult? 1. A child's airway is shorter but wider than an adult's. 2. A child's intercostal muscles are fully developed at birth. 3. Newborns are obligatory nose breathers until about four weeks of age. 4. The cartilage around a child's trachea is less flexible than an adult's.

3. This is true, newborns ARE obligatory nose breathers until four weeks of age.

The parent of a 7-year-old girl who has just received her second influenza vaccine ask when she will be protected from the flu. What answer should the nurse give? 1. Immediately after vaccination 2. 5 days 3. Two weeks 4. 1 month

3. A child will be protected from the flu 2 weeks after receiving the second vaccine

The nursing student asks the nurse about genetic implications related to cystic fibrosis (CF). How should the nurse respond? 1. "It is inherited as an autosomal dominant trait." 2. "It is a genetic defect found primarily in non-Caucasian people." 3. "If it is present in a child, both parents are carriers of the defective gene." 4. "There is a 50% chance the siblings of an affected child will also be affected."

3. Both parents must be carriers of the defective gene for it to be present in the child.

A six-week-old is admitted for respiratory syncytial virus (RSV). The infant has expiratory wheezes, increased work of breathing, substernal and subcostal retractions. Oxygen saturations is 87% on room air. An intravenous line (IV) is placed and the infant is placed on 2 liters via nasal cannula. Nursing care for this infant should include: 1. Administering Acyclovir intravenously 2. Giving morphine as needed for pain 3. Providing nebulized hypertonic saline 4. Giving Palivizumab (Synagis) prior to discharge

3. Nebulized hypertonic saline has been proposed as a potentially effective therapy for acute bronchiolitis in infant. It decreases airway edema, unblocking mucus plugging, and improving mucociliary clearance.

The nurse is caring for a child who swallowed a quarter. A bronchoscopy with retrieval is planned for later that day. Which assessment finding would be most concerning? 1. The child develops a fever of 101. 2. Intermittent audible stridor is noted. 3. The respiratory rate is 45 with the increased work of breathing. 4. Oxygen saturations drop to 96% on room air.

3. This is the most concerning. The child has an increased respiratory rate and is working harder to breathe. They physician should be notified immediately.

A nine-month-old is admitted with influenza. Which statement made by the nurse would be the best response when caring for this infant? 1. "This infection could have been prevented if the parents washed their hands at home." 2. "Antibiotics should be given as soon as a diagnosis is made to prevent further infection." 3. "Supportive care such as encouraging fluids to liquefy secretions will help prevent dehydration." 4. "Antiviral medications such as Oseltamivir (Tamiflu) can be given at any time during the illness

3. Yes. Encouraging fluids will help liquefy secretions and assist in preventing dehydration. If adequate oral fluids cannot be taken, intravenous fluids should be started.

At what age should annual influenza vaccines first be given to children? 1. Newborn 2. Two weeks 3. Three months 4. Six months

4

The nurse is providing discharge teaching to the mother of a four-year-old who had a tonsillectomy. Which statement made by the mother indicates she has a good understanding of what to feed her child when they arrive home? 1. "I will give her whatever she wants so she eats something." 2. "She likes to eat pretzels & tortilla chips for a snack. I will give that to her if she asks." 3. "I will give her cool apple juice and orange juice to drink to help her throat." 4. "I will give her Italian ice and yogurt initially when she gets hungry."

4. "I will give her Italian ice and yogurt initially when she gets hungry." Cool ices, and popsicles as well as soft foods like yogurt are ideal in the initial period and just after a tonsillectomy.

A four-year-old is presented to the urgent care center with a history of a sudden onset of a severe sore throat. He began drooling and has difficulty swallowing. The temperature is 102.2F (39.0C). Lung sounds are clear and there is no cough. The child is very anxious and flushed and is leaning forward in a tripod position. Based on these symptoms, the nurse anticipates a diagnosis of: 1. Acute Asthma Attack 2. Laryngotracheomalacia 3. Acute laryngotracheobronchitis (Croup) 4. Acute Epiglottitis

4. A sudden onset of high fever, sore throat with the four D's (drooling, dysphagia, dysphonia, distressed air movement; stridor), and the tripod position are classic signs of acute epiglottitis

A 13-year-old diagnosed with asthma is learning about their medication regime. Which statement indicates a need for further education regarding the management of their chronic asthma? 1. "I'll need to keep my bedroom cleaner and dusted." 2. "I will keep my Albuterol with me for quick treatment." 3. "I can still try out for the baseball team." 4. "I should take my Montelukast (Singulair) for a sudden onset of wheezing."

4. Singulair is a long-term maintenance medication. It should not be used for a sudden onset of wheezing.

The nurse is caring for a child who has cystic fibrosis (CF). Which type of diet does the nurse anticipate will be ordered for this child? 1. High fat, high salt 2. High carbohydrate, high calorie 3. High sodium, high fat 4. High calorie, high protein

4. This is best diet for a CF client with high metabolic needs.

The nurse is assigned to take care of a five-month-old with Respiratory Syncytial Virus (RSV). The baby presents with a temperature of 102.2o F rectally, HR 165, RR, 72, and O2 saturation of 96% on room air. The best initial intervention for the nurse to do would be to: 1. Make sure the baby eats to maintain hydration. 2. Place an intravenous catheter (IV) and promptly and hang IV antibiotics ASAP. 3. Put the infant on 4 liters of oxygen via nasal cannula. 4. Suction the nares bilaterally frequently.

4. This is the most useful and best initial intervention to maintain a patent airway and facilitate breathing.

69. The parents of a 3 year old from India state that the child has been losing weight and coughing for a year. Additionally, the childs grandmother was diagnosed with TB. Which of the following is the most accurate statement regarding this situation? 1. Tubercules are more prevalent in children than adults, and all family members should be tested for TB. 2. Prevalence is high in developing countries, and only 20 percent of complete treatment because the length, intensity, and cost of treatment. 3. A blood culture is the definitive diagnosis for TB after a negative skin test. 4. Diagnosing TB in children is difficult because it varies with the changes in the seasons, and the symptoms can be vague.

68. An infant is tachypneic, retracting, and tachycardic with a temp of 39.0 C and a pulse oximetry of 92 percent. You place the infant on 1L nasal cannula oxygen and raise the head of the bed. What intervention would the nurse expect the physician to order next? 1. MRI 2. CT 3. Bronchoscopy 4. Chest x-ray

The nurse understands that, in a child with cystic fibrosis (CF), which vitamin absorption is impaired? Select all that apply. a. A b. B c. C d. D e. E

A D E

The nurse educator has just completed a lecture on congenital heart defects and its symptoms. The educator knows that teaching was effective when a student defines central cyanosis as what? A reddish discoloration of the mucous membranes, tongue, circumoral area, or core body A bluish discoloration of the mucous membranes, tongue, circumoral area, or core body A bluish discoloration of the hands or feet Decreased hemoglobin levels

A bluish discoloration of the mucous membranes, tongue, circumoral area, or core body

The nurse is providing care for a neonate diagnosed with tetralogy of Fallot. Prostaglandin E1 therapy is prescribed to keep the foramen ovale and the ductus arteriosus open. Which is the most important intervention for the nurse to include in the neonate's plan of care? 1. Maintain a separate IV access for continuous administration of the medication. 2. Watch for respiratory distress or apnea after adding medication to the breathing tube. 3. Monitor for and document evidence of flushing, bradycardia, and irritability as expected. 4. Monitor weight and adjust the dosage using a scale of 0.05 to 0.1 mcg/kg/min IV infusion.

ANS 1 1 This is correct. During the administration of prostaglandin E1, the nurse starts and maintains a separate IV access for continuous administration of the medication. 2 This is incorrect. During the administration of prostaglandin E1, the nurse monitors for respiratory distress or apnea. However, the medication is not administered via the neonate's breathing tube. 3 This is incorrect. During the administration of prostaglandin E1, the nurse will watch the neonate for flushing, bradycardia, irritability, and diarrhea, and monitor for bleeding. The nurse always documents assessment findings; however, the manifestations need to be reported to the primary health-care provider. 4 This is incorrect. The dosage for prostaglandin is usually 0.05 to 0.1 mcg/kg/min IV infusion, as ordered by the primary health-care provider. The nurse does not adjust the dose using any parameter.

The nurse is providing patient teaching for a school-age patient and parents. Chemotherapy is prescribed for cancer treatment. Before the first dose is administered on an outpatient basis, which teaching is most important for the nurse to provide? 1. Prompt recognition of adverse effects after the therapy. 2. Explain how to immediately initiate protective precautions. 3. Promote nutrition with the preparation of favorite foods. 4. Have icy drinks available to improve oral fluid intake.

ANS 1 1 This is correct. It is most important for the patient and family to recognize adverse effects to the chemotherapy and seek medical assistance as directed. 2 This is incorrect. Protective precautions are explained only as needed. 3 This is incorrect. Promoting the preparation of the patient's favorite food may or may not be necessary. Some favorite foods can be available after the first treatment if needed or desired. 4 This is incorrect. During chemotherapy, patients may experience mouth sores and have difficulty eating and drinking. Some patients like cold liquids, and others prefer warm liquids. It is unlikely that this is an issue after the first chemotherapy treatment.

A 3-month-old infant is diagnosed with pulmonary stenosis. Which parent teaching does the nurse provide? 1. Options for treatment include a repair of the artery or the valve. 2. Balloon angioplasty is performed as an outpatient procedure. 3. Pulmonary stenosis repair can be delayed until 1 year of age. 4. After repair, the child is no longer at risk for cardiac problems.

ANS 1 1 This is correct. Options are to repair the pulmonary artery and/or pulmonic valve as soon as possible to avoid worsening side effects of the condition. 2 This is incorrect. Side effects of pulmonary stenosis include increased workload of the right ventricle, CHF, hepatomegaly, development of murmurs, shortness of breath, and cyanosis. Delaying corrective surgery will worsen effects of the condition. 3 This is incorrect. The infant with pulmonary stenosis will be hospitalized for assessment and treatment. 4 This is incorrect. The child must return to the cardiologist frequently for follow-up, and caregivers should be provided information on the recurrence of symptoms that may occur due to restenosis.

The nurse is counseling a couple with a 3-year-old toddler diagnosed with hemophilia after experiencing excessive bleeding after a minor injury from a fall. Which teaching is most important for the nurse to provide to the parents? 1. Stressing the need for preventing injury in the environment with supervision, helmet use, and activity restrictions 2. Sharing the availability of resources from the National Hemophilia Foundation publications 3. Preparing them for the possibility of altered family dynamics and how illness may impact financial resources 4. Explaining the importance of allowing the toddler to grow up as normally as possible

ANS 1 1 This is correct. Preventing injury in the environment with supervision, helmet use, and activity restrictions is the most important teaching for the nurse to provide to the family. The toddler will be especially challenging because of normal development, which places the toddler at a high risk for injury. 2 This is incorrect. Sharing the availability of resources from the National Hemophilia Foundation publications is important; however, another topic of teaching is more important. 3 This is incorrect. Helping the parents prepare for the possibility of altered family dynamics and how the illness may impact financial resources is important; however, the prevention of injury is most important. 4 This is incorrect. It is important for the nurse to teach the parents about allowing the toddler to grow up as normally as possible, but the nature of hemophilia will grossly alter the toddler's future activities, such as the avoidance of contact sports. There is another topic of more importance.

5. The nurse is assisting with high school sports physicals. The nurse performs a physical assessment on a male student who is tall and thin, with disproportionately long arms. Which additional finding will prompt the nurse to recommend a cardiac evaluation? 1. Notable laxity of joints 2. Sparsity of body hair 3. Deep tone to the voice 4. Slow, rhythmic gait

ANS 1 1 This is correct. Tall and thin with arms disproportionately long and with laxity of joints are physical manifestations of Marfan syndrome. The nurse may also notice dislocation of lenses, spinal problems, stretch marks, hernia, pectus abnormalities, and/or restrictive lung disease. Marfan syndrome is also associated with aortic aneurism as well as aortic and/or mitral regurgitation. 2 This is incorrect. A sparsity of body hair is not an indication of cardiac issues; male high school students may not exhibit heavy body hair. 3 This is incorrect. A male high school student is expected to have a deep tone to his voice. 4 This is incorrect. A slow, rhythmic gait may be unique to this student; however, it does not support the presence of Marfan syndrome.

The nurse is providing care for a 4-year-old child diagnosed with non-Hodgkin's lymphoma. The nurse is aware that multiple blood samples are ordered, and the child is scheduled for a lumbar puncture with bone marrow aspiration. Which intervention will the nurse plan to prepare the patient for the testing? 1. Use simple terms and provide a needless syringe for treatment on a stuffed animal. 2. Provide information to the child's parents and encourage them to explain procedures. 3. Withhold details from the patient until right before testing procedures are performed. 4. Expect that a 4-year-old patient is incapable of understanding or cooperating during procedures.

ANS 1 1 This is correct. The nurse needs to allow for medical play when age appropriate; a needleless syringe for stuffed animal treatment is a good choice for a 4-year-old child. 2 This is incorrect. The nurse will explain procedures to the patient's parents; however, it is the nurse's responsibility to provide teaching to the patient and ask for parenteral support. 3 This is incorrect. Even at the age of 4 years, the patient is much more likely to be cooperative with some preparation prior to the testing. 4 This is incorrect. The nurse can teach the patient on an age-appropriate level and elicit more cooperation than without teaching and preparation.

The nurse is aware the neonate's blood circulation is different before birth than after birth. Which circulation pattern does the nurse recognize as occurring prior to birth? 1. Oxygenated blood flows from the right atrium to the left atrium through the foramen ovale. 2. Oxygenated blood flows from the right ventricle to the lungs and then to the left ventricle. 3. For a short time after birth, the neonate continues to depend on the mother for oxygen supply. 4. Once the neonate takes a first breath, the ductus venosus closes and blood goes to the lungs.

ANS 1 Feedback 1 This is correct. Prior to birth, oxygenated blood crosses from the right atrium to the left atrium via the patent foramen ovale (PFO) and is pumped by the left ventricle. 2 This is incorrect. Prior to birth, oxygenated blood does not flow from the right ventricle to the lungs and then to the left ventricle. 3 This is incorrect. Before birth, 90% of blood bypasses the lungs; the placenta is the organ of respiration. After the cord is cut and the placenta is delivered, the infant is expected to independently breathe. 4 This is incorrect. Upon birth and first breath, the foramen ovale and ductus arteriosus close.

The nurse is providing care for an infant with an inner ear infection. The nurse is aware that the condition has occurred multiple times in a 3-month period. Which comment by the parent indicates to the nurse that specific teaching is needed regarding the incidences of infection? 1. "I now put her to bed with a bottle." 2. "I clean her ears with cotton swabs." 3. "She likes her ears submerged while bathing." 4. "Her older brother brings colds home from school."

ANS 1 This is correct. Eustachian tubes are shorter and more horizontal in children than adults and are more prone for migration of substances in the mouth to the inner ear, causing an inner ear infection. The migration of milk is enhanced by putting an infant to bed with a bottle. The nurse will present teaching on this topic. The nurse will reinforce that cotton swabs should not be placed in a child's ear. However, the practice is less likely to be the cause of an inner infection. Some infants do not mind having their ears submerged when in a bath. This practice alone is not likely to be the cause of an inner infection. An infant may be susceptible to an upper respiratory infection from an older sibling who attends school. However, the scenario does not present other symptoms of a cold, such as a stuffy nose.

An adolescent 16 years of age and the parents and health-care team agree that the patient is to undergo HSCT. Which specific nursing interventions will be included for the care of this patient? Select all that apply. 1. Sterility of the central line must be maintained. 2. High-dose chemotherapy and/or total body irradiation is administered. 3. IV for the administration of stem cells is established and maintained. 4. Fluid loss from urine, vomiting, and/or diarrhea is closely monitored. 5. Anti-T-cell immunotoxins are administered immediately after the transfusion.

ANS 1,2,3 1. This is correct. Strict asepsis with central venous catheter care is a critical intervention that all health-care providers must follow. 2. This is correct. Chemotherapy will treat the diagnosed cancer and will also reduce the immune response. The reduction of immune responses will aid in acceptance of the transplanted stem cells. 3. This is correct. It is important to carefully initiate, monitor, and maintain the IV transfusion of the stem cells harvested from bone marrow, peripheral blood, or the umbilical vein of the placenta. 4. This is incorrect. Monitoring both intake and output will provide information about kidney function and fluid balance. Vomiting and diarrhea is not expected during HSCT; however, signs and symptoms of chronic GVHD occur about 100 days after transplant and affect the liver, gastrointestinal system, oral mucosa, and lungs. 5. This is incorrect. High-level nutritional intake must be provided to encourage healing and the body's acceptance of the transplanted cells.

The nurse case manager is providing care and support to a mother with AIDS who has an infant who also tests positive for the condition. Which statement by the mother indicates to the nurse that additional teaching is needed? Select all that apply. 1. "I wish I could control the baby's pain without so much medication." 2. "I am so looking forward to the time when the baby is no longer infectious." 3. "I absolutely do not want the day-care staff to know about the baby's condition." 4. "If I am to become a better mother, I need to work on my self-esteem and self-value." 5. "I keep hoping every day that a cure for my baby and me can be found."

ANS 1,2,3 1. This is correct. The nurse needs to teach pain management techniques and reinforce that tolerance to opioids may require increased dosing. The nurse also needs to teach and encourage use of nonpharmacological pain interventions. 2. This is correct. When the mother expresses the expectation that the infant will eventually be noninfectious, additional teaching by the nurse is needed. 3. This is correct. When the mother expresses opposition to the day-care staff knowing about the infant's condition, additional teaching is needed. The nurse needs to support the mother with making complex day-care arrangements. The nurse also has a responsibility to educate day-care and school staff on current AIDS information. 4. This is incorrect. The nurse needs to encourage positive self-concept and suggestions of how to avoid the HIV-related stigma. The nurse needs to provide support but not necessarily teaching. 5. This is incorrect. The hope expressed by the mother for a cure for herself and her infant does not require additional teaching by the nurse.

The nurse works in a pediatric clinic with patients who have heart disease or who have undergone treatment for heart disease. The nurse is frequently asked about the need for prophylactic antibiotics for these patients during invasive dental care. Which patients does the nurse identify as being at risk for endocarditis and being in need of antibiotic therapy? Select all that apply. 1. A child with a prosthetic heart valve 2. A child with a congenital defect scheduled for surgical repair 3. A child who was previously diagnosed with endocarditis 4. A child with a confirmed diagnosis of rheumatic heart disease 5. A child who had a congenital heart defect repaired 9 months ago

ANS 1,2,3,4 1. This is correct. The American Heart Association recommends prophylactic antibiotics for the child with a valve replacement. 2. This is correct. The American Heart Association recommends prophylactic antibiotics for the child with a congenital defect who is scheduled for surgical repair. 3. This is correct. The American Heart Association recommends prophylactic antibiotics for the child who was previously diagnosed with endocarditis. 4. This is correct. The American Heart Association recommends prophylactic antibiotics for the child who is diagnosed with rheumatic heart disease, which affects the heart valves. 5. This is incorrect. Prophylactic antibiotics are recommended for the child with a congenital heart defect that has been corrected for the first 6 months after the repair. The child who is 9 months past the repair does not need prophylactic antibiotics.

The nurse is providing care for a 9-year-old patient who was recently diagnosed with cardiomyopathy after a viral infection. Which teaching does the nurse provide to the patient's parents about the diagnosed condition? Select all that apply. 1. Need for intensive care of the patient 2. Preparation for anticipatory grieving 3. The necessity for physical activity 4. Allowing patient to discuss feelings 5. Reasons for frequent medical visits

ANS 1,2,4,5 1. This is correct. The nurse will provide teaching about the intensive care necessary for a child with this life-threatening condition. 2. This is correct. The patient with cardiomyopathy has a possible terminal status. The nurse will prepare the parents for expected anticipatory grieving, which is expected with this life-threatening condition. 3. This is incorrect. The nurse will provide teaching about the activity restrictions to prevent overstimulation of the heart. 4. This is correct. The nurse will instruct the parents to allow the child to discuss feelings concerning the restriction of activity in the previously active child. 5. This is correct. The parents need to understand that frequent echocardiograms will be required to monitor the size and function of the heart.

During the treatment of a preschool child for anemia, laboratory tests reveal the child is also positive for lead poisoning. The child is currently living in an older home being renovated by the parents. Which teaching does the nurse provide to the parents? Select all that apply. 1. The possibility of removing the child from the environment 2. Methods to remove paint chips or dust from the environment 3. Details about behavior changes indicating additional exposure 4. Recommendation of foods that will decrease absorption of lead 5. The need for a child development specialist to evaluate the child

ANS 1,2,5 1. This is correct. It is very important that the child diagnosed or being treated for lead poisoning be removed from the contaminated environment. 2. This is correct. It is very important that the parents know how to remove lead from the environment. The recommended method is to remove paint chips or dust with a damp paper towel and discard it in the trash. Use another wet paper towel to clean the surface and discard the paper towel. 3. This is incorrect. Central nervous system signs of lead poisoning include hyperactivity, impulsiveness, lethargy, irritability, loss of developmental progress, hearing impairment, and learning difficulties. However, the goal is to avoid exposure. 4. This is incorrect. Iron-deficiency anemia causes a greater level of lead absorption, but correcting the anemia does not prevent the child in the event of additional exposure. 5. This is correct. Because of the impact of lead poisoning on the development of the child, the parents need to know the importance of having a child development specialist evaluate the child as needed.

A new mother brings her 2-week-old neonate to the pediatrician's office, stating, "I think something is wrong with my baby." When the infant is undressed, the nurse notices signs of possible cardiac problems. Which assessment findings support the nurse's suspicions? Select all that apply. 1. Prolonged capillary refill time 2. Bluish tinge to oral structures 3. Peripheral cyanosis of left leg 4. Amount of urinary output 5. Mottled appearance of skin

ANS 1,2,5 1. This is correct. Prolonged capillary refill time is a sign of poor perfusion; it can be indicative of possible cardiac or lung issues. The assessment is made by pressing on the sternum or forehead. 2. This is correct. Central cyanosis is the presence of bluish discoloration of mucous membranes, tongue, circumoral, or core body and is due to problems with the heart or lungs. 3. This is incorrect. Peripheral cyanosis (acrocyanosis) is often due to interruption in blood flow to the extremity. 4. This is incorrect. Urine output is indicative of perfusion to the kidneys; however, the nurse is unable to determine the amount of urinary output through observation. 5. This is correct. Skin that is pale, mottled, or gray in appearance indicates poor perfusion.

A neonate became dusky and developed respiratory distress at the age of 4 days and is diagnosed with a hypoplastic left heart. The surgeon obtains an informed consent from the parents to perform emergency surgery. Which information will the nurse provide to promote parental understanding? Select all that apply. 1. The left side of the neonate's heart did not develop correctly. 2. The function of left side of the heart is to pump blood to the body. 3. Provide reassurance to the parents that surgery will fix the problem. 4. Share that medical management of the condition is needed for one year. 5. A normally existing hole in the wall of the heart at birth will be enlarged.

ANS 1,2,5 1. This is correct. The hypoplastic left heart is the second most common congenital heart defect, caused by underdevelopment of the left side of the heart, aorta, aortic valve, left ventricle, and mitral valve. 2. This is correct. The nurse is correct in providing information about the normal functioning of the left side of the heart to clarify the existing condition. 3. This is incorrect. The nurse needs to provide the parents with information about the severity of the condition. The nurse anticipates possible transport out of the facility for heart transplant. 4. This is incorrect. The neonate will require clinical and medication management for life. 5. This is correct. Symptoms appear when the PDA closes; the condition is usually fatal within the first days or months of life unless treated. It is appropriate for the nurse to describe the surgery in terms the parents can understand.

The pediatric nurse is providing care for a school-age patient diagnosed with rheumatic heart disease. When developing a plan of care for the patient's hospital stay, which interventions does the nurse include? Select all that apply. 1. Administer inflammatory and antibiotic medications as prescribed. 2. Assess for the presence of strep throat or other throat infections. 3. Include chest pain and heart palpation assessment with vital signs. 4. Begin patient/family teaching about the possibility of surgery. 5. Organize daily care and treatments to provide for joint pain relief.

ANS 1,3,5 1. This is correct. The nurse will address interventions for the administration of inflammatory and antibiotic medications as prescribed. 2. This is incorrect. The nurse does not need to add assessment interventions for strep throat or other throat infections in a patient diagnosed with rheumatic heart disease. 3. This is correct. The nurse will include the intervention of including assessment for chest pain or heart palpitations with vital signs, because rheumatic heart disease causes heart valve damage. 4. This is incorrect. The scenario does not specify the reason for the patient's hospitalization. Patient/family teaching is not initiated until surgery is planned. 5. This is correct. Rheumatic heart disease can cause polyarthritis; it is necessary for the nurse to include interventions that will promote rest and relieve joint pain.

The nurse in a pediatric oncology unit understands that painful tests and treatments are common for children with cancer. Which interventions does the nurse decide to use to help manage pain for a 6-year-old patient scheduled for diagnostic testing? 1. Avoid too many details that may scare the child. 2. Use language that is age appropriate for the child. 3. Promise the child a toy for cooperating and not crying. 4. Explain why the test is performed without pain medication.

ANS 2 1 This is incorrect. Consider the developmental level of the child and use words he or she can understand. 2 This is correct. State what the child will hear, see, feel, touch, and smell. The unknown is more frightening to a child than experiencing expected happenings, equipment, and procedures. 3 This is incorrect. Provide a special box of toys to have a treat to look forward to after the procedure. Tell the child that it is okay to cry. 4 This is incorrect. Anesthesia may put the child to sleep with a mask or IV medication. Administer medication for any pain, including headache, as needed.

The parents are preparing to take their newborn, who was diagnosed with tetralogy of Fallot with pulmonary atresia, home. The nurse is developing a teaching sheet regarding care of the newborn for the parents. Which information does the nurse need to include in the teaching plan? 1. There is no need to limit activities. 2. It is important to maintain caloric intake. 3. No secondary complications are expected. 4. The neonate has natural immunity to infections.

ANS 2 1 This is incorrect. The parents will be taught to calm the infant by holding the infant over the caregiver's shoulders with the infant's knees drawn up toward the chest. This will increase the blood flow to the lungs. The parents will limit cardiac stress by keeping the infant calm. 2 This is correct. Due to a clinical finding of failure to gain weight, the parents are instructed on the importance of maintaining caloric intake. Frequent small feedings are necessary to meet this need and not increase cardiac stress. 3 This is incorrect. Parents need to monitor for signs and symptoms of heart failure. 4 This is incorrect. Educate parents on the increased risk of bacterial endocarditis and the prescribed medication regimen.

A 19-year-old college student is being treated for non-metastatic Hodgkin's lymphoma with chemotherapy. Which recommendation by the nurse was made prior to beginning chemotherapy in order to promote a level of health? 1. Gain weight before treatment to offset weight loss. 2. Consider sperm banking because of expected sterilization. 3. Move in with parents to initiate quarantine protocols. 4. Withdraw from all college courses because of fatigue.

ANS 2 1 This is incorrect. The patient is likely to be experiencing weight loss from anorexia triggered by the disease process. Weight gain is unlikely; other nutrition recommendations are needed. 2 This is correct. Treatment with chemotherapy is likely to cause sterilization; the nurse needs to recommend sperm banking. Patients may not be able to think about a future and children; the nurse will explain the importance of having sperm at a future time. 3 This is incorrect. If the patient is living independently, there is no need to move in with parents unless care assistance is needed. The patient can initiate quarantine measures if necessary, which may be easier in an independent environment. However, other care needs may be met if living with caregivers. 4 This is incorrect. It is likely the patient will experience fatigue from chemotherapy. However, the patient may benefit from activities focused on something other than the illness and treatment. The patient may want to consider one class a term and the availability of computerized classes.

The nurse is preparing teaching materials for parents with children diagnosed with anemia. Which information will be marked as being specifically for the child diagnosed with sickle cell anemia? 1. Follow a balanced nutritious diet. 2. Encourage drinking as much fluids as possible. 3. Allow low-energy activities and rest periods. 4. Make sure that immunizations are up to date.

ANS 2 2 This is correct. The child with sickle cell anemia is encouraged to drink as much fluid as possible to help avoid the clumping of sickled cells. Hydration can be accomplished with IV therapy and oral fluids. Children with iron-deficiency anemia need a normal daily fluid intake. 1 This is incorrect. The nurse recommends a balanced nutritious diet for all children, especially those with anemias. 3 This is incorrect. Anemias can cause fatigue; children should participate in low-level activities, as able. Caretakers will need to provide for adequate rest periods. 4 This is incorrect. Anemias put children at risk for infections; immunizations need to be up to date. Influenza and pneumonia vaccines are strongly recommended.

The nurse is providing care for a neonate diagnosed with a cardiovascular disorder immediately after birth. When gathering assessment information from the mother, which comment will the nurse recognize as the most likely contributing factor for the defect? 1. "We live in the country, and we get all our water from a well." 2. "I quit my preschool job when a child was diagnosed with measles." 3. "The baby was born a week early; I hope that is not the cause." 4. "We were in a European country before pregnancy was confirmed."

ANS 2 2 This is correct. The nurse is most likely to contribute exposure to a child with measles as a causative factor for the neonate's heart defect. The nurse will assess further for timelines and manifestations. 1 This is incorrect. Using a well as a water source does not necessarily place a neonate at risk for developing a heart defect. 3 This is incorrect. The mother's concern about the neonate being born a week early is not recognized as a contributing factor to the neonate's heart defect. 4 This is incorrect. Traveling to a foreign country during the first trimester may or may not effect fetal development. The nurse needs additional information before making a connection between the travel and heart defect.

The nurse is providing care to two children on a pediatric unit. One child is diagnosed with iron-deficiency anemia, and the other has sickle cell disease. Which manifestation does the nurse recognize as being different between the two children? 1. A child with iron deficiency expresses significant pain and discomfort. 2. A child with sickle cell disease experiences varying amounts of joint pain. 3. A child with iron-deficiency anemia experiences normal physical growth. 4. Sickle cell disease is transmitted as a dominant trait from one parent.

ANS 2 2 This is correct. Vaso-occlusive crisis (obstruction of blood flow causing tissue hypoxia and necrosis) is a painful episode with hand-foot syndrome (dactylitis), causing symmetrical infarct in the bones of the hands and feet along with very painful swelling of soft tissue. 1 This is incorrect. A child with iron-deficiency anemia may experience irritability, anorexia, tachycardia, systolic murmur, brittle and concave nails, and poor muscle tone, and may be prone to infection. Significant pain and discomfort is not expected. 3 This is incorrect. A child with either iron-deficiency anemia or sickle cell disease may experience delayed growth and development. 4 This is incorrect. When both parents have the recessive sickle cell trait, there is a 1 in 4 chance with each pregnancy that the child will have sickle cell disease (SCD). About 1 in 13 African American babies are born with this sickle cell trait.

The nurse is performing a physical assessment on a 7-year-old child as a requirement for playing a sport at school. The nurse reports which assessment finding as abnormal and requests a follow-up from a primary care physician? 1. Systolic blood pressure is 84 mm Hg. 2. Systolic blood pressure is 90 mm Hg and diastolic is 20 mm Hg. 3. Pulse oximeter reading is 95% on room air. 4. PMI is at 4th or 5th intercostal space at the midclavicular line.

ANS 2 2 This is correct. Wide pulse pressures—diastolic pressures are low, with a wide gap between diastolic and systolic pressures—are indicative of such processes as patent ductus arteriosus. 1 This is incorrect. Average systolic blood pressures are considered 70+ (2´ age in years). This would be 84. 3 This is incorrect. Pulse oximeter readings of 95% of room air are considered normal. 4 This is incorrect. The point of maximum impulse (PMI) is heard at the fourth or fifth intercostal space at the midclavicular line (MCL).

The nurse on a pediatric acute care unit is providing care for an infant who is 11 months of age. The infant is diagnosed with a lower respiratory infection that produces large amounts of thick secretions the infant cannot cough up. When planning to suction the infant, which factor is important to remember? 1. The parents are likely to become angry about the procedure. 2. The infant will respond negatively to a temporary loss of breath. 3. The nurse should apologize after the procedure for stress related to the procedure. 4. The nurse should have assistance to immobilize the infant during the procedure.

ANS 2 During the suctioning, the nurse will hold the infant tightly and the infant will temporarily experience a loss of breath. During the suctioning, the nurse will hold the infant tightly and the infant will temporarily experience a loss of breath. Providing emotional support and verbal explanation of the procedure to both child and parent is crucial for establishing a good rapport; however, the explanation and support needs to occur before the procedure. The nurse should be able to manage an infant during the process of suctioning. Additional assistance may cause additional stress and should only be acquired if necessary.

A toddler who is 2 years old is playing in the playroom at the hospital and suddenly begins to choke and cough. The nurse attending the toddler places the child in which position to dislodge a possibly inhaled object? 1. Head down and on the left side 2. Head down and on the right side 3. Head horizontal to the floor and supine 4. Head in a neutral position and prone

ANS 2 In children, the bifurcation of the right and left bronchi occurs higher in the airway, and the right bronchus enters the lung at a steeper angle than does the bronchi of an adult. Placing the child head down and on the right side will help to dislodge the object. The right bronchus is more likely to be blocked because of its steep angle into the lungs; turning the child to the left will impede the removal of the object. The object needs to move up and out of the bronchus; it cannot do that lying flat or horizontal. Lying prone and in a neutral position will not enable the object to be coughed up and out.

Parents bring a toddler who is 2-1/2 years old to the hospital because of observed difficulty with breathing. In addition, they share that at bedtime the toddler has a barky cough. The toddler is diagnosed with laryngotracheobronchitis, commonly referred to as croup. Which assessment finding does the nurse expect related to the diagnosis? 1. Fever accompanied by a congested cough 2. Inspiratory stridor heard in the upper airway 3. Elevated temperature and diaphoresis 4. Snoring sounds throughout respirations

ANS 2 Inspiratory stridor, hoarseness, and air hunger are attributed to croup. Although fever accompanies croup, the cough is not congested but dry and barky. Although an elevated temperature may accompany croup, diaphoresis or sweating does not. Snoring (rhonchi) is low-pitched sounds heard throughout the respiration as air passes through thick secretions. Snoring does not occur with croup.

A 6-year-old patient is being assessed by the pediatrician for breathing difficulties. The pediatrician expresses a need for diagnostic tests to identify or rule out asthma. Which tests does the nurse anticipate ordering? 1. Throat culture 2. Pulmonary function tests 3. Electrocardiogram 4. Peak flow meter

ANS 2 This is correct. Pulmonary function tests measure the volumes of inhalation and exhalation, normal and forced. The test evaluates the effectiveness of the lungs. Electrocardiograms record the electrical activity of the heart, not the function of the lungs. Peak flow meters are portable, handheld devices that measure the ability to push air out of the lungs. It is not diagnostic but is used for monitoring therapy. The parameters range from below "personal best" to the personal best level measured when in optimal health. The need for interventions is based on color zones.

A pediatric patient has acute myelocytic leukemia, and patient and family are considering treatment options aimed at long-term success. Which contemplated treatment involves using the patient's stem cells collected before radiation of cancerous bone marrow? Select all that apply. 1. Allogenic 2. Autologous 3. Peripheral blood stem cells (PBSC) 4. Umbilical cord stem cells 5. Platelet transfusion

ANS 2,3 1. This is incorrect. Allogenic transplantation comes from another person rather than from the patient. 2. This is correct. Autologous cells are the cells that were drawn from the patient. 3. This is correct. Peripheral blood stem cells can be drawn from the patient and are therefore autologous. 4. This is incorrect. Umbilical cord stem cells come from a newborn infant and not the patient. 5. This is incorrect. Stem cells are those initial (parent) cells from which other blood cells are grown. Platelets are one of the products of cell formation.

A toddler who is 2 years of age is cared for by a grandmother because of the death of the toddler's mother from AIDS. A critical nursing intervention is for the nurse to provide home-care instructions. Which points should the nurse emphasize? Select all that apply. 1. Hand hygiene precautions during hospitalization 2. Importance of keeping up to date with all childhood immunizations 3. Proper nutrition for a toddler diagnosed with the disease 4. Monitoring playmates in order to avoid childhood viruses 5. Prevention of bacterial infections as the treatment focus

ANS 2,3,4 1. This is incorrect. The nurse will emphasize the importance of following standard precautions, such as hand hygiene, at all times when providing care. 2. This is correct. Immunization against common childhood illnesses is required if the toddler is exposed to HIV/AIDS or tests positive for either condition. 3. This is correct. Nutritional management with high-calorie, nutrient-dense foods is critical to putting the infant's immune system in the best possible state to fight off infections. 4. This is correct. Prevention and management of opportunistic infections (OPs) is essential for children with severe immune suppressions. Playmates will be excluded from contact with the toddler if the playmates are sick. 5. This is incorrect. Administration of highly active antiretroviral therapy drug (HAART) combination therapy (a strategy analogous to the treatment of infectious diseases) has improved efficacy, minimized toxicity, and delayed drug resistance. The treatment of the toddler is not just focused on bacterial infections.

The nurse is providing care for an adolescent female who just gave birth to a neonate. The mother tests positive for HIV; however, the mother did not receive prenatal care. Based on the nurse's understanding about HIV/AIDS, which interventions does the nurse expect? Select all that apply. 1. Placement of the neonate in foster care because of deficient parenteral care 2. Performance of HIV polymerase chain reaction (PCR) test on the neonate 3. Recommended virological diagnostic testing for the neonate 4. Immediate immunization of the neonate against common childhood illnesses 5. Nutritional management that includes high-calorie, nutrient-dense foods

ANS 2,3,5 1. This is incorrect. Neonates born to HIV-positive mothers are not routinely placed in foster care. The scenario does not present enough information to determine deficient parenteral care of the neonate. 2. This is correct. HIV polymerase chain reaction (PCR) is used for detection in infants born to HIV-infected mothers because of the presence of maternal antibodies transferred transplacentally; preferred virological assays include HIV DNA PCR and HIV RNA assays. 3. This is correct. Virological diagnostic testing is recommended at birth in infants at high risk for HIV infection (e.g., infants born to HIV-infected mothers who did not receive prenatal care or prenatal antiretroviral therapy). 4. This is incorrect. Immediate immunization is not recommended; however, immunization against common childhood illnesses is strongly recommended if exposed to HIV. The immunizations will be given when age appropriate. 5. This is correct. The nurse will expect initiation of nutritional management that includes high-calorie, nutrient-dense foods.

A parent brings a 2-year-old child with a fever and a rash to the pediatric clinic. The health-care provider suggests the child may have one of several conditions that present with similar symptoms, but wants to rule out Kawasaki disease. Which tests does the nurse expect to be performed? Select all that apply. 1. Chest x-ray 2. White blood cell count 3. Allergy testing 4. Baseline echocardiograms 5. MRI of the chest

ANS 2,4 1. This is incorrect. A chest x-ray will not reveal Kawasaki's, as it is a blood infection. 2 This is correct. The WBC will reveal lymphocytosis and thrombosis, which are present with Kawasaki disease. 3. This is incorrect. Kawasaki is not an allergy reaction, and allergy testing is inappropriate. 4. This is correct. A life-threatening complication of Kawasaki disease is the development of coronary artery aneurysms. Baseline echocardiogram 6 to 8 weeks after the onset of symptoms is used to rule out this complication. 5. This is incorrect. Magnetic resonance imaging will not reveal Kawasaki disease.

The nurse is reviewing medications for the treatment of a heart rhythm disorder in a patient who is 8 years of age. The parent of the patient states that the physician recently prescribed medication to treat the patient's attention deficit-hyperactivity disorder. Using knowledge of recent professional recommendations, which statement by the nurse is correct? 1. "We need to remind the physician there is a heart condition." 2. "Do not start the medication until I can check for safety warnings." 3. "Children with heart disorders have a higher incidence of ADHD." 4. "Giving the medication can cause death if there is a cardiac issue."

ANS 3 1 This is incorrect. A black box warning about giving medications for ADHD to children with heart problems was issued in 2006. The AAP and the AHA state that medications used to treat ADHD have not been shown in most cases to cause heart disease or result in sudden cardiac death (CDC, 2016; Magellan Health, 2016a). There is no reason to contact the prescribing physician. 2 This is incorrect. If the nurse is knowledgeable about current medication recommendations, the nurse does not tell the parent to hold the medication until the safety can be checked by the nurse. 3 This is correct. Joint statements by the AAP and the AHA show that children with heart conditions have a higher incidence of ADHD, but that medications used to treat ADHD have not been shown in most cases to cause heart disease or result in sudden cardiac death. 4 This is incorrect. It is inaccurate and inappropriate for the nurse to tell the parent that the prescribed medication can cause death in children with cardiac issues.

A 12-year-old patient is experiencing pain in the abdomen after receiving chemotherapy. When developing a care plan, for which interventions will the nurse plan for pain management? 1. Application of heating pads to the abdomen 2. Application of cold packs to the chest area 3. Alternative therapy such as aromatherapy 4. Medication for alleviation of abdominal pain

ANS 3 1 This is incorrect. Applying heating pads will increase circulation to the area and increase pain. 2 This is incorrect. Cold packs to the abdomen may resolve abdominal issues but will not be effective if applied to the chest. 3 This is correct. Aromatherapy is the therapeutic use of essential oils from plants (flowers, herbs, or trees) for the improvement of physical, emotional, and spiritual well-being. 4 This is incorrect. Antacids neutralize acids, which are not the direct cause of the patient's abdominal pain.

An 8-year-old child arrives at the emergency department with abdominal pain and fever. The child has a medical history of leukemia in remission. The nurse receives medical orders for the child. Which order does the nurse recognize as the priority? 1. Draw blood work for a CBC and blood culture. 2. Complete a thorough physical assessment. 3. Set up neutropenic precautions for the child. 4. Prepare the child to be transported to radiology.

ANS 3 1 This is incorrect. The nurse will avoid invasive procedures and use aseptic technique to draw from central lines if present. Laboratory outcomes are important, but this action is not the priority. 2 This is incorrect. The nurse will complete a thorough physical assessment. Vital signs, duration of current symptoms, and systems evaluations are of special interest. This information is important and will guide the physician for additional orders; however, this action is not the priority. 3 This is correct. Patients with a history of leukemia need to be triaged quickly and placed in a treatment room as soon as possible. Neutropenic precautions (protective isolation) need to be initiated. The cancer cells, along with the cancer treatments, place a burden on the child's immune system. The disease pathophysiology makes observation and prevention of infection a priority. 4 This is incorrect. Radiological studies may be in order after the initial assessments and antibiotics have begun. The administration of antibiotics within 60 minutes of assessment for the presence of fever is of greater importance.

The parents of a 4-year-old toddler bring the child to the pediatrician because a lump is found in the toddler's waist area. Diagnostic testing verifies the toddler has a Wilms' tumor on the right kidney. The toddler is to be sent home until scheduled surgery. Which teaching is essential for the parents regarding preoperative care? 1. Promote hydration by increasing fluid intake. 2. Maintain a side position with pillows for sleeping. 3. Avoid palpating the tumor and pushing or lifting in the area. 4. Provide information about postoperative care.

ANS 3 1 This is incorrect. The parents need to provide the toddler with a balanced and healthy diet. There is no reason to expect dehydration or to increase fluids. 2 This is incorrect. There is no identifiable benefit to keeping the toddler in a side-lying position with pillow placement during sleep; keeping a 4-year-old toddler in one position may be challenging. 3 This is correct. Do not palpate the tumor, because this can cause the proliferation or spread of cancer cells. Avoid pushing or lifting in tumor area when handling and bathing the child. This is essential teaching before surgery. 4 This is incorrect. The nurse will provide information about the toddler's postoperative care; however, this is not the essential teaching before surgery.

A 9-year-old child is being treated for a brain tumor. The patient asks the nurse why there is pain in his head. Which reply by the nurse includes pathophysiology and age-appropriate communication? 1. Explain there is a blockage of blood flow to the brain. 2. State the cancer puts pressure on the neck and causes nerve pain. 3. Explain the growing brain tumor presses on some nerves in the head. 4. State the medications being given are causing the headache.

ANS 3 1 This is incorrect. Vaso-occlusive crisis (obstruction of blood flow causing tissue hypoxia and necrosis) is a painful episode with hand-foot syndrome (dactylitis), causing symmetrical infarct in the bones of the hands and feet along with very painful swelling of soft tissue in sickle cell crisis. 2 This is incorrect. A hematoma will cause pain, but there is no information in the scenario confirming the presence of a hematoma. The patient is experiencing pain in the head, not the neck. 3 This is correct. The patient is experiencing pain from a neoplasm in the brain directly affecting nerve receptors. The nurse's answer is pathologically correct and age appropriate. 4 This is incorrect. The medications are not a likely the cause of the patient's headache. Medication therapy is aimed at decreasing the size of the tumor and pain.

A 5-month-old infant is brought to the pediatrician's office, and the nurse is collecting information from the mother. The mother reports the infant is irritable, eats poorly, and is less active. The nurse identifies tachycardia and a systolic murmur. Which comment by the mother helps the nurse identify the infant's condition? 1. "I am pleased that he sleeps well at night." 2. "Up until recently he was a happy baby." 3. "Formula made him gassy and constipated." 4. "He has not been what I call really sick."

ANS 3 3 This is correct. The infant is exhibiting manifestations related to iron deficiency anemia. The mother's comment is expressing a negative reaction to formula at 4 months of age. The nurse now needs to identify what the infant is being fed; cow's milk is an option but should not be introduced until 12 months old. This change can cause iron deficiency anemia. 1 This is incorrect. The nurse does not identify any cause of the infant's condition from a comment about the infant sleeping well at night. 2 This is incorrect. The mother is confirming a change in the infant's behavior from a happy infant to one that is irritable. 4 This is incorrect. The mother's comment about the infant not being identifiably sick does not help the nurse identify the infant's condition.

The nurse is providing care for a 12-year-old patient who is hospitalized with generalized weakness and muscle wasting, which began in the hips, pelvic area, thighs, and shoulders. The physician suspects Duchenne muscular dystrophy. Which action by the physician does the nurse expect? 1. Prescribe physical therapy to improve muscle strength. 2. Suggest homeschooling until the acute stage ends. 3. Perform an echocardiogram to evaluate cardiac functioning. 4. Perform muscle biopsies to identify the stage of the disease.

ANS 3 3 This is correct. The physician is likely to order cardiac testing, especially to rule out cardiomyopathy, a condition frequently associated with Duchenne muscular dystrophy. 1 This is incorrect. Duchenne muscular dystrophy is a progressive genetic disease. Physical therapy is not prescribed to improve muscle strength; it is used to maintain body function for as long as possible. 2 This is incorrect. Duchenne muscular dystrophy is a progressive genetic disease. The physician may recommend homeschooling when the patient is no longer able to attend classes. There is no acute stage to this disease that will improve. 4 This is incorrect. The physician is more likely to prescribe neuromuscular tests to determine the extent of the disease.

The nurse is teaching a pediatric electrocardiogram (EKG) class to nurses in a pediatric cardiac unit. Which anatomical structure does the nurse use to describe the initiation of cardiac electrical conduction? 1. The Purkinje fibers in the ventricles 2. The bundle branch in the left atrium 3. The sinoatrial node in the right atrium 4. The bundle of His in the ventricle walls

ANS 3 4 This is incorrect. From the SA, the electrical impulse is conducted to the fibers called the bundle of His located in the walls of the ventricles. The process of electrical conduction does not start here. 1 This is incorrect. The Purkinje fibers are located in the ventricles; however, the process of electrical conduction does not start here. The fibers initiate contractions of the ventricles. 2 This is incorrect. The bundle branches are divided into either left or right bundles; however, the process of electrical conduction does not start here. 3 This is correct. When teaching the electrical conduction of the heart, the nurse starts with the sinoatrial (SA) node in the right ventricle, which is known as the pacemaker of the heart.

The nurse in a pediatric clinic is assessing an infant who is 3 months old during a well-baby visit. Which assessment finding will be of greatest concern to the nurse? 1. The infant mouth breathes when crying. 2. The infant's eardrums are pink in color. 3. The infant exhibits 15-second periods of apnea. 4. The infant's respiratory rate is fast and irregular.

ANS 3 This is correct. Periods of apnea (the absence of respiration) that last up to 15 seconds are typical of newborns; however, at 3 months of age this patient is considered an infant. The finding is not expected and causes the nurse concern. Newborns are obligatory nose breathers until 4 weeks of age. It is not unexpected or of concern for a 3-month-old infant to take mouth breaths when crying. The nurse will perform further assessment to determine if the infant is exhibiting signs of a mild ear infection. This finding is of concern, but periods of apnea are of greatest concern. A child's metabolic rate is higher than that of an adult's, creating a higher oxygen demand. An infant's respiratory rate is expected to be faster, with an irregular pattern.

The nurse on a pediatric unit is providing care for a 5-year-old child diagnosed with congestive heart failure. The physician prescribes digoxin therapy. Which medication-focused interventions does the nurse include when creating a plan of care for the patient? Select all that apply. 1. Hold medication if an antibiotic is prescribed. 2. Evaluate parent's ability to obtain radial pulse. 3. Administer medication at the same time every day. 4. Administer 1 hour before or 2 hours after meals. 5. Replace medication if a dose is vomited within 1 hour.

ANS 3,4 1. This is incorrect. The child on digoxin therapy needs to be closely monitored for digoxin toxicity with antibiotic therapy, which is caused by changes in intestinal flora. The nurse will not include any interventions for holding digoxin on the plan of care. 2. This is incorrect. Educating parents on how to assess an infant's apical pulse rate and to notify the health-care practitioner if the heart rate is out of the range set by the health-care provider is included in the plan of care. However, the nurse will not include teaching parents how to take radial pulses. 3. This is correct. The nurse will include interventions on the plan of care regarding medication administration. The medication is administered at the same time every day and at the correct frequency. 4. This is correct. In order to promote absorption of digoxin, the nurse will administer the medication 1 hour before or 2 hours after meals. This intervention can be included on the plan of care. 5. This is incorrect. The nurse will not include an intervention on the plan of care addressing when to replace vomited or missed doses; the medication is not to be replaced. There is no way to determine how medication was already absorbed.

The nurse in a pediatric clinic is assessing an infant who is 6 months of age. The infant is pale with poor muscle tone. Auscultation reveals tachycardia; in addition, the infant is at the 35th percentile for both height and weight. The physician orders laboratory testing. Which laboratory value does the nurse expect? 1. Erythropoietin level of 1.9 mIU 2. Serum iron 24 mcg/dL 3. Normal RBC index in CBC 4. RBCs small in size and pale

ANS 4 4. This is correct. When an infant is diagnosed with iron deficiency anemia, the RBCs will appear small and pale in color. 1 This is incorrect. Erythropoietin level of 1.9 mIU is within normal limits. 2 This is incorrect. Serum iron 24 mcg/dL is within normal limits. 3 This is incorrect. CBC indicating a normal RBC index is not a sign of iron deficiency anemia.

A 16-year-old adolescent is diagnosed with osteosarcoma and has evidence of metastasis. The nurse calculates the absolute neutrophil count (ANC) daily. The patient asks the nurse to explain the ANC. Which information does the nurse provide? 1. The ANC is a measure of how well treatment is being tolerated by the body. 2. The ANC is total percentage of eosinophils plus the total percentage of basophils divided by the RBC. 3. The ANC provides a daily count of the number of antibodies in the circulating blood. 4. ANC measures of three types of white blood cells. A lower ANC indicates vulnerability to infection.

ANS 4 1 This is incorrect. The ANC does not specifically measure how the body is tolerating treatment. When the ANC is less than 500, the client is at high risk for infection. 2 This is incorrect. The ANC is a measure of bands, segmented cells, and white blood cells, which does not include eosinophils. 3 This is incorrect. Antibodies are not factored into the ANC. 4 This is correct. Calculating the ANC requires three numbers from the CBC and differential. The following formula calculates the ANC: (% bands + % segmented cells) x number of WBC= ANC. The result indicates the patient's vulnerability to infection and directs the level of isolation required.

The nurse is providing information to the parents of a toddler who is scheduled for surgery for the replacement of the pulmonic valve. The parents have many questions about the function of the valve. Which information from the nurse is correct? 1. The valve must work correctly to get oxygen from the lungs to the body. 2. If the valve does not work correctly, blood is kept from entering the heart. 3. When the valve is defective, the blood leaving the heart is decreased. 4. A defect in the valve causes less blood to get to the lungs for oxygenation.

ANS 4 4 This is correct. When there is a defect in the right pulmonic valve, the blood has difficulty leaving the right ventricle and getting to the lungs for reoxygenation. This explanation by the nurse correctly describes the function of the valve and the purpose of the surgery. 1 This is incorrect. The aortic valve must work correctly for oxygenated blood to be carried from the left side of the heart to the rest of the body. 2 This is incorrect. Blood must enter the right side of the heart from the body in order to be reoxygenated. The unoxygenated blood enters the right atrium through the tricuspid valve into the right ventricle. 3 This is incorrect. A defect in the aortic valve would cause problems with oxygenated blood leaving the left ventricle.

The pediatric nurse is preparing a teaching plan for new mothers with small infants. Which is a key point for the nurse to include in the teaching plan? 1. Infants are obligatory mouth breathers for the first month. 2. All sinuses are formed and aerating within 2 months of birth. 3. Infants are abdominal breathers until they are 12 months old. 4. Infant airways get blocked more easily than those in older children.

ANS 4 Newborn airways are approximately 4 mm in diameter compared with 20 mm for the average adult's airway. Inflammation 1 mm in circumference would decrease a child's airway diameter 50% but only 20% for an adult. Newborns are obligatory nose breathers until 4 weeks of age, which is the reason newborns with upper respiratory infections have difficulty with feeding from the breast or bottle. Only ethmoid and maxillary sinuses are present at birth and are not aerated until 4 months. Sphenoid and frontal sinuses develop later in childhood and continue to mature into adolescence. The infant's intercostal muscles are not fully developed, and pronounced abdominal wall movement with respiration is normal until 6 years of age.

The nurse is providing care for a school-age patient who received a head injury while playing sports. Which initial assessment finding causes the nurse greatest concern? 1. Confusion and disorientation 2. Headache with periods of nausea 3. Immediate loss of consciousness 4. Changes in breathing and heart rates

ANS 4 Normal breathing is involuntary; the central nervous system controls rate and volume of respiration. Adjustments are made in respiration rate, heart rate, and cardiac output to maintain adequate gas exchange. The finding will alert the nurse to either hypoxia in the brain or injury to the part of the brain that controls respiratory function. The scenario does not specify an increase or decrease in the rates. Confusion and disorientation are common manifestations of a head injury. This finding does not cause the greatest concern for the nurse. Headache and periods of nausea are not uncommon after a head injury. While initially this finding does not cause the nurse greatest concern, frequent reassessment is necessary to identify manifestations of increasing intracranial pressure. Immediate loss of consciousness at the time of a head injury is not uncommon. However, the nurse will continue to monitor for manifestations of increasing intracranial pressure.

The nurse in a pediatric clinic is performing assessments on multiple infants. Which infant does the nurse recognize as being at greatest risk for a respiratory disorder? 1. The infant born at 36 weeks who exhibited respiratory problems at birth 2. The infant who was born at term and recently adopted from another country 3. The infant who sleeps all night, exhibits eczema, and has a family history of asthma 4. The infant with recurrent sore throats and both pets and smokers in the house

ANS 4 The infant with recurrent sore throats and exposure to environmental irritants such as pets and smokers in the household is at greatest risk for developing a respiratory disorder. This patient is recognized by the nurse as having three risk factors Without additional manifestations or information, the premature infant who exhibited respiratory problems at birth is not the patient with the greatest risk for respiratory disorders. The only risk factor is gestational age. The infant who was recently adopted from another country is at risk for respiratory problems because of unknown environmental, physiologic, and/or genetic influences. The only risk factor is being born in another country. The infant with eczema and a family history of asthma has two risks for the development of a respiratory disorder. Sleeping all night is not a risk factor.

The nurse in an acute care pediatric facility is preparing to assume care of multiple patients at the change of shift. Which patient will the nurse plan to assess first? 1. The toddler who exhibits clubbing of the fingertips 2. The preschooler with pneumonia who has poor skin turgor 3. The infant who can sleep only with the head of the bed elevated 4. The infant who prefers a tripod position instead of lying down

ANS 4 When an infant prefers to sit in a tripod position, exhibits a jaw thrust, or is insistent on sitting upright, the indications are relevant to air hunger and oxygen deficiency. This is the patient the nurse will assess first. A toddler may exhibit clubbing of the fingertips if diagnosed with chronic hypoxia from a chronic respiratory disorder. The manifestation occurs over a period of time and does not alone indicate a need for being assessed first. The preschooler with pneumonia and poor skin turgor may be dehydrated from mouth breathing, tachypnea, fever, and/or anorexia. Without additional symptoms, this is not the patient the nurse will assess first. The infant who can sleep only with the head of the bed elevated is not presently in distress. This patient does not need to be assessed first.

The nurse is interviewing an adolescent patient 17 years of age who was diagnosed with cystic fibrosis (CF) as an infant. The patient shares feelings of frustration about needing to always live with parents. Which information provided by the nurse is likely to be most important to the patient? 1. How chest physiotherapy (CPT) can be performed independently 2. The availability of home meal delivery to those needing a therapeutic diet 3. Organizations that will provide transportation for persons with chronic illness 4. A list of social organizations available for young persons who have special needs

ANS: 1 Chest physiotherapy is necessary three to four times daily for the patient with CF. The nurse can inform the patient about equipment and techniques. Some suggestions will include handheld massager and an oscillating vest. The nurse will use a multidisciplinary approach to promote independence for the patient. The patient will benefit from teaching about how to independently meet dietary needs and does not necessarily need home-delivered meals. Persons with CF are not excluded from driving. The patient is likely to have a social circle of friends. Referring the patient to organizations that are designed for persons with special needs is not necessary/appropriate.

43. A child with hyperlipidemia should consume no more than _________ mg of cholesterol a day. 1. 200 2. 500 3. 50 4. 100

ANS: 1 Feedback 1. A child should not consume more than 200 mg of cholesterol a day. 2. Too much cholesterol for a healthy diet 3. Too little cholesterol for a healthy diet 4. Too little cholesterol for a healthy diet

31. A school nurse has been made aware that an eighth grader has latent tuberculosis (TB). Education for the teaching staff should include: 1. A document with the signs and symptoms of illness for a person with TB. 2. Do not allow the child into the classroom when he coughs. Send him to the nurses office to prevent the spread of the illness. 3. Provide universal precautions with the child. 4. The child does not need any interventions at this time because the TB is dormant.

ANS: 1 Feedback 1. A signs and symptoms document will help increase the awareness of the disease and can also help identify those who are infected early. 2. The spread of the disease cannot occur just because of coughing. 3. Universal precautions should be used with every student, not just the ill children. 4. Interventions will help prevent the illness from spreading.

37. An 8-year-old boy with a long history with cystic fibrosis has been admitted for malnutrition. The doctor has ordered labs for the child. The nurse clarifies which doctors order before proceeding? 1. Obtain a stool sample for Clostridium difficile 2. Metabolic panel for hydration status 3. Serum albumin level to measure the nutritional status 4. Provide chest physiotherapy before bedtime

ANS: 1 Feedback 1. A stool sample should be used for the absence of trypsin. 2. Malnutrition may be caused by metabolic issues. 3. Serum albumin levels will help indicate nutritional status and are appropriate for this patient. 4. Chest physiotherapy is needed at bedtime to rid as many secretions as possible prior to lower activity levels.

15. A nurse is assessing a child with a known VSD. The nurse anticipates auscultating: 1. A systolic thrill in the lower left sternal border. 2. Wet lung sounds bilaterally. 3. A diastolic thrill in the upper left sternal border. 4. A diastolic wetness in the right sternal border.

ANS: 1 Feedback 1. A thrill sound in the left sternal border will be heard because of where the valve is located. 2. The lung sounds should be clear. 3. A systolic thrill and lower left sternal border thrill will be noted. 4. A thrill sound in the left sternal border with diastolic sounds will be heard because of where the valve is located.

8. A 5-year-old child has been admitted for complications related to asthma. When the nurse auscultates the childs lungs, she would anticipate hearing: 1. Wheezes because the bronchioles have been restricted. 2. Rhonchi because of thick secretions from the flare-up. 3. Crackles because there is fluid in the alveoli. 4. All of the above may be heard.

ANS: 1 Feedback 1. Asthma constricts the airway and alveoli in children, causing wheezing to be heard when in auscultation. 2. Rhonchi usually will clear with a cough. A child with an asthma exacerbation will not stop the sound after coughing. 3. Asthma causes the narrowing of airways. Crackles occur only when fluid is present. 4. The airway and alveoli constriction causes wheezing.

3. Acquired thrombocytopenia involves antibodies against: 1. Platelets. 2. Basophils. 3. Neutrophils. 4. Eosinophils.

ANS: 1 Feedback 1. Decreased platelet production, increased platelet destruction, or splenic sequestration is common with thrombocytopenia. 2. The basophils are intact with thrombocytopenia. 3. The neutrophils function with thrombocytopenia. 4. The eosinophils are intact with thrombocytopenia.

20. ECHMO is commonly used as a treatment for a baby with which defect/syndrome? 1. Eisenmengers syndrome 2. Coarctation of the aorta 3. ASD 4. Tetralogy of Fallot

ANS: 1 Feedback 1. ECHMO acts as a bypass for the heart and lungs to obtain enough oxygen for the body. 2. ECHMO acts as a bypass for the heart and lungs to obtain enough oxygen for the body. In coarctation, the heart and lungs are able to perfuse. 3. ECHMO acts as a bypass for the heart and lungs to obtain enough oxygen for the body. 4. ECHMO acts as a bypass for the heart and lungs to obtain enough oxygen for the body.

11. Evidence of iron deficiency anemia in infants 9 to 12 months of age is most likely to be caused by: 1. Excessive milk intake. 2. Addition of solid foods. 3. Prematurity of the infant. 4. Rh and ABO incompatibility.

ANS: 1 Feedback 1. Excessive milk intake will decrease appetite and result in fewer intakes of foods containing iron. 2. Solid foods will contain iron, thus not causing a deficiency. 3. A premature infant may be anemic for other reasons besides iron deficiency. 4. The incompatibilities would have been noticed earlier in the infants life. This does not cause iron deficiency.

7. When performing an assessment on an 8-year-old boy who is hospitalized for pneumonia, the nurse would anticipate what type of lung sounds? 1. Crackles 2. Stridor 3. Normal 4. Wheezes

ANS: 1 Feedback 1. Fluid is built up in the lungs because of the infection, causing crackles to be heard. 2. Stridor is common in children with larynx issues, not pneumonia. 3. When fluid builds up in the lungs, it will cause the lungs sounds to be abnormal with a diagnosis of pneumonia. 4. A child will have wheezes if the airway is constricted, not full of fluid.

7. What factor contributes to a vaso-occlusive crisis in a child with Sickle Cell anemia? 1. Dehydration 2. Alkalosis 3. Infection 4. Stress

ANS: 1 Feedback 1. Fluid replacement with IV fluids increases the flow of blood, which decreases tissue hypoxia and the potential for dactylitis. 2. The child may become alkalotic, but this s not the contributing factor to the vaso-occlusive crisis. 3. Infections can cause dehydration, but this is not the leading cause of the vaso-occlusive crisis. 4. Stress is a manageable condition and does not lead to a vas-occlusive crisis.

26. A school-age child is admitted with vaso-occlusive Sickle Cell crisis. The childs care should include which of the following? 1. Hydration and pain management 2. Oxygenation and factor VIII replacement 3. Electrolyte replacement and the administration of heparin 4. Correction of alkalosis and reduction of energy expenditure

ANS: 1 Feedback 1. Hydration and pain medication are needed in order for the child to heal. 2. Oxygenation is needed, but factor VIII is not used in Sickle Cell treatment. 3. Hydration is important, but the use of heparin will not decrease the pain. 4. The child needs a balanced electrolyte state and can be as active as possible without pain.

50. A 14-year-old girl is admitted to the pediatric emergency room with symptoms of fever, rash, syncope, nausea, and vomiting. The most important information that the admitting nurse should obtain is: 1. Are you currently menstruating? 2. When did you last eat? 3. Have you been in contact with anyone who has had the flu? 4. Are you sexually active?

ANS: 1 Feedback 1. If the girl is currently menstruating, the use of tampons may indicate toxic shock syndrome. 2. This is not the most important factor to obtain given the symptoms the patient has exhibited. 3. While knowledge of a childs exposure to viruses is important, it is not the most important factor to obtain. 4. This is not the most important factor to obtain given the symptoms the patient has exhibited

29. A common cause of viral pneumonia in children is: 1. The influenza virus. 2. Streptococcus. 3. Fungus. 4. Beta-hemolytic streptococcus pneumoni.

ANS: 1 Feedback 1. Influenza is a common cause for viral pneumonia in children as a secondary infection. 2. Streptococcus is a bacterium, not a virus. 3. Fungus is not a virus. 4. Beta-hemolytic strep is bacterial, not viral.

28. What is the bacteria that most commonly results in respiratory tract infections in patients receiving chemotherapy? 1. Klebsiella 2. E. coli 3. Epstein Barre Virus 4. Proteus

ANS: 1 Feedback 1. Klebsiella is the most common bacteria found in respiratory infections in patients receiving chemotherapy. 2. E. coli is commonly found in GI infections, not respiratory infections. 3. The Epstein Barre Virus is found in neurological tract infections, not respiratory infections. 4. Proteus are not commonly found in patients with respiratory infections that are undergoing chemotherapy.

35. Clubbing of the nailbeds in the fingers would be a clinical finding on which patient? 1. A child with cystic fibrosis 2. A child with croup 3. A child with respiratory distress syndrome 4. A child with RSV

ANS: 1 Feedback 1. Long-term hypoxia causes clubbing of the nailbeds because of the lack of oxygen. 2. Croup is a short-term respiratory issue, which does not causing clubbing. 3. Respiratory distress syndrome is short lived and does not cause clubbing. 4. RSV is short lived and does not cause clubbing.

32. A neonate has been diagnosed with respiratory distress syndrome. The nurse notes the neonate is retracting and is hypoxic. The best intervention at this time would be: 1. Providing oxygen support via a mask. 2. Providing oxygen support via nasal cannula. 3. Attempt to reposition the neonate. 4. Check the temperature of the neonate so that the child does not experience cold stress.

ANS: 1 Feedback 1. Oxygen delivered by mask is the highest percentage of oxygen to be delivered other than intubation. 2. The neonate does not receive as high of a rate of oxygen saturation with a nasal cannula. 3. Repositioning may open the airway more, but the retracting occurs because of deterioration, thus requiring oxygen support. 4. Cold stress can cause respiratory issues, but is short term once the neonate is warm.

An infant with a patent ductus arteriosus will exhibit which type of heart murmur? 1. Washing machine murmur 2. Gallop style murmur 3. Clicking murmur 4. Harsh, loud murmur

ANS: 1 Feedback 1. PDA murmurs sound like a washing machine due to pulmonary congestion. 2. This is not the type of murmur heard in a PDA. 3. This is not the type of murmur heard in a PDA. 4. This is not the type of murmur heard in a PDA.

60. A newborn, premature twin exhibits respiratory distress with retractions, nasal flaring, cyanosis, grunting, and fine, scattered rales. What nursing interventions would you expect the physician to order? 1. Place an NG tube for feeds, monitor respiratory status on ventilator, record I& Os, start an IV, and send electrolyte panel to the laboratory and monitor temperatures 2. Cardio- respiratory monitoring, frequent suctioning on ventilator, and monitoring blood glucose level hourly 3. Placing infant in semi-fowlers position on affected side with head of the bed elevated, oxygen via nasal cannula, keeping NPO, and preparing parents for surgery 4. Giving surfactant intravenously within the first 12 hours of life and repeating every 12 hours for three days.

ANS: 1 Feedback 1. Place an NG tube for feeds, monitor respiratory status on ventilator, record I& Os, start an IV, and send electrolyte panel to the lab and monitor temperatures 2. A ventilator is not needed at this time. Blood glucose should be monitored because it can cause an increase in respiratory distress. 3. Surgery is not indicated at this time. 4. The statement does not indicate the level of prematurity for the infant. Surfactant is not needed at this particular time.

9. What is the most common opportunistic infection in children with the Human Immunodeficiency Virus? 1. Pneumocystic pneumonia 2. CMV 3. Meningitis 4. Encephalitis

ANS: 1 Feedback 1. Pneumocystic pneumonia is most common, and children are treated prophylactically. 2. CMV is not considered an opportunistic infection. 3. Meningitis is not common in children with HIV. 4. Encephalitis can occur, but it is not a cause of infection.

3. When a child exhibits difficulty breathing, the best positioning would be: 1. Having the head of the bed at 45 degrees. 2. Placing the child in a 90 degree angle on the parents lap. 3. Placing the child in a side lying position. 4. Having the child sit in a chair.

ANS: 1 Feedback 1. Positioning the head of the bed slightly elevated will take weight off of the diaphragm and allow for full chest expansion. 2. Placing the child at a 90 degree angle will put too much pressure on the diaphragm, thus causing the shortness of breath to continue. 3. A side lying position does not help to support the diaphragm or aid in relieving the shortness of breath. 4. Sitting in a chair will place more stress on the accessory muscles, thus the child will continue to have shortness of breath.

32. A baby is born with a known hypoplastic left heart. At the delivery, the nurse should anticipate which of the following in the initial assessment after birth if the PDA closes? 1. Tachypnea and an ashy color 2. Tachycardia, pale in color, and apnea 3. Bradycardia and apnea 4. Ruddy color and tachycardia

ANS: 1 Feedback 1. Rapid breathing and an ashen look will be present because the baby is not able to push the blood to the lungs in order to get oxygen. 2. Tachycardia and apnea are not present. 3. Bradycardia and apnea are not present. 4. The baby would be ashen and have rapid tachypnea.

56. A child with a known neuroblastoma begins vomiting uncontrollably. The nurse assesses the childs _____ for signs of increased intracranial pressure. 1. Pupils and hand grasps 2. Intake and output 3. Respiratory rate 4. Verbal responses

ANS: 1 Feedback 1. Sluggish and dilated pupils along with weakened hand grasps are an indication of increased ICP. 2. Intake and output are not a concern at this time. 3. The childs respiratory rate should be assessed, but it is not the priority at this time. 4. Verbal responses can be assessed, but it is not the priority at this time.

16. An 18 month old with known Tetralogy of Fallot is seen squatting after running in the hospital playroom. The nurse knows the child is: 1. Having a rapid drop in the amount of oxygen in the blood and is short of breath. 2. Having a bowel movement. 3. About to faint because of the lack of oxygen in his blood. 4. Mimicking others in the playroom.

ANS: 1 Feedback 1. Squatting allows the child to take breaths and gain oxygen. 2. The child is attempting to inhale oxygen quickly. 3. The child is maintaining a position to gain oxygen in order to prevent fainting. 4. The child is squatting in order to take deep breaths and gain oxygen after the playing.

63. Which of the following symptoms would an 18-month-old child exhibit when experiencing left-sided heart failure? 1. Tachypnea 2. Tachycardia 3. Syncope 4. Nausea and vomiting

ANS: 1 Feedback 1. Tachypnea is caused by the pooling of secretions and increased incidence of congestive heart disease. 2. Tachycardia is a late sign of left-sided heart failure. 3. Syncope is not a sign of left-sided heart failure. 4. Nausea and vomiting are not symptoms of left-sided heart failure.

10. Which of the following diagnostic tests confirms Hodgkins disease? 1. Reed-Sternberg cells in the lymph nodes 2. Lymphocytes in the bone marrow 3. Neutrophils in the blood 4. Bacteria in the urine

ANS: 1 Feedback 1. The Reed-Sternberg cells are a diagnostic of Hodgkins disease. 2. Lymphocytes are naturally found in the bone marrow. 3. Neutrophils are naturally found in the blood to help fight infections. 4. Bacteria in the urine is not a confirmation of Hodgkins disease.

18. Which of the following is a reason to do a lumbar puncture on a child with a diagnosis of leukemia? 1. To assess the central nervous system for infiltration 2. To determine increased intracranial pressure 3. To stage the leukemia 4. To rule out meningitis

ANS: 1 Feedback 1. The abnormal cells of leukemia are definitive for the disease. 2. A lumbar puncture can increase ICP, not diagnose it. 3. Staging is done with blood cells, not cerebral spinal fluid. 4. If a child has a known diagnosis of leukemia, the lumbar puncture does not need to be done to identify meningitis unless other blood tests indicate infection.

55. A father brings his 3-year-old daughter to the cancer treatment center for a follow-up appointment. The previous rounds of chemotherapy were not successful in treating his daughters cancer. The plan is to provide an allogeneic transplantation. The father verbalizes an understanding of the process when he states: 1. An allogeneic transplant puts my daughter as the lowest risk for rejection. 2. I will need to get high doses of chemotherapy for the bone marrow to be appropriate for my daughters body. 3. I will need to be in isolation after the donation procedure. 4. My daughter will need a transplant from her mother and myself to be successful in curing the disease.

ANS: 1 Feedback 1. The allogeneic transplant has the lowest risk of rejection and would be appropriate for the child. 2. The receiver needs the high doses of chemotherapy, not the donor. 3. The daughter will need to be in isolation after the donation to help decrease the risk for infection because the chemotherapy increases her susceptibility to infection. 4. The transplant can come from the mother or the father. The important factor is having an allogeneic match.

33. A mother has called the triage nurse of the pediatric clinic to ask what she should do for her sons frequent epistaxis episodes. The treatment should include: 1. Having the child sit up with his head tilted forward so that blood does not go down the throat. 2. Encouraging the child to breathe through his nose. 3. Having the child attempt to keep his nose clean. 4. Applying heat to the nares.

ANS: 1 Feedback 1. The blood could be aspirated, so sitting forward will help decrease the occurrence. 2. Breathing through the nose can cause increased pressure and not stop the bleeding. 3. Cleansing and digging in the nose can cause clots to break and cause continued bleeding. 4. Ice should be applied to vasoconstrict the blood vessels to stop the bleeding.

14. The most frequent presenting signs of leukemia are related to bone marrow infiltration. The main symptoms are: 1. Anemia, infection, and bleeding. 2. Thrombocytopenia, headache, and abdominal pain. 3. Respiratory distress and pain. 4. Confusion and decreased peripheral vascular resistance.

ANS: 1 Feedback 1. The normal bone marrow is replaced with abnormal cells, leading to decreased red blood cells and anemia. The risk of infection relates to the lack of white blood cells and the reduction in platelets of thrombocytopenia. 2. The normal bone marrow is replaced with abnormal cells, leading to decreased red blood cells, which causes ischemia and pain. The risk of infection relates to the lack of white blood cells and the reduction in platelets of thrombocytopenia. Headaches are not a common sign. 3. The normal bone marrow is replaced with abnormal cells, leading to decreased red blood cells, which causes pain. The patient rarely exhibits signs of respiratory distress. 4. Neurological issues and peripheral vascular resistance are rare.

19. A child with cancer has a platelet count of 4,500/mm3. Which of the following should the nurse prepare to administer? 1. Platelets 2. Packed red blood cells 3. Neupogen 4. Erythropoeitin.

ANS: 1 Feedback 1. The normal level of platelets in greater than 5000/mm3. The child is at a high risk for bleeding, thus platelets are needed to help build clotting factors. 2. PRBCs will not increase clotting time. It will help increase the red blood cell carrying capacity of oxygen. 3. Neupogen stimulates the production of red blood cells. 4. Erythropoietin stimulates bone marrow to produce red blood cells.

A child has the following ABG results: pH: 7.38 pCO2: 52.6 HCO3: 32.5 The nurse interprets these results as: 1. Compensated Respiratory Acidosis. 2. Uncompensated Respiratory Alkalosis. 3. Compensated Respiratory Alkalosis. 4. Uncompensated Respiratory Acidosis.

ANS: 1 Feedback 1. The pH is on the low end, creating a more acidotic state along with the CO2 in an acidotic state, thus indicating the respiratory acidosis. The HCO3 is alkalotic, creating compensation. 2. The pH and the CO2 are acidotic and the HCO3 is alkalotic, creating compensation. 3. The pH and CO2 are in acidotic states, not alkalotic states. 4. Compensation has occurred because of the HCO3 being alkalotic.

3. In fetal development, the _________ is open to allow blood to flow in the heart. 1. Patent ductus arteriosus 2. Pulmonic valve 3. Aortic valve 4. Bicuspid valve

ANS: 1 Feedback 1. The patient ductus arteriosus is the opening in the heart that allows blood to flow in the heart. 2. The pulmonic valve is not the opening. 3. The aortic valve is open before and after birth. 4. The bicuspid valve is open before and after birth.

22. Following a tonsillectomy, a nurse should provide the patient with: 1. Ice chips, no pillow, and no straw for drinking. 2. Ice chips and orange juice. 3. A sippy cup and pudding. 4. A pillow, red Gatorade, and a straw.

ANS: 1 Feedback 1. The patient should lie flat to help clotting occur, ice chips will provide hydration, and no straw should be given because this can cause the clots to break and increase bleeding. 2. Orange juice should not be used because the pulp may lodge into the surgical site. 3. A sippy cup can cause clots to break because of the sucking motion and pudding is too thick to swallow at this point. 4. A patient should lie flat to help with clotting, Gatorade should not be used because you cannot assess for blood because of the color, and a straw will cause the clots to break and increase bleeding.

52. In educating the caregivers about the administration of Digoxin (Lanoxin) to their child, the nurse instructs the caregivers to: 1. Notify the physician of weight gain of two pounds or more per day. 2. Administer the medication at any set time during the day, every day. 3. Administer the medication two hours before meals and one hour after meals. 4. Hold the Digoxin if the heart rate is <60/minute or >120/minute

ANS: 1 Feedback 1. The physician should be notified of weight gain, which can place extra strain on the heart and may indicate congestive heart failure. 2. The Digoxin should be administered in the morning. 3. The medication should be given one hour before meals and two hours after meals. 4. The medication should be held for a heart rate <60/minute and >100/minute.

43. The nurse is assessing a child that was in a motor vehicle accident, which occurred two hours ago. The childs chest is not rising on the right and lacks lung sounds. The X-ray confirmed a hemothorax. The nurse should anticipate the order for: 1. A chest tube and pnuemovac. 2. IV fluids. 3. Placing a nasogastric tube. 4. None of the above would be appropriate for the situation.

ANS: 1 Feedback 1. The pnuemovac will aid in the creation of a sterile container to help decompress the hemothorax. 2. IV fluids may be ordered eventually, but they are not a priority at this time. Airway security is the priority. 3. A nasogastric tube will not influence the hemothorax. 4. The nurse should anticipate the use of the pneumovac to help decompress the hemothorax.

25. Which nursing action is implemented first when a child is admitted to the transplant center for a hematopoietic stem cell transplant? 1. Prepare the child and family for an intensive ablative dose of chemotherapy. 2. Place the child in protective isolation. 3. Maintain a central line catheter. 4. Serve irradiated food and water to the child.

ANS: 1 Feedback 1. The preparation of the child and the caregivers for the weeks of hospitalization and social isolation required is a priority. A discussion with the child and caregivers will include the factors of the critical period of recovery for the client from the removal of cancer cells, transfusing histocompatible stem cells, and monitoring for signs of rejection. 2. Isolation is not the top priority at this time. 3. It is not stated that the child has a central line at this time, thus it is not a priority. 4. The concentration should be on the preparation of what is going to occur and how goals will be met.

15. A toddler with cancer has a central line catheter for chemotherapy. The priority for the nurse is to: 1. Administer nutrition using the central line. 2. Provide privacy during medication administration. 3. Use the central line to administer antibiotics. 4. Limit visitors to family members.

ANS: 1 Feedback 1. The priority is the prevention of infection during the care of the child as well as making sure that the child has adequate procedures to prevent infection when using the central line. 2. Privacy is important for all patients. 3. The central line can be used for nutrition and medication administration. 4. Additional precautions do not need to be taken for visitors at this time.

18. A newborn with a diagnosis of Tetrology of Fallot is demonstrating heart failure. The doctor orders a prostaglandin E1 drip. The nurse knows this is used to: 1. Maintain blood flow to the lungs. 2. Open the patent foramen ovale. 3. Increase blood flow to the extremities. 4. Decrease resistance of blood flow through the heart.

ANS: 1 Feedback 1. The prostaglandin will allow the Patent Ductus Arteriosis to have patency. 2. The patent foramen ovale is already open when a Tetrology of Fallot is present. 3. Blood flow to the heart and lungs rather than the extremities is the priority. 4. Because of the holes in the heart, the resistance is already low.

32. Which of the following should the nurse include when teaching the mother of a 9-month-old infant about administering liquid iron preparations? 1. Adequate dosage will turn the stools a tarry green color. 2. Stop immediately if nausea and vomiting occur. 3. Give the medication with meals. 4. Allow the preparation to mix with saliva and bathe the teeth before swallowing.

ANS: 1 Feedback 1. The side effect of oral iron supplements is tarry green stools. 2. Nausea and vomiting are not common side effects of iron supplementation. 3. The medication should be spaced out from the meals to decrease the chance for nausea and vomiting. 4. Most infants do not like the taste of iron supplements and will spit it out if mixed with the saliva.

47. A father of an 18-month-old boy reports that his son has been constipated, has pain in his abdomen when touched, and seems lethargic since they moved to a new house. The nurse should asses if the environment: 1. Contains lead-based paint. 2. Has pollen in the area surrounding the house. 3. Is near a factory. 4. Is near power lines.

ANS: 1 Feedback 1. The symptoms are common with exposure to lead-based paint, and the nurse should anticipate a blood test. 2. Pollen may cause sneezing as well as watery, itchy eyes, but not the symptoms described. 3. Living near a factory may cause environmental hazards, but the child is showing signs of lead poisoning. 4. The child is showing signs of lead poisoning, which is not found near power lines.

24. A common bacteria that causes scarring on the aortic valve is: 1. Group A streptococcus bacteria. 2. Group B streptococcus bacteria. 3. Staphylococcus aureus. 4. E. coli.

ANS: 1 Feedback 1. This common bacteria causes scarring on the aortic valve. 2. This is not a common bacteria in the heart. 3. Common in endocarditis, but not a common cause of scarring on the aortic valve. 4. Usually attacks the GI tract

48. A 6 year old had a tonsillectomy today. When the nurse goes into the room to give him his antibiotics, she finds him irritable, coughing, nauseated, and swallowing repeatedly. What is the next action the nurse should take? 1. Assess for signs of frank red blood in the mouth and nose and get a complete set of vital signs. 2. Ask the child for a pain score and if he would like a popsicle with his pain medicine. 3. Suction mouth vigorously to avoid aspiration of blood, and then hang antibiotic. 4. Take a complete set of vital signs and divert the childs attention to the cartoon on TV.

ANS: 1 Feedback 1. This intervention assesses for bleeding. 2. An assessment for blood needs to occur because the child continues to swallow. 3. Suctioning can cause clots to loosen and increases bleeding. It should be avoided. 4. Vital signs are needed and a focused assessment needs to be completed in order to identify complications.

5. A nurse is discussing heart disorders that cause the mixing of oxygenated and deoxygenated blood with a new nurse. The nurse should explain that the mixed disorders consist of all of the following except: 1. Tetralogy of Fallot. 2. Hypoplastic left heart. 3. Truncus afteriosus. 4. Transposition of the great vessels.

ANS: 1 Feedback 1. This is an obstructive disorder. 2. A mixed blood heart defect 3. A mixed blood heart defect 4. A mixed blood heart defect

66. A 12 year old is suspected of having rheumatic heart disease. What factors would indicate that this child has this disease? 1. The onset of symptoms occurs around 20 days after streptococcus throat infection or scarlet fever. 2. The child lives in the most common area of the western United States. 3. The disease produces lesions in the mouth and oropharynx. 4. The disease results in damage to the peripheral sensory nerves.

ANS: 1 Feedback 1. This is the normal course of this disease. 2. Most cases occur in the northeastern part of the United States. 3. The disease produces polyarthritis, carditis, subcu nodules, and a low-grade fever. 4. The disease produces polyarthritis, carditis, St. Vitus Dance, and a low-grade fever.

54. An infant born an hour ago exhibits coughing and drooling, cyanosis, abdominal distention, and moderate retractions and grunting. Based on these symptoms, what would be the most likely diagnosis? 1. Tracheoesophageal fistula 2. Laryngomalacia 3. Respiratory distress syndrome 4. Bronchopulmonary dysplasia

ANS: 1 Feedback 1. Tracheoesophageal fistula is the most likely diagnosis. 2. Laryngomalacia would cause more grunting. 3. The child may initially present similar respiratory distress, but the drooling indicates that more is involved. 4. Bronchopulmonary dysplasia occurs after long-term ventilator support, not soon after birth.

48. A common opportunistic infection for a child with immune suppression related to HIV is: 1. Tuberculosis. 2. Syphilis. 3. Meningitis. 4. E. Coli

ANS: 1 Feedback 1. Tuberculosis is one of the most common opportunistic infections in children with HIV. 2. Syphilis is not common in children with HIV. 3. Meningitis is not common in children with HIV. 4. E. Coli may be present in the GI tract, but does not commonly cause further complications for children with HIV.

16. A mother calls the triage nurse because her 8-year-old son is having trouble keeping his balance, but has otherwise appeared healthy for the past few days. The nurse should advise the mother to: 1. Make a doctors appointment because the child could have issues with his inner ear. 2. Take the child immediately to the ER because this is a neurological emergency. 3. Ask the child if he has consumed any drugs or alcohol in the last few days. 4. Call back in a few days with an update.

ANS: 1 Feedback 1. Unknown etiologies of unsteady balance are a sign of inner ear infections. 2. Since the mother feels the child is healthy and does not exhibit any other neurological symptoms, a doctors appointment is advisable. 3. A child would be exhibiting more symptoms than unsteady balance if he was taking a substance. 4. The concern should be addressed and an appointment made to find the cause of the unsteady balance.

56. It is May, and a mother brings in her 3-year-old son, who has had a harsh whooping cough, runny nose, and watery eyes for the past five days. What would be the most appropriate question to ask the mother? 1. Are the childs immunizations up-to-date, including his Tdap vaccine? 2. Did the child receive his Hib vaccine? 3. Have you taken the child outside in the rain? If so, what happened? 4. When was the last time your child was ill?

ANS: 1 Feedback 1. Up-to-date immunizations will include the Tdap vaccine. If the child has had the vaccine the occurrence/severity of the illness is less. 2. Hib does not include the Whooping Cough vaccine. The question would not be appropriate at this time. 3. Weather does not influence the vaccines. 4. Past illnesses is not the focus of the current assessment and is not appropriate at this time.

The nurse in the newborn unit of a pediatric hospital is providing care for a neonate born at 34 weeks' gestation. The nurse is aware that the immediate risk to the neonate is which condition? 1. A lack of a phospholipid in the alveoli 2. Inability to maintain body temperature 3. Delay in closure of cardiac foramen 4. A decrease in renal function

ANS: 1 This is correct. The nurse's immediate concern is related to respiratory function. A premature neonate is likely to have a low level of surfactant, which is a phospholipid in the alveoli that keeps alveoli pliable, preventing them from collapsing completely at the end of each expiration. phospholipid in the alveoli that keeps alveoli pliable, preventing them from collapsing completely at the end of each expiration. Neonates at any level of maturity can have delays in the closure of cardiac foramen. The immediate risk for a premature neonate is the ability to provide adequate oxygenation. A decrease in renal function in a premature neonate can be related to poor oxygenation because of compromised respiratory function.

58. Identify food sources that are recommended for children to receive iron. Select all that apply. 1. Spinach 2. Broccoli 3. Carrots 4. Chicken breasts 5. Chick peas

ANS: 1, 2 Feedback 1. Green spinach is an iron source. 2. Broccoli is an iron source. 3. Carrots have beta-carotene and fiber, not iron. 4. Chicken breasts do not contain iron, but are a good source of protein. 5. Chick peas are not a good source of iron.

68. Identify the common nursing practices for a newborn with a known patent ductus arteriosus diagnosis. Select all that apply. 1. Maintain intake and output 2. Daily weight checks 3. Monitor feeding tolerance 4. Weekly weight checks 5. Monitor output only

ANS: 1, 2, 3 Feedback 1. Fluid balance will indicate if the newborns body is able to excrete fluid. 2. Daily weight checks will indicate the cardiac performance. 3. Feeding tolerance will indicate the level of energy needed to digest food. 4. Weight checks need to be monitored closer because the newborn could go into fluid overload if only checked once a week. 5. Intake monitoring is needed so a measure can be made of the amount of fluid the body is retaining and voiding.

Parents have been given the news that their unborn baby will be born with Tetralogy of Fallot. The parents are asking the nurse what the defects will consist of with this diagnosis. The nurse knows the defects are: (Select all that apply.) 1. VSD 2. Pulmonary stenosis 3. Overriding aorta 4. Thickening of the left ventricle 5. Patent Foramen Ovale

ANS: 1, 2, 3 Feedback 1. Present in Tetralogy of Fallot 2. Present in Tetralogy of Fallot 3. Present in Tetralogy of Fallot 4. Thickening of the right ventricle occurs because of the restrictive blood flow. 5. The patent foramen ovale is not present in Tetralogy of Fallot.

74. The nurse is aware that the risk factors for the development of congenital heart disease include: (Select all that apply.) 1. Family history of congenital heart disease or genetic disorders. 2. Exposure to alcohol, cocaine, or phenytoin. 3. Exposure to teratogens. 4. Weight at birth. 5. Infants of diabetic mothers.

ANS: 1, 2, 3, 5 1. A family history of CHD or genetic defects significantly increases the chances of an infant being born with a CHD. 2. Alcohol, cocaine, or other drugs act as teratogens to the developing fetus. 3. A teratogen results in congenital defects, including heart disease. 4. Weight at birth has no influence on the development of CHD. 5. Infants of diabetic mothers have a higher incidence of developing CHD.

73. A nurse is giving discharge instructions to parents taking a newborn home with apneic episodes. The newborn has an apnea monitor for home. The instructions should include: (Select all that apply.) 1. How to clean the monitor pieces. 2. Allowing the monitor to be off when the parents are sitting with the newborn in an awake state. 3. Never take the monitor off. 4. Take the monitor off when bathing the baby. 5. Reset the alarm limits if the monitor is ringing frequently.

ANS: 1, 2, 4 Feedback 1. Keeping the pieces clean will aid in decreasing the chances for infection and help maintain a working monitor. 2. The newborn can be off the monitor while awake, and being supervised helps decrease skin breakdown. 3. The monitor should be taken off for periods while the newborn is awake and supervised. 4. Since the monitor is electric, it should not become wet at any time. 5. The alarm limits are prescribed by a provider and should not be reset.

The nurse is conducting a class for parents of children with asthma. After covering the topic of asthma triggers, the nurse asks for feedback on the information. Which statements indicate the parents understand actions to reduce asthma triggers? Select all that apply. 1. "I think that we will need to stop using the fireplace." 2. "We will be rethinking the possibility of a family pet." 3. "Now may be the time for relocating to a warmer climate." 4. "No more going outside without a mask from now on." 5. "Reorganizing our schedules will definitely reduce stress."

ANS: 1, 2, 5 This is correct. Smoke from any source is a commonly recognized trigger for asthma; understanding the need to stop using the fireplace is indicative of an appropriate action. This is correct. Pet dander is a commonly recognized trigger for asthma; voicing the need to reconsider getting a family pet is indicative of an appropriate action. This is correct. Stress is a commonly recognized trigger for asthma; reorganizing schedules may be an effective way to reduce stress. This is incorrect. Cold air is commonly recognized as a trigger for asthma; however, moving to a warmer climate is not necessary. The patient needs to place a scarf over the mouth and nose before being exposed to cold air. This is incorrect. Viral infections are commonly recognized as a trigger for asthma; however, wearing a mask whenever going outside the home is not a necessary action. Staying away from people who are ill, good hand hygiene, and promoting overall good health is important. Masks may be considered during flu season.

71. Identify common characteristics in an assessment of a child with a truncus arteriosus heart defect. Select all that apply. 1. Cyanosis 2. Narrow pulse pressures 3. Grunting and retractions while breathing 4. Diaphoresis 5. Bradycardic

ANS: 1, 3, 4 Feedback 1. Cyanosis is present because of an incompetent truncal valve. 2. Pulse pressures widen with the disorder. 3. These reactions are present because of the lack of blood being oxygenated and the vascular resistance on the vessel going over the right and left ventricles. 4. Diaphoresis is present because of the increased workload on the heart and also due to vascular resistance. 5. The heart rate is usually within normal limits.

The nurse is providing teaching to the parent of a toddler 2 years of age diagnosed with otitis media. The toddler presented with a fever of 100.9°F (38.3°C) and does not indicate symptoms related to pain. Which information does the nurse give the parent when the physician orders 48 to 72 hours of supportive care? Select all that apply. 1. Provide age-appropriate analgesics as needed. 2. Administer all of the prescribed antibiotic. 3. Support hydration with fluid increases. 4. Monitor temperature and report increases. 5. Apply topical steroid preparations as instructed.

ANS: 1, 3, 4 The nurse will instruct the parent to administer age-appropriate analgesics as needed. This supportive care is recommended if a child older than 23 months is not in severe discomfort and fever is lower than 102.2°F. Observation for 48 to 72 hours is an important option. The nurse will instruct the parent to avoid dehydration in the child by frequently offering fluids. The nurse will instruct the parent to monitor the child's temperature and report increases, especially if it is 102.2°F or higher. This is incorrect. Antibiotics will not be prescribed immediately and then will be prescribed conservatively. Supportive care is cost-effective and decreases overuse of antibiotics that lead to resistant strains of pathogens. Topical antibiotics, an acidifying agent, and steroid preparation are prescribed for otitis externa.

The nurse in a pediatric office is aware that certain factors may be indicators of heart disease in children. Which children does the nurse recognize with manifestations related to heart disease? Select all that apply. 1. The newborn with dysmorphic facial features 2. The school-age patient with slow capillary refill 3. Identification of scoliosis in a new adolescent patient 4. An infant who is unable to meet developmental milestones 5. A toddler with clubbing and erythema of the fingers and toes

ANS: 1, 3, 4, 5 1. This is correct. Dysmorphic facial features can be identified at any age and can be indicative of heart disease. 2. This is incorrect. Slow capillary refill can be from multiple causes, such as poor circulation, low oxygenation, anemia, exposure to cold, and/or stress. The finding alone does not cause the nurse to recognize a manifestation of heart disease. 3. This is correct. When assessing a new patient, the nurse recognizes that scoliosis is common in adolescents with congenital heart disease. The nurse will perform additional assessment. 4. This is correct. When an infant is unable to meet developmental milestones, the nurse recognizes that children with congenital heart defects are more likely to have developmental disabilities. 5. This is correct. Clubbing and erythema in fingers and toes may result from longstanding cyanosis due to increased formation and enlargement of the capillaries in the periphery to improve circulation. This is a manifestation of heart disease in a toddler.

57. You are caring for a client who is receiving a bone marrow transplant to treat cancer. Important aspects of care to consider include: (Select all that apply.) 1. A patient receiving BMT is immune-suppressed and should be on reverse precautions. 2. BMT is done as a first line of treatment for children with cancer. 3. BMT should be given to the patient after they have received chemotherapy or radiation treatment so that his/her newly given healthy cells are not harmed. 4. Donor matches do not have to be exact and are easily found. 5. There are three major types of BMT, including allogeneic, autologous, and peripheral stem cell transplant (PSCT).

ANS: 1, 3, 5 Feedback 1. Reverse precautions help the patient to not get infections from other individuals. 2. BMT is usually a last resort for treatment. 3. Chemotherapy and radiation kill cells, so the BMT should be done after this to keep the healthy cells alive. 4. Donor matches are difficult to find, and an exact match must be used. 5. The three types of BMT can benefit many different patients.

74. Parents are attending a pre-baby class and receiving information on SIDS. Identify important information the nurse should provide during the course. Select all that apply. 1. A firm mattress 2. A bendy bumper around the entire bed 3. A pillow 4. Tight-fitting sheets 5. A well-ventilated room

ANS: 1, 4, 5 Feedback 1. A firm mattress keeps the baby from sinking into the bedding, thus preventing suffocation. 2. Bendy bumpers can create pockets for the infants face to become stuck, thus creating a suffocation risk. 3. A pillow is too bulky and can cause an infant to become stuck, thus creating a suffocation risk. 4. Tight-fitting sheets decrease the chance for suffocation because there is little room for the infants head to get stuck. 5. A well-ventilated room creates air movement and a good exchange of oxygen and carbon dioxide.

71. Signs that a child is exhibiting respiratory distress include: (Select all that apply.) 1. Nasal flaring. 2. Synchronized rise and fall of the abdomen and the chest. 3. A capillary refill of less than three seconds. 4. Grunting. 5. Intercostal retractions.

ANS: 1, 4, 5 Feedback 1. Nasal flaring indicates that the child is struggling with breathing. 2. Synchronized rise and fall is a normal breathing pattern of a child. 3. A capillary refill of less than 3 seconds is normal for a child. 4. Grunting indicates that the child has to exhale harder than normal, thus indicating respiratory distress. 5. Intercostal retractions indicate that the child needs to use accessory muscles, creating respiratory distress.

67. A child has been struck by a car and has perfuse bleeding from the left leg. The nurse at the scene is assessing the patient. Identify the signs and symptoms the patient will have if hypovolemic shock is occurring. Select all that apply. 1. Sweating 2. Ruddy skin 3. Bounding pulses in the lower extremities 4. A rapid respiratory rate 5. A rapid heart rate

ANS: 1, 4, 5 Feedback 1. Will be present if the child is in hypovolemic shock 2. The skin will be pale. 3. The pulse will be weak in the extremities because the body is trying to perfuse vital organs only. 4. The respiratory rate will be increased due to the ineffective pumping by the heart to the lungs. 5. The heart rate will be increased because the heart is trying to push the limited amount of blood available to the body.

73. Cardiovascular disease in children can be classified according to: (Select all that apply.) 1. Increased or decreased pulmonary blood flow. 2. Acyanotic flow. 3. Obstructive flow. 4. Acquired. 5. Mixed blood flow.

ANS: 1,3,4,5 Feedback 1. Nurses know that one of the classifications of heart disease is increased or decreased pulmonary blood flow. 2. This is a current incorrect classification of cardiovascular disease in children. 3. Obstructive flow is a current classification of cardiovascular disease. 4. Acquired cardiovascular disease is accurate. 5. Mixed blood flow is accurate for cardiovascular disease.

47. The mother of a 3 year old complains to the nurse after the physician leaves the room, saying, My baby is sick with a fever, bad cough, runny nose, and flushed cheeks. He didnt give me any medicine to make him better! What is the nurses best response? 1. It is okay to give your child over-the-counter medicine. Just make sure you get a cold and fever medication. 2. The doctor stated that he believes this to be a virus, so antibiotics will not relieve the symptoms. 3. The best way to treat your child is to give him plenty of fluids, bedrest, and coloring books. 4. The doctor believes this to be a viral illness, so you can use over-the-counter cold medications as long as they say pediatric on the label.

ANS: 2 1. You should not use cold medicine in children under the age of 5. 2. The doctor stated that he believes this to be a virus, so antibiotics will not relieve the symptoms. 3. Fluids, bedrest, and limiting contacts would help the management of current symptoms. This does not address the mothers concern of not receiving medication. 4. You should not use cold medicine in children under the age of 5.

28. An 18-pound, 12-month-old child with a known diagnosis of Tetralogy of Fallot with pulmonary atresia has been ordered to receive a calorie intake of 150 calories/kg per day. The total caloric intake prescribed is: 1. 1528 2. 1227 3. 2700 4. None of the above

ANS: 2 18/2.2= 8.18 kg 8.18 kg x 150 calories= 1227 calories per day Feedback 1. Too many calories per day 2. Adequate calories per day 3. Too many calories per day 4. One answer is correct.

The parents of three children aged 4, 6, and 9 years are preparing to travel abroad as missionaries. The children are in good health and up to date for immunizations; however, the parents are concerned about the high level of TB in the area to which they are assigned. Which recommendation does the nurse make for the protection of the children? 1. Protect the children with good nutrition. 2. Have the children receive the BCG vaccine. 3. Arrange for monthly TB testing for the family. 4. Start the children on preventive medication.

ANS: 2 Before traveling abroad, the children should get a BCG vaccine, which is a live attenuated strain of Mycobacterium bovis. The vaccine is not widely used in the United States because there is less risk for infection in the United States. It provides incomplete protection, and other precautions are warranted. The BCG vaccine does provide better protection for children than adults. Good nutrition and providing overall good health to prevent infections is beneficial for the entire family regardless of their location. The diagnosis of TB in children is challenging, because symptoms are nonspecific and vary and are dependent on the age of the child. Children 5 to 10 years of age may not have clinical symptoms, but an x-ray will confirm if TB symptoms exist. Laboratory tests are less likely to be positive in children, and diagnosis is often made without laboratory confirmation. The children can be protected with nutrition, hand hygiene, avoidance of persons identified with TB, and the BCG vaccine. Preventive medications can have undesirable side effects.

55. A 12 year old comes in with her mother and has the following symptoms: a 40.0 C fever, chills, coughing, and chest pains. Her mother states that she just finished Amoxicillin for strep throat and her chest x-ray shows consolidation. Based on these findings, what would be possible nursing interventions to manage this patient? 1. Monitor oxygenation status and results of sputum culture, CBC, PTT, and sweat chloride test from the laboratory 2. Monitor respiratory, oxygenation, and hydration status and give antibiotics as ordered 3. Monitor respiratory and oxygenation status and give pneumococcal vaccine injection as ordered 4. Monitor oxygenation and hydration status and inform mother that antibiotics would be ineffective for her daughter

ANS: 2 Feedback 1. A PTT and sweat chloride test are not needed at this time because this is the initial incidence of respiratory issues. 2. Although RSV causes 80%85% of all pneumonia in children, the nurse suspects bacterial pneumonia due to the recent strep infection. This is why antibiotics are expected to be ordered. 3. A pneumococcal vaccine should be given prior to the illness. 4. Although RSV causes 80%85% of all pneumonia in children, the nurse suspects bacterial pneumonia due to the recent strep infection. Antibiotics can be effective in this situation.

13. A 10-month-old boy is being given a sweat test because: 1. The child has had several high fevers. 2. The test is assessing for cystic fibrosis. 3. The test is assessing for respiratory failure. 4. The child does not demonstrate thermoregulation.

ANS: 2 Feedback 1. A child with a high fever does not require a sweat test. Sweating can be a normal occurrence during fevers. 2. The sweat test is a common test for cystic fibrosis diagnostics. 3. The sweat test will not give an indication as to respiratory failure. 4. The sweat test does not deal with the thermal regulation of a child.

15. A mother has brought her 18-month-old boy into the pediatric clinic because of irritability, high fever, and has been tugging at his ear for the last 24 hours. The nurse would anticipate which of the following orders? 1. Place the child NPO and attempt to get a head CT. 2. Administering antibiotics for otitis media and acetaminophen for pain and fever control. 3. No orders, as this is a common childhood ailment that requires no interventions. 4. Admitting the child to the hospital to control the high fever.

ANS: 2 Feedback 1. A child with a high fever is normally irritable and this would not be an indication for a head CT as a first priority. 2. The tugging at the ear can be an indication of a child having otitis media. Acetaminophen can help control the ear pain and fever in order to help decrease irritability. 3. Due to the high fever and irritability, the child is demonstrating pain. An intervention is needed. 4. Not enough information is provided to indicate the fever level. Normally this can be controlled at home with acetaminophen.

17. The laboratory test ordered to determine the presence of the Human Immunodeficiency Virus antibodies is a: 1. Complete blood cell count. 2. Western blot immunoassay. 3. Bone marrow aspiration. 4. Biopsy of the tumor.

ANS: 2 Feedback 1. A complete blood cell count is a common test to identify other types of infections and anemia. 2. The Western blot test confirms the presence of Human Immunodeficiency Virus antibodies. 3. A bone marrow aspiration is used in patients with cancer, not HIV. 4. A tumor biopsy is not a common test for identifying a patient with HIV.

8. What is included in neurologic checks for children of all ages with a brain tumor? 1. Papillary response, head circumference, vital signs 2. Motor activity, papillary response, vital signs 3. Level of consciousness, palpate fontanels, motor activity 4. Blood pressure, head circumference, level of consciousness

ANS: 2 Feedback 1. A head circumference is not an indicated neurological check for all ages of children. 2. The indicators of the neurological system for all ages include a widening pulse pressure, papilledema, and the ability to move each extremity. 3. Vital signs need to be assessed in a child with a brain tumor to closely monitor changes in pulse pressures. Older children will have closed fontanels. 4. Head circumference is not measured in all children.

41. A newborn is experiencing apneic episodes. The nurse should do which of the following when an episode occurs? 1. Give the newborn CPR 2. Stimulate the newborn by rubbing its back 3. Reposition the newborn 4. Hold the newborn

ANS: 2 Feedback 1. An assessment to see if the newborn has a heart rate is needed. 2. Stimulating the newborn may help his/her breathing. 3. Repositioning the newborn is important and should occur after breathing stimulation is provided. 4. Holding the newborn will not stimulate him/her to breathe.

30. What type of bone marrow transplant must have compatible human leukocyte antigen in both the donor and recipient? 1. Intravenous transplant 2. Allogeneic 3. Autologous 4. Peripheral stem cell transplant

ANS: 2 Feedback 1. An intravenous transplant does not require compatible human leukocyte antigens. 2. Allogeneic transplants have to be compatible in order to decrease the risk for rejection. 3. Autologus does not have the same human leukocyte antigen and increases the chance for rejection. 4. Stem cell transplants must be from the childs own stem cells in order to have compatibility.

6. When assessing a newborn, a nurse should check capillary refill: 1. On the fingernail beds. 2. On the sternum. 3. On the arm. 4. On the hand.

ANS: 2 Feedback 1. Because of peripheral cyanosis, the fingernail beds will not respond quickly for an adequate measurement of capillary refill. 2. The sternum responds quickly for an adequate measurement of capillary refill. 3. The hand does not respond quickly for an adequate measurement of capillary refill. 4. The hand does not respond quickly for an adequate measurement of capillary refill.

21. A child with a known diagnosis of coarctation of the aorta will have an increase in: 1. Blood pressure in the lower extremities. 2. Blood pressure in the upper extremities. 3. Blood pressure in the heart. 4. Blood pressure in the aortic arch.

ANS: 2 Feedback 1. Because of the low flow of blood, the lower extremities will have a lower flow of blood. 2. The upper extremities will demonstrate an increase in blood pressure. 3. The blood pressure affects the extremities. 4. The blood pressure affects the extremities.

45. The mother of a child with cystic fibrosis calls the triage nurse and asks which type of antihistamine would be the most beneficial for her sons head cold. The nurse should: 1. Recommend Benadryl for her son. 2. Discourage the use of antihistamines because the drug can dry out the mucous and make it harder to expel. 3. Encourage the mother to give the child a dose of the antihistamine every four hours. 3. Recommend any over-the-counter antihistamine that states it is a pediatric formula.

ANS: 2 Feedback 1. Benadryl will dry out the mucous membranes and cause further problems for the child. 2. Discouragement of antihistamine usage is important because the medication can dry out the mucous membranes too much for a child with cystic fibrosis. 3. Antihistamine medication can dry out the mucous membranes too much for a child with cystic fibrosis, creating further problems. 4. Antihistamine medication can dry out the mucous membranes too much for a child with cystic fibrosis and create further problems.

42. A former 24-week, premature infant is now adjusted in age to be one year of age. The baby has a known history of bronchopulmonary dysplasia. The parents of the child are asking if their baby will catch up in height and weight to her peers by the time she is 2 years old. The best reply from the nurse would be: 1. Normally, premature infants will be the same height and weight as their peers by their second birthday. 2. The bronchopulmonary dysplasia requires your childs lungs to work harder to breath. This causes the body to have a higher metabolism, so she may remain on the small side for several years. 3. You baby is now healthy and will continue to grow at her own rate. 4. Your baby will remain small for most of her life due to the bronchopulmonary dysplasia.

ANS: 2 Feedback 1. Children with bronchopulmonary dysplasia require high nutritional demands to the body. The growth of children with this diagnosis tends to be slower than their peers. 2. Children with this diagnosis tend to be smaller than their peers for a longer period of time. 3. This is a true statement, but does not address why the child is not growing at the same rate. 4. The childs body can grow and may be the same as peers later in life.

62. Cystic fibrosis is best categorized as: 1. An autosomal recessive disease with deletion of Chromosome 17 that affects the lungs and finances of the parents. 2. An autosomal recessive disorder of the exocrine glands marked by increased mucus and sodium chloride production and decreased pancreatic enzyme production. 3. An autosomal recessive disorder that affects the respiratory, cardiac, and digestive systems. 4. An autosomal recessive disorder that is marked by the increased mucus destruction and decreased pancreatic enzyme production.

ANS: 2 Feedback 1. Cystic fibrosis is an autosomal recessive disorder of exocrine glands and is not seen on chromosome 17. 2. Cystic fibrosis is an autosomal recessive disorder of exocrine glands marked by increased mucus and sodium chloride production and decreased pancreatic enzyme production. 3. Cystic fibrosis is an autosomal recessive disorder that impacts the respiratory and GI tract, not the heart. 4. Cystic fibrosis is an autosomal recessive disorder of the exocrine glands marked by increased mucus and sodium chloride production and decreased pancreatic enzyme production.

2. Classic hemophilia (hemophilia A) involves a deficiency in: 1. Factor V. 2. Factor VIII. 3. Factor IX. 4. Factor XIII.

ANS: 2 Feedback 1. Factor V is not noted in hemophilia A. 2. Administration of Factor VIII, derived from pooled plasma, will increase the clotting ability of the body. 3. Factor IX is not an issue with hemophilia A. 4. Factor XIII is not a factor in hemophilia A

30. The best way to prevent pertussis in children is with: 1. Good hand hygiene. 2. Keeping immunizations up-to-date. 3. Isolation precautions. 4. All of the above are correct.

ANS: 2 Feedback 1. Hand hygiene is important but the pertussis virus is usually airborne. 2. Immunizations help to build immunity to the disease. 3. Isolation precautions are needed after a child has the illness. 4. Immunizations to help build immunity to the disease is the priority.

1. Which of the following is a condition in which the normal hemoglobin is partially or completely replaced by abnormal hemoglobin? 1. Iron deficiency anemia 2. Sickle Cell anemia 3. Leukemia 4. Aplastic anemia

ANS: 2 Feedback 1. Iron deficiency anemia occurs because there is not enough iron to support the production of hemoglobin. 2. Sickle Cell anemia has abnormal hemoglobin S. The deformed cell changes from a round shape to a sickle shape. 3. Leukemia a disease process that causes the destruction of hemoglobin. 4. Aplastic anemia occurs because the bone marrow is not producing enough red blood cells, making the patient anemic.

34. A 6-month-old patient is admitted for iron deficiency anemia. The nurse knows that the child was put at risk for developing the illness when the mother states: 1. I have a house that does not have lead-based paint. 2. He drinks milk instead of formula because it is cheaper. 3. I make sure to monitor the amount of juice intake. 4. I provide water in his bottles one time a day.

ANS: 2 Feedback 1. Lead-based paint can lead to lead poisoning, not anemia. 2. A 6 month old is not able to store iron. Milk is not fortified with iron, thus creating the deficiency for the child. The child should remain on formula until at least 12 months of age. 3. Juice provides empty calories and nutrients and should not be given to an infant. 4. Water should not be given to an infant because it fills the stomach, and the child does not receive the needed nutrition from the formula.

46. The mother of a 16-year-old teen with hemophilia calls the nurse triage and asks if it is appropriate to give her son a dose of NSAIDS to reduce his fever. The nurse knows that: 1. NSAIDS are the most effective treatment for fever reduction and should be given to the teen. 2. NSAIDS are not recommended because it may interfere with platelet formation. 3. NSAIDS are not recommended because they can cause Reyess Syndrome. 4. NSAIDS are only effective when given with large amounts of water.

ANS: 2 Feedback 1. NSAIDS can interfere with platelet formation in patients with hemophilia. 2. Instruction on not giving the medication because of the risk of platelet formation interference is needed. 3. At 16 years of age, Reyes Syndrome is decreased. NSAIDS interfere with the production of platelets for hemophiliacs. 4. NSAIDS do not need to be given with large amounts of water in order to be effective.

6. Which of the following events places a preschool child at high risk for lead poisoning? 1. Using pencils and pens 2. Living in a home built before 1965 3. Drinking from the water fountain at school 4. Climbing on playground equipment

ANS: 2 Feedback 1. Pencils and pens contain a low lead content. 2. Paint used in homes before 1965 contained lead, and the paint has not been removed. 3. Water is tested for lead levels and does not place the child at high risk. 4. Playground equipment has low levels of lead.

44. The purpose of administering surfactant to a preterm neonate is: 1. Because the preterm neonates lungs do not produce it. 2. To prevent the alveoli from collapsing. 3. To help the diaphragm function. 4. Because a preterm neonate needs more surfactant than an older child.

ANS: 2 Feedback 1. Preterm neonates do have some surfactant in the lungs, but not enough to keep the alveoli open for a long period of time. 2. Surfactant is the lubricant in the lungs that allows all for alveoli to remain moist and prevents them from collapsing. 3. The diaphragm is outside of the lung tissue and does not receive surfactant. 4. A preterm neonates needs do not differ from those of an older child.

44. A nursing student is assessing children at a well-child clinic. The nursing student should know that routine monitoring of blood pressure should begin at what age? 1. 4 years of age 2. 3 years of age 3. 10 years of age 4. 12 years of age

ANS: 2 Feedback 1. Screening should be done prior to this age. 2. Screening should begin at this age. 3. Screening should be done prior to this age. 4. Screening should be done prior to this age.

8. Identify the child that is demonstrating acrocyanosis. 1. A newborn shows slow capillary refill. 2. A newborns hands are blue following delivery. 3. A newborn with a fever has red hands. 4. A newborn with a lack of oxygen has blue hands.

ANS: 2 Feedback 1. Slow capillary refill can be demonstrated with acrocyanosis, but is not the cause. 2. The vasoconstriction after birth is a cause of acrocyanosis 3. A newborn with a fever will not demonstrate acrocyanosis. 4. Lack of oxygen will cause central cyanosis.

58. The nurse knows that a child on Digoxin (Lanoxin) requires frequent monitoring of: 1. Sodium levels. 2. Potassium levels. 3. Complete blood count. 4. Creatinine levels.

ANS: 2 Feedback 1. Sodium levels do not affect the function of Digoxin. 2. Potassium levels affect the contractility of the heart muscle and affect whether Digoxin should be administered. 3. Complete blood count would not affect the administration of Digoxin. 4. Creatinine levels do not affect the function of Digoxin.

51. The primary mechanism responsible for the closure of fetal ducts following birth is: 1. Suctioning of the nose and mouth. 2. The first breath of the infant. 3. High carbon dioxide levels. 4. Stimulation of the infant.

ANS: 2 Feedback 1. Suctioning of the nose and mouth do not assist in closing the fetal ducts, but instead result in hypoxia. 2. The first breath by the infant results in higher oxygen levels and relaxation of the pulmonary artery. In turn, the pressure gradients within the heart change and force the fetal ducts to close. 3. High carbon dioxide levels would result in pulmonary vasoconstriction, which would not close the fetal ducts. 4. Stimulation of the infant may assist the infant in taking his/her first breath, but it is the first breath that assists in closing the fetal ducts.

61. The Jones Criteria established in 1944 was established to assist in the diagnosis of: 1. Kawasaki disease. 2. Rheumatic heart disease(RHD). 3. Subacute Bacterial Endocarditis. 4. Toxic Shock Syndrome.

ANS: 2 Feedback 1. The Jones Criteria was established for RHD. 2. The Jones Criteria has established minor and major criteria for the diagnosis of RHD. 3. The Jones Criteria was established for RHD. 4. The Jones Criteria was established for RHD.

31. Which of the following is an accurate description of the physiologic defect caused by anemia? 1. Presence of abnormal hemoglobin 2. Decreased oxygen carrying capacity of blood 3. Increased blood viscosity 4. Depressed hematopoietic system

ANS: 2 Feedback 1. The abnormal hemoglobin may be present, but it contributes to anemia and is not caused by it. 2. The lack of oxygen carrying capacity is a physiological defect caused by the anemia. 3. Anemia causes the viscosity. 4. The decrease in the hematopoietic system is caused by the lack of production, not anemia.

1. An 8-year-old child with a history of cystic fibrosis has a chest that is larger than normal. This type of feature on a child is known as: 1. A concaved chest. 2. A barrel chest. 3. An asymmetrical chest. 4. All of the above are correct.

ANS: 2 Feedback 1. The chest does not bow inward in a child with cystic fibrosis. 2. A barrel chest is common in a child with cystic fibrosis because of the air trapping that occurs within the lungs. 3. The chest is symmetrical in appearance with cystic fibrosis. 4. Not all of the options are correct.

19. A nurse caring for a child with Eisenmengers syndrome should assess for all of the following except: 1. Fatigue. 2. Acrocyanosis. 3. Shortness of breath. 4. Blood pressure.

ANS: 2 Feedback 1. The child will have increased fatigue due to the lack of oxygen in the body. 2. The child will have cyanosis. 3. The child will demonstrate a shortness of breath because of the lack of oxygen being perfused to the lungs. 4. The blood pressure should be assessed to monitor how the heart is pumping.

22. A nurse is assessing a child with coarctation of the aorta. The nurse knows she will find all the following except: 1. Decreased femoral pulses. 2. A report of chest pain. 3. Shortness of breath. 4. Poor growth.

ANS: 2 Feedback 1. The decreased blood flow to the lower extremities is noted in coarctation by assessing the pulses. 2. Coarctation can be asymptomatic. 3. Shortness of breath may be noted when exerting the body. 4. The child will have a slower growth rate than peers.

27. A 2 year old has been placed in contact isolation because of a diagnosis of Respiratory Syncytial Virus (RSV). The father questions why the staff is wearing masks and gowns every time someone comes into the room. The best response would be: 1. The equipment is needed to protect myself and others from your childs illness. 2. Since bronchiolitis is highly contagious for other children, it is important for the staff to wear the equipment to prevent spreading it to others. 3. Every child that comes in with a respiratory illness is required to be in isolation. 4. The equipment is needed to protect your child from acquiring an illness from the staff.

ANS: 2 Feedback 1. The equipment is protecting the health-care worker from transmitting the virus to other patients. 2. Prevention of the spread of the disease is the primary reason for the equipment. 3. Not all respiratory illnesses require isolation. 4. The equipment is protecting the health-care worker from transmitting the virus to other patients.

7. Weak peripheral pulses can indicate: 1. A weak heart. 2. Poor cardiac output. 3. Hypertension. 4. Patent ductus arteriosus.

ANS: 2 Feedback 1. The heart may be weak, but does not indicate that the pulses will be weak. 2. A lower amount of output does not allow for peripheral pulses to be easily felt. 3. If the patient has hypertension, the pulses may be bounding. 4. Patent ductus arteriosus may have bounding pulses.

42. Parents of a 5-year-old with Sickle Cell disease are asking about care during their childs painful episodes. The nurse knows that the pain of Sickle Cell disease is caused by: 1. The lack of iron in the body. 2. The hypoxia that happens to the tissue, causing ischemia. 3. The red blood cell destruction. 4. A buildup of oxygen in the body

ANS: 2 Feedback 1. The lack of oxygen is the causative agent for pain. 2. The ischemia causes the painful episodes. 3. Red blood cells are malformation with this disease, not destroyed. 4. The transport of oxygen to the tissue causes the painful episodes.

36. Typical signs of a child with iron deficiency anemia include all of the following except: 1. Tachycardia. 2. Bradycardia. 3. Thinning of hair. 4. Shortness of breath.

ANS: 2 Feedback 1. The lack of red blood cells does not allow for enough oxygen carrying capacity. 2. Tachycardia is a sign, not bradycardia. 3. Thinning hair is a common sign of long-term iron deficiency anemia in children. 4. Shortness of breath is present because of the lack of oxygen carrying capacity of the red blood cells when the hemoglobin is low.

20. Amantadine hydrochloride has been prescribed for a patient. The nurse knows this medication is used for: 1. Sinusitis. 2. Influenza. 3. Upper respiratory tract infections. 4. Asthma.

ANS: 2 Feedback 1. The medication is not prescribed for sinusitis. 2. The medication helps reduce the symptoms and spread of the influenza virus. 3. Upper respiratory tract infections do not benefit from the use of the medication. 4. Asthma exacerbations do not benefit from the use of this medication.

5. A nurse is attempting to assess the skin color of a child with dark skin. The nurse knows the best place to assess the childs skin color is: 1. The nailbeds. 2. Inside the mouth in the cheek area. 3. The eyes. 4. On the chest.

ANS: 2 Feedback 1. The nailbeds should be used to assess capillary refill. 2. A pen light can be used to examine the inside of a childs mouth in the cheek area for color. 3. The eyes can indicate jaundice, but not any other type of color changes. 4. Capillary refill can be assessed on the chest since the oral mucous membranes are more accurate.

11. A childs ABG results are: pH: 7.14 pCO2: 24.6 HCO3: 8.0 The nurse interprets these results as: 1. Normal ABG. 2. Partially Compensated Metabolic Acidosis. 3. Uncompensated Metabolic Acidosis. 4. Uncompensated Respiratory Acidosis.

ANS: 2 Feedback 1. The pH is acidotic, the CO2 is alkalotic, and the HCO3 is acidotic. Because the pH and the HCO3 are acidotic, it causes the Metabolic Acidosis. Compensation occurs because the pH and the CO2 go in the opposite direction, and the pH is not in the normal range to cause the partial. 2. The pH is acidotic, the CO2 is alkalotic, and the HCO3 is acidotic. Because the pH and the HCO3 are acidotic, it causes the Metabolic Acidosis. Compensation occurs because the pH and the CO2 go in the opposite direction, and the pH is not in the normal range to cause the partial. 3. Compensation occurs because the pH and the CO2 go in the opposite direction, and the pH is not in the normal range to cause the partial. 4. Because the pH and the HCO3 are acidotic, it causes the Metabolic Acidosis.

9. A newborn is born with patent ductus arteriosus. If the patent ductus arteriosus does not close during this time, the newborn will exhibit: 1. Narrowing pulse pressures. 2. Widening pulse pressures. 3. A decreased heart rate. 4. Quick capillary refill.

ANS: 2 Feedback 1. The pulse pressures widen because of the low pressure gradient within the heart. 2. The widening occurs because of the low pressure gradient within the heart. 3. The heart rate will increase because of not perfusing to the lungs. 4. The capillary refill will be sluggish because the oxygenated blood is not going out to the rest of the body.

39. A nurse is discussing the process of hematopoiesis with a new nurse. It is important for the new nurse to understand that: 1. Red blood cells live for less than five days. 2. Red blood cells are produced with erythropoietin and iron. 3. Red blood cells develop in the long bones. 4. Hemolysis of red blood cells occurs in the kidneys.

ANS: 2 Feedback 1. The red blood cells life span does not influence the hematopoiesis process. 2. Erythropoeitin and iron are included in the process of hematopoiesis. 3. Red blood cell production occurs in the bone marrow. 4. Hemolysis of red blood cells does not affect hematopoiesis.

31. Identify the heart condition that will cause profound cyanosis, shock, and congestive heart failure if the PDA closes. 1. Truncus arteriosus 2. Total anomalous pulmonary venous return 3. Transposition of the greater vessels 4. ASD

ANS: 2 Feedback 1. The truncal valve remains open, so these signs and symptoms will not be noted. 2. The PDA must remain open to allow the oxygen from the pulmonary veins to move to the left atrium. 3. Transposition causes cyanosis, but does not lead to shock. 4. An ASD does not lead to shock.

1. The right ventricle is responsible for: 1. Pumping blood to the left atrium. 2. Pumping deoxygenated blood to the lungs. 3. Pumping oxygenated blood to the body. 4. Returning oxygenated blood from the lungs.

ANS: 2 Feedback 1. The ventricle pumps blood to the lungs via the pulmonary artery. 2. The right ventricle pumps blood to the lungs to become oxygenated. 3. The left ventricle pumps oxygenated blood to the body. 4. The pulmonary artery returns the oxygenated blood from the lungs.

23. A newborn is lying in his crib in the hospital nursery. The nurse picks up the newborn to prepare for a feeding and notes frothy oral secretions around the newborns mouth. The nurse should: 1. Wipe the newborns mouth and give the feeding. 2. Clean the newborns mouth and notify the doctor of the findings. 3. Feed the newborn. 4. Take the baby to the mother to feed.

ANS: 2 Feedback 1. The wiping the mouth for an assessment is needed, but the newborn should not be fed because the secretions are an indication of lack of secretion drainage. 2. These actions should occur because the child is at risk for tracheal esophageal atresia. 3. The newborn should not be fed because the secretions are an indication of lack of secretion drainage and increases the chance for aspiration. 4. The newborn should not be fed because the secretions are an indication of lack of secretion drainage and increase the chance for aspiration.

40. An 11-year-old child has been brought to the emergency room because the mother has noted petechiae on his elbows. When the nurse assesses the child, she notes that the boy has purpura on his legs. What action should be taken? 1. Notify the doctor of the medical emergency. 2. Assess if the child has had hematemesis. 3. An abuse investigation should be discussed with a social worker. 4. All of the above would be correct actions.

ANS: 2 Feedback 1. This is not a medical emergency, but should be assessed. 2. The signs are consistent with hematemesis. 3. Because of where the marks are on the childs body, further assessment is needed before notifying a social worker. 4. The signs are consistent with hematemesis.

55. In a newborn nursery, a nurse indicates she hears a soft murmur in one of the newborns. After obtaining blood pressures on all four extremities, she finds that the blood pressure is higher in the right arm than the right leg. The nurse knows this can indicate: 1. Nothing, as this is a normal finding. 2. Coarctation of the aorta. 3. Ventricular septal defect. 4. Shock due to poor perfusion in the lower extremities.

ANS: 2 Feedback 1. This is not a normal finding. 2. This is a predominant sign of COA due to a narrowing of the aortic arch after the innervation to the upper extremities, thus resulting in a lower blood pressure in the lower extremities. 3. This is not a sign of VSD. 4. Shock would indicate a low blood pressure in all extremities.

53. A 15 month old admitted with croup is sleeping in a cool mist tent. The nurse checks on him and notices that he is retracting and tachypneic. What is the first thing she should do? 1. Increase the oxygen flow to the tent 2. Check the childs pulse oximetry 3. Check the childs temperature 4. Notify the physician

ANS: 2 Feedback 1. This is not the first intervention. A pulse oximetry should be assessed to identify the need for oxygen. 2. The first intervention should be to check the childs pulse oximetry. 3. Fever can cause tachypnea. This is not the first action needed. 4. Notifying the physician is not the first action needed.

26. Pulmonary stenosis causes an increased workload on: 1. The left atrium. 2. The right ventricle. 3. The left ventricle. 4. The right atrium.

ANS: 2 Feedback 1. This type of stenosis occurs in the right ventricle. 2. The right ventricle has an increased workload because of the lack of blood being pushed out of the heart. 3. The left ventricle does not have the workload because it is pushing oxygenated blood to the body. 4. The right atrium is filling with blood and does not have the resistance to push it to the lungs, thus decreasing the workload.

2. The heart valve that connects the left atria and the left ventricle is: 1. The tricuspid valve. 2. The bicuspid valve. 3. The pulmonic valve. 4. The aortic valve.

ANS: 2 Feedback 1. This valve connects the right atria to the right ventricle. 2. This valve connects the left atria and the left ventricle. 3. This valve connects the right ventricle and the pulmonary artery. 4. This valve connects the left ventricle and the ascending aorta.

The pediatric nurse is reviewing anatomy and physiology in order to have a better understanding of the pediatric respiratory system. The nurse is aware that fluid in the chest cavity can be normal. Which application of this knowledge is correct? 1. Pleural fluid is abundant at birth and decreases over the lifetime. 2. Only enough fluid is present to promote painless movement. 3. Fluid will accumulate in the plural cavity from immobility. 4. Infections such as pneumonia cause fluid in the plural cavity.

ANS: 2 This is correct. There are two pleural membranes: one around the lungs and one covering the inside of the pleural cavity. The two pleural membranes are normally separated by only enough fluid to lubricate the surfaces for painless movement. Fluid is not abundant in the plural cavity at birth; fluid is in the lungs at birth. It is suctioned in order to promote normal respirations. Normal fluid in the pleural cavity does not decrease over the lifetime. Fluid can accumulate in the lungs as a result of immobility. Pneumonia is an infection that causes fluid to build up in the lungs.

70. Identify the characteristics of a patient with Tetralogy of Fallot with pulmonary atresia. Select all that apply. 1. Ventricle defects 2. Anal anomalies 3. Transesophageal anomalies 4. Atrial anomalies 5. Acrocyanosis

ANS: 2, 3 Feedback 1. The defects are not in the ventricles. 2. The defects of the heart are usually associated with other defects, primarily in the anal and transesophageal regions. 3. The defects of the heart are usually associated with other defects, primarily in the anal and transesophageal regions. 4. Atrial anomalies are not noted in a patient with these defects. Cyanosis is noted in the patient.

The nurse is caring for a newborn diagnosed with esophageal atresia and tracheoesophageal fistula. Which information does the nurse provide to the parents? Select all that apply. 1. Prenatal conditions that contribute to the problem 2. The manifestation supporting the diagnosis 3. Diagnostic tests performed since the birth 4. Methods of treating the condition 5. Actions for promoting recovery

ANS: 2, 3, 4, 5 This is correct. The parents need to know the manifestations that support the diagnosis. The newborn is likely to experience respiratory distress within minutes, days, or weeks of birth; excessive oral secretions; cyanosis; coughing spells; abdominal distention; and upper airway instability. This is correct. The nurse will inform the parents that x-rays, tracheoscopy, echocardiography, and ultrasound was performed after the birth and with the onset of manifestations. This is correct. The nurse will inform the parents that the newborn will require surgery for the repair of the defect. After the physician acquires an informed consent, the nurse will ask about questions and concerns. This is correct. Parent teaching aimed at recovery of the newborn will include information about preventing aspiration, feeding, positioning, and the importance of adherence to frequent follow-up appointments. Parent teaching aimed at recovery of the newborn will include information about preventing aspiration, feeding, positioning, and the importance of adherence to frequent follow-up appointments. This is incorrect. Even if known, the nurse will not present information about any prenatal conditions that contributed to the newborn's condition. The physician may need to discuss the topic after the initial treatment is performed.

72. A nurse assessing a 6 year old with cardiomyopathy would anticipate which of the following signs in his assessment? Select all that apply. 1. A murmur with a thrill 2. Fatigue when eating 3. Dysrhythmia after playing with toy cars 4. Sweating while sitting in bed watching cartoons 5. Dizziness when standing at the bedside

ANS: 2, 3, 5 Feedback 1. The murmur will be a gallop. 2. Fatigue occurs with many activities, including eating. 3. Dysrhythmias can occur at any time. 4. Sweating occurs when eating, not during quiet times. 5. Dizziness while standing or changing positions is common.

The nurse is providing postoperative teaching to the parents of a preschool child after a tonsillectomy. For which events does the nurse prompt the parents to contact the physician? Select all that apply. 1. The child keeps an emesis basin close by. 2. The child is frequently swallowing without food or fluids. 3. Bright red blood is noticed in the child's mouth. 4. The child is asking for ice chips and popsicles. 5. The child refuses pain pills because it hurts to swallow.

ANS: 2, 3, 5 This is correct. Frequent swallowing without the presence of food or fluid is an indication of bleeding; the parents are instructed to call the physician if this occurs. This is correct. Bright red blood in the child's nose or mouth is indicative of bleeding, and the physician needs to be called. This is correct. If the child refuses to take pain pills because it hurts to swallow, the doctor needs to be contacted. It is likely the prescription can be changed to a liquid.. It is not unusual for a child to experience nausea from drainage after a tonsillectomy. There is no need to contact the physician unless the nausea is severe and there is a risk for vomiting This is incorrect. The child's willingness to eat ice chips and popsicles will sooth the throat, help to stop bleeding, and promote hydration.

2. The father of a 13-year-old boy with sinusitis calls the triage nurse at the pediatric clinic to ask what he can do to rest. The nurse should advise the father to: (Select all that apply.) 1. Place a cold compress on the infected sinus areas. 2. Have the child blow his nose with one nostril closed off at a time. 3. Use a warm mist humidifier in his bedroom. 4. Use saline drops to help clear the nasal passage. 5. Use a bulb syringe to remove secretions.

ANS: 2, 4, 5 Feedback 1. Cold compresses will not encourage drainage. 2. Attempting to blow a nose with one nostril closed at a time helps provide pressure to remove the secretions. 3. A cool mist humidifier should be used to help reduce the chance of steam burns. 4. Saline drops can keep the airways moist and help remove secretions. 5. The child is too old for bulb syringe suction. Blowing the nose is just as effective.

The nurse is preparing an 8-year-old patient for a cardiac catheterization. Which intervention will the nurse initiate immediately postprocedure? 1. Observe for signs and symptoms of infection. 2. Hold food and fluids until gag reflex returns. 3. Keep the involved extremity straight for 4 to 6 hours. 4. Notify physician if green or yellow drainage is noted.

ANS: 3 1 This is incorrect. It is not likely for the patient to develop an infection immediately following a cardiac catheterization. 2 This is incorrect. The patient will receive a sedative before the procedure to reduce anxiety; a local anesthetic is used at the puncture site. Neither intervention is likely to cause an absent or depressed gag reflex. 3 This is correct. Immediately after the procedure the nurse will assure that the limb used for cardiac catheterization is kept straight with no movement for 4 to 6 hours. The child should be positioned flat on the back; a sandbag may be used on the extremity. All precautions are to prevent bleeding from the puncture site. 4 This is incorrect. Immediately after a cardiac catheterization, the nurse will not expect to see yellow or green drainage, which is indicative of an infection. The nurse will include this intervention in parent teaching.

46. The mother of an 18 month old states that she is concerned due to the fact that her child has been diagnosed with otitis media three times in the last year. Which answer would be appropriate to alleviate the mothers concerns? 1. A childs airway is short and narrow. As the child grows, the airway will grow, and the number of alveoli will increase. 2. A childs tonsils are larger than an adults and block emptying of the Eustachian tubes. As the child grows, the tubes get longer even though tonsils dont change. 3. A childs Eustachian tubes are shorter and more horizontal, allowing nasopharyngeal secretions to enter. As the child grows, the incidence of OM will decrease. 4. A childs larynx is more flexible than an adults and easily stimulated to spasm. As he grows, he will be less sensitive to laryngospasms and pooling of secretions.

ANS: 3 1. Although choice 1 is correct, it does not address the ears and recurrent infection. 2. A childs tonsils are not larger than an adults. They do not block the emptying of the Eustachian tubes. 3. A childs Eustachian tubes are shorter and more horizontal, allowing nasopharyngeal secretions to enter. As the child grows, the incidence of OM will decrease. 4. A childs larynx is not more flexible than an adults

41. A father is discussing dietary needs for his son, who has a diagnosis of CHF. The nurse knows the father understands the dietary needs when he states: 1. I can let my son have french fries once a week. 2. I will need to make sure he drinks eight 8-ounce glasses of water every day. 3. When having scalloped potatoes and ham for dinner, we should plan an alternative meal for my son. 4. I will need to give my child his diuretic at bedtime because food can interfere with the medication action.

ANS: 3 Feedback 1. A diet that is low in sodium is needed. 2. Close monitoring of fluid intake is needed because the heart can easily become overloaded with fluid. 3. Ham has a large concentration of sodium, thus the child should have an alternative meal. 4. The diuretic should be given earlier in the day to decrease the number of times needed to urinate during sleep time.

39. A diet for a child with cystic fibrosis should include: 1. Foods with high protein and high fat content. 2. Foods with low fat and high protein content. 3. A daily dose of fat-soluble vitamin supplements. 4. A daily dose of water-soluble vitamin supplements.

ANS: 3 Feedback 1. A diet with a high fat content can cause digestion issues because of the lack of enzymes. 2. A diet with low protein is needed for the child to aid in health. 3. The fat-soluble vitamins are needed because the child is not able to digest fat easily. 4. A child with cystic fibrosis should be able to receive the needed water-soluble vitamins in a regular diet.

12. A child has the following results for an ABG: pH: 7.42 pCO2: 43.9 HCO3: 26.8 The nurse interprets these results to be: 1. Compensated Respiratory Acidosis. 2. Compensated Respiratory Alkalosis. 3. Normal ABG. 4. Compensated Metabolic Acidosis.

ANS: 3 Feedback 1. All results are within normal range and are not causing acidosis or compensation. 2. All results are within normal range and are not causing alkalosis or compensation. 3. All results are within normal ranges, thus this is a normal ABG finding. 4. All results are within normal range and are not causing compensation or acidosis.

41. The doctor has prescribed an injection of solu-medrol and occult of all stools for a child with acquired thrombocytopenia. Why should the nurse question the order? 1. Blood in the stools is rare with this illness. 2. Steroids are not an effective treatment. 3. Injections should not be given to a child with this condition. 4. None of the orders are appropriate for a child with this illness.

ANS: 3 Feedback 1. Blood in the stools is possible with this disease process. 2. Steroids are a common treatment for thrombocytopenia. 3. Injection can further exacerbate the condition and should not be given to the child. 4. An injection should not be given to the child.

4. A definitive diagnosis for leukemia is based on results of: 1. Fatigue and pallor. 2. A urinalysis. 3. A bone marrow aspirate. 4. A history and a physical.

ANS: 3 Feedback 1. Fatigue and pallor are presumptive signs of leukemia. 2. A urinalysis does not give indications as to a diagnosis for leukemia. 3. The abnormal cells of leukemia are found in the bone marrow. 4. A history and a physical will help identify signs and symptoms, but not give a definitive diagnosis.

10. A nurse is assessing a newborn with a known patent ductus arteriosus defect of the heart. The mother asks when she can start breastfeeding her infant. The best explanation by the nurse would be: 1. The newborn will need to have the defect repaired before oral feedings can start. 2. The newborn will need to have extensive rest time between feedings, so plan on breastfeeding one time, then we will give a nasogastric feeding the next time. 3. The nursing staff will monitor the newborn during feedings because she may sweat and have increased difficulty breathing. 4. The newborn should have no issues while breastfeeding.

ANS: 3 Feedback 1. Feedings can begin before the repair if the newborn does not demonstrate difficulties in cardiac and respiratory status. 2. Rest time will be needed before and after feedings, but there is no need to alternate between breast and nasogastic feedings. 3. Monitoring will enable the nurse to assess when the newborn needs a break in feedings. 4. The newborn will have some issues with feedings because of the heart issues.

16. Common adverse effects of chemotherapy are nausea and vomiting. As a result, the nurse should initiate which of the following nursing actions? 1. Remove food with a lot of color. 2. Wait until the nausea begins to administer the antiemetic as ordered. 3. Give an antiemetic 30 minutes before the start of chemotherapy. 4. Establish a nothing by mouth status during chemotherapy.

ANS: 3 Feedback 1. Food in general may make the child have nausea and cause vomiting. 2. An antiemetic should be provided 30 minutes prior to chemotherapy to prevent the feelings of nausea and vomiting. 3. The onset of the antiemetic will occur with the start of chemotherapy and prevent nausea. 4. The patient can have food throughout chemotherapy if they feel well enough for food.

52. What is the most accurate statement regarding Palivizumab? 1. It is a humanized monoclonal antibody given as an IM injection before the start of HPV season. 2. It is recommended for premature infants with 29-35 weeks gestation, children with congenital heart defects, and the elderly. 3. It is costly and is given usually between October to May in a series of five injections. 4. Before administering, you need to evaluate results of complete blood count and electrolyte panel from the laboratory.

ANS: 3 Feedback 1. Given prior to RSV season 2. Not given to the elderly 3. It is given prophylactically before the start of RSV season. The nurse needs to evaluate platelets and coagulants before administering. 4. The nurse needs to evaluate platelets and coagulants before administering.

36. Children with cystic fibrosis should be frequently checked for: 1. Hypernatremia. 2. Hypocalcemia. 3. Hyponatremia. 4. Hypercalcemia.

ANS: 3 Feedback 1. High sodium is not an issue in children with cystic fibrosis. 2. Low calcium levels are not an issue for children with cystic fibrosis. 3. The lack of sodium is noted in children with this diagnosis. 4. High calcium levels are not common in children with cystic fibrosis.

The nurse is doing discharge teaching with the parents of a child with new diagnosis of CF. What is the most important concept for parents of CF patients to remember? 1. Hospitalizations can be avoided with consistent chest physiotherapy. 2. There are multiple support groups in the community available to help them cope when the symptoms increase as the child grows older. 3. It affects multiple body systems over a lifetime, which requires vigilant respiratory care and individualized dietary modifications. 4. All symptoms of cystic fibrosis can be managed by diet modifications and increasing the fluids and salt intake of the child.

ANS: 3 Feedback 1. Multiple adaptations to the lifestyle will be needed to maintain a healthy body and avoid hospitalizations. 2. Support groups and summer camps should be implemented right away to learn how to adapt to the illness emotionally. 3. It affects multiple body systems over a lifetime, which requires vigilant respiratory care and individualized dietary modifications. 4. Some individuals will be more ill than others and need different modifications to their diet.

48. The nurse has been caring for a child on the pediatric floor and notes that the childs PMI has shifted to the midline. The nurse is aware that this can indicate: 1. Poor inspiratory effort by the child. 2. Pneumonia in the left lung. 3. Pneumothorax. 4. Neurological integrity has been compromised.

ANS: 3 Feedback 1. Point of maximum impulse is the hearts sounds. 2. Pneumonia may mask the hearts sounds, but should not indicate a shift in the PMI. 3. Air is very heavy and will shift the heart to the midline of the sternum, resulting in a shift of the point of maximum impulse. 4. PMI does not refer to any neurological alterations.

4. A father is concerned that his newborn baby girl is cold because her hands are blue. The nurse explains to the father that: 1. This is a sign of respiratory distress, and the baby needs to return to the nursery. 2. Most newborns have trouble regulating their body temperature. 3. This is acrocyanosis and should go away within 48 hours after her birth. 4. This is bruising the baby received during the birth process.

ANS: 3 Feedback 1. Respiratory distress would be noted if the newborn had circumoral cyanosis. 2. Healthy newborns are able to regulate their body temperature soon after birth if dressed for the environment. 3. The newborn is exhibiting acrocyanosis. It is not a sign of coldness. 4. Bruising usually does not occur on the hands.

37. A mother asks the nurse why a reticulocyte test is performed on her daughters blood. The nurse knows the reticulocyte test will indicate: 1. White blood cell production. 2. Hemoglobin production. 3. Red blood cell production. 4. Hematocrit production.

ANS: 3 Feedback 1. Reticulocyte tests indicate red blood cell production. 2. The reticulocytes are the immature red blood cells that will indicate if enough is going to be produced. 3. The reticulocyte test is the smallest form that measures for red blood cell production. 4. The reticulocyte test is the smallest form that measures for red blood cell production.

51. A mother brought her 8 year old into the emergency room because although she was fine when she woke up this morning, she now has a fever of 39.8 C, cannot speak, is drooling, and is tachypneic and stridorous. Her pulse oximetry reading is 90 percent on a rebreather mask. What would be the next appropriate nursing action? 1. Suction her mouth, then conduct throat and blood cultures as well as a test for gram positive bacteria. 2. Prepare the child and mother for an MRI scan to evaluate for a thumb sign. 3. Monitor respiratory status closely, prepare for intubation, and keep the child calm to avoid crying. 4. Suction her mouth, monitor respiratory status closely, and give a Palivizumab injection.

ANS: 3 Feedback 1. Suctioning can cause more traumas to the area. 2. The thumb sign will not occur in this condition. 3. The nurse would monitor and be prepared for possible rapid decline in respiratory status and try to keep the child from crying. 4. Suctioning the mouth can cause more damage, and the injection should not be given at this time.

43. A child has been admitted with acute Sickle Cell crisis. During the assessment, the nurse would expect to see all of the following except: 1. Swollen joints. 2. Jaundice in the eyes. 3. An enlarged liver. 4. Severe pain in the abdomen.

ANS: 3 Feedback 1. Swollen joints occur because of the ischemia and inflammatory response. 2. Jaundice occurs because of the lack of blood flow to the liver. 3. The liver does not enlarge. 4. Severe abdominal pain occurs because of the lack of oxygen carried to the tissue.

46. The nurse is caring for a 4-year-old child who comes to the pediatricians office with cold symptoms and appears pale. Vital signs are obtained and the childs blood pressure is 68/42 mmHg, a pulse of 98, and respirations of 20. The nurse is aware that: 1. The vital signs are normal in a child this age with cold symptoms. 2. The respiratory rate is elevated and should be communicated to the physician immediately. 3. The systolic blood pressure is too low and should be communicated to the physician. 4. The pulse is too rapid and should be communicated to the physician

ANS: 3 Feedback 1. The blood pressure is too low. 2. The respiratory rate is consistent with a 4 year old with cold symptoms. 3. The systolic blood pressure is 70 mmHg + (two times the age in years) or 78 mmHg. 4. The pulse is normal for a 4-year-old child.

2. When a nurse enters the room of a child with chronic lung disease, she notes that the child is sitting in a tripod position. Identify the reason for this positioning by the child. 1. The child feels more comfortable playing in this position. 2. The child is attempting to have a bowel movement. 3. The child is having trouble breathing, and the position is comfortable 4. The child is in a resting position after walking in the hallway.

ANS: 3 Feedback 1. The child may feel comfortable in this position, but it is not the primary reason for the positioning. 2. A child will squat on their haunches when having a bowel movement. 3. The tripod position enables the diaphragm to fully expand and attempt to get as much oxygen into the body as possible. 4. A child who is resting will sit or lie down on the bed.

23. A child undergoing radiation therapy for cancer has a nursing diagnosis of risk for impaired skin integrity. What is a priority nursing action? 1. Cover the wound with a sterile dressing. 2. Use mild soap on the radiation area. 3. Use water to cleanse the area and leave markings on the skin. 4. Use antiseptic soap to cleanse the area.

ANS: 3 Feedback 1. The diagnosis is a risk, so an actual wound would not be present. 2. Soap may irritate the skin, and the markings are needed for future radiation treatments. 3. Water will not irritate the skin, and the markings are needed for future radiation treatments. 4. Antiseptic soap will irritate the skin, and the markings are needed for future radiation treatments.

45. Parents of a child with hemophilia want to have a second child. Genetic testing is recommended because: 1. The disease is carried by a dominant X-linked gene. 2. The disease is a mutation and has a low occurrence with subsequent pregnancies. 3. The X-linked recessive trait must be present in both parents, and a subsequent child may have the same disorder. 4. The Y-linked recessive trait causes the disease to occur and will happen in subsequent pregnancies.

ANS: 3 Feedback 1. The disease is carried by an X-linked recessive gene. 2. The disease has a high occurrence if both parents are carriers. 3. The X-linked recessive trait must be present in both parents to occur. 4. The X-linked recessive trait is the carrier, not the Y-linked trait.

The nurse knows that Kawasaki disease is: 1. Contagious, especially among children in close settings, such as day-care institutions. 2. Mainly due to a cascading process. 3. Diagnosed as an exclusion process. 4. Most prevalent when the signs and symptoms have been present for less than three days.

ANS: 3 Feedback 1. The disease is not contagious among children, but is seen in certain geographic areas during certain times of the year. 2. This disease is thought to be caused by an infectious organism. 3. Kawasaki disease is diagnosed as a process of exclusion. 4. Symptoms of red lips and a strawberry tongue usually occur following a fever after 5 days.

44. A mother is asking questions about the type of diet her child should be receiving with the diagnosis of aplastic anemia. The nurse informs the mother that the childs diet should include: 1. Hamburgers and french fries. 2. Carrots and potatoes. 3. Turkey sandwiches and spinach salads. 4. Macaroni and hamburger casseroles.

ANS: 3 Feedback 1. The fat content of the french fries should be avoided. 2. These are part of a healthy diet, but do not contain high levels of iron or protein. 3. The protein from the turkey and the iron from the spinach are appropriate for this child. 4. The macaroni has empty nutrients for a child with aplastic anemia.

28. A nurse is assessing a 3-month-old child with RSV. The nurse identifies the following: HR of 140; RR of 32; Oxygen saturation is 89% on room air; inspiratory and expiratory wheezing of the upper lungs; temperature of 38.1 degrees Celsius; large amounts of thin secretions. Identify the priority at this time. 1. Administering acetaminophen to reduce the fever 2. Providing oxygen for the low saturation 3. Suctioning the nares and oropharnyx to remove the secretions 4. Providing a quiet environment

ANS: 3 Feedback 1. The fever is low grade and not a priority at this time. 2. 89 percent oxygen saturation on room air needs to have a further assessment to see why the child is low in saturations. 3. Suctioning helps remove all the secretions and opens the airway with the possibility of increasing oxygen saturations. 4. A quiet environment will help the child rest, but is not a priority at this time.

34. A doctor orders a blood-thinning medication for a 2 year old with known cardiomyopathy. The type of cardiomyopathy that may require blood-thinning medications would be: 1. Hypertrophic cardiomyopathy. 2. Restrictive cardiomyopathy. 3. Dilated cardiomyopathy. 4. Chronic cardiomyopathy.

ANS: 3 Feedback 1. The heart enlarges with hypertrophic cardiomyopathy and has leaky valves, but blood flows through at an adequate rate, decreasing the risk for clotting. 2. Restrictive cardiomyopathy makes the heart rigid, so blood does not have time to pool to create clots. 3. Dilated cardiomyopathy has slow-moving blood because the heart is not effectively pumping blood out of the chambers, increasing clot formation. 4. Chronic is not a type of cardiomyopathy.

4. A nurse is assessing a 4-year-old child with a known atrial septal defect. Identify what the nurse should expect to see in the assessment. 1. An increased heart rate 2. An increased respiratory rate 3. Lower oxygen saturation 4. A lower heart rate

ANS: 3 Feedback 1. The heart rate will be a normal rate for a 4-year-old child. 2. The respiratory rate will be a normal rate for a 4-year-old child. 3. Oxygen saturations are expected to be lower because of the leakage caused by the defect. 4. The heart rate will be a normal rate for a 4-year-old child.

49. A father brings his 6-month-old infant into the clinic with a four day history of nighttime, seal-like cough. The infant is afebrile, tachycardic, and tachypneic with a pulse oximetry reading of 98 percent. What interventions would you expect the physician to order for this child? 1. Cool mist tent with supplemental oxygen, racemic epinephrine, and corticosteroids 2. Beta adrenergics aerosolized, cool mist tent, and periodic testing of blood glucose levels 3. Close monitoring of respiratory status, cool mist tent, beta adrenergics, and corticosteroids 4. Close monitoring of respiratory status, supplemental oxygen with simple mask, and racemic epinephrine

ANS: 3 Feedback 1. The infants pulse oximetry is 98 percent and does not need supplemental oxygen. 2. Beta adrenergic meds do not increase blood glucose levels. 3. These interventions are appropriate for croup-like symptoms. 4. The infants pulse oximetry is 98 percent and does not need supplemental oxygen.

40. A 16 year old who is in the ER after an automobile accident is exhibiting signs of shock. The assessment indicates that the teen has a steering wheel bruise mark on his chest. The teen is exhibiting signs of cardiogenic shock. The nurse working with the trauma team knows the patient: 1. Has had a large loss of blood, which causes cardiogenic shock. 2. Has overwhelming sepsis from the accident, which is causing the cardiogenic shock. 3. The bruising to the chest could have caused damage to the heart, causing it to not be an effective pump, which results in cardiogenic shock. 4. The trauma to the chest has caused capillary leaking, leading to cardiogenic shock.

ANS: 3 Feedback 1. The large loss of blood volume depletes the patient, causing hypovolemic shock. 2. Overwhelming sepsis causes septic shock. 3. The bruising indicates that the chest and heart could have damaged the heart, causing cardiogenic shock. 4. The trauma can cause bruising to the heart muscle, but the capillary leaking will not cause cardiogenic shock.

30. A newborn with transposition of the grater arteries has been prescribed Captopril. The mother asks why the child needs to be on such a medication. The best response by the nurse would be: 1. Your child needs the beta-blocker to decrease the angiotensin in the body. 2. The medication will help decrease the shortness-of-breath episodes. 3. The medication is an antihypertensive that helps relax the coronary arteries. 4. Your child is not responding to the prostaglandin E drip, so the Captopril needs to be started to decrease the blood pressure.

ANS: 3 Feedback 1. The medication is an ACE inhibitor. 2. The medication will not alter the respiratory rate. 3. The medication is an antihypertensive and relaxes the coronary arteries to help decrease blood pressure. 4. A prostaglandin E drip is not a long-term solution.

36. A physicians assistant has ordered Adderall for a 14-year-old boy with ADHD and a history of cardiomyopathy. The nurse should question the order because: 1. The dosing will need to be high because of the cardiac history and the bodys ability to quickly metabolize the medication. 2. Adderall is not effective for children over the age of 12. 3. ADHD medications should be avoided because they can cause sudden death in children with a history of cardiomyopathy. 4. The ADHD medication will not reach a therapeutic level because of the negative interactions with cardiac medications.

ANS: 3 Feedback 1. The medication should not be given with a history of cardiomyopathy. 2. Adderall can be effective in children over the age of 12. The issue is the past history of cardiomyopathy. 3. Sudden cardiac arrest can occur because of the history of cardiomyopathy. 4. Cardiomyopathy and ADHD medications cause an increased risk for sudden death.

25. A mother calls the pediatric triage nurse to report that her son has a barky cough, and it started about midnight. The nurse should instruct the mother to: 1. Take the child to the emergency room right away. 2. Sleep with the child in an upright position. 3. Take the child into a room with a cool mist humidifier or go outside and see if the barky cough subsides. 4. All of the above would be appropriate responses for the mother.

ANS: 3 Feedback 1. The mother should attempt to relieve the symptoms at home prior to coming to the emergency room. 2. The child will more than likely not sleep. 3. A cool mist humidifier or going outside can help reduce the inflammation of the trachea and larynx area. 4. Only using the cool mist humidifier or taking this child into the cool night is effective treatment.

27. A nurse sees a new nurse on the floor handling chemotherapy medications without gloves or protective clothing. What should the nurse do first? 1. Talk about it with other employees at the nurses station later that day. 2. Nothing. The new nurse is doing nothing wrong. 3. Remind the new nurse that it is not safe to handle these medications without protective clothing. 4. Report the new nurse to the supervisor for unsafe use of medications.

ANS: 3 Feedback 1. The nurse needs to speak directly with the new nurse to teach her about the safe handling of chemotherapy agents. 2. The new nurse needs education on how to handle chemotherapy agents. 3. Reminding the new nurse will help facilitate learning on how to handle chemotherapy agents. 4. The issue should be directly discussed with the new nurse.

20. A child with the diagnosis of AML is receiving chemotherapy. The platelet count is 10,000/mm3. The teaching plan for the caregiver should include: 1. Maintaining isolation precautions. 2. Visitors being limited with visiting time. 3. Using a soft toothbrush for mouth care. 4. An assessment of the vital signs every four hours.

ANS: 3 Feedback 1. The platelet count is not in a range to need precautions at this time. 2. The patient may be tired, but visiting times do not need to be limited. 3. The soft toothbrush will minimize bleeding while performing mouth care. 4. Vital signs need to occur per protocol and as frequently as the patients status renders.

38. A mother calls the nursing clinic to report that her 13-year-old daughter has been using tampons for the last two days and now has a high fever. She has developed a rash over her entire body in the last hour. The mother asks if she should make a clinic appointment. The best response by the nurse would be: 1. Your daughter probably has a virus, so provide her with plenty of fluids. They symptoms should subside in a few days. 2. If you feel your daughter should be seen, then I will transfer you to the front desk to make an appointment. 3. Since your daughter is using tampons and has a high fever, she needs to be seen soon. Let me make an appointment for you as soon as possible today. 4. You daughter has toxic shock syndrome and should be seen right away.

ANS: 3 Feedback 1. The symptoms are similar to those of toxic shock syndrome, and the teen should be seen immediately. 2. Passing the patient to someone else for an appointment increases the chance of the parent hanging up the phone. The nurse should take responsibility in this situation. 3. The mother is reporting signs of toxic shock syndrome and should be seen immediately for confirmation of the syndrome. 4. A full assessment needs to be made by the doctor to confirm a medical diagnosis.

13. A nurse knows that the mother understands the discharge instructions for an 8 month old that had a cardiac catheterization for an ASD when the mother states: 1. We will need to schedule weekly visits to make sure the heart is functioning properly. 2. The surgical site will require us to keep our child in isolation at home. 3. We will need to monitor the insertion site for drainage and temperature changes. 4. My child will not have any more issues with arrhythmias

ANS: 3 Feedback 1. The visits will need to be prescribed by the doctor. Visits usually take place three months to one year after the procedure area heals. 2. The child will have a short recovery time and does not need to be in isolation. 3. The insertion site must be monitored for signs and symptoms of infection and bleeding. 4. The child may have arrhythmias his/her entire life.

33. A mother arrives at a birth care center in full labor with no prenatal history. The mother states she has a form of mental illness that requires her to take lithium daily. The nurse knows that lithium use during pregnancy can cause which type of heart defect? 1. Hypoplastic left heart 2. Truncus arteriosus 3. An Epstein anomaly 4. Cardiomyopathy

ANS: 3 Feedback 1. There is no known reason for the development of a hypoplastic left heart. 2. Truncus arteriosus is not linked to use of lithium during pregnancy. 3. Maternal use of lithium has a strong correlation with an Epstein anomaly, which causes the tricuspid valve to be in the right ventricle, along with an enlarged right atrium and cardiomegaly. 4. Cardiomegaly occurs in an Epstein anomaly, but is not the direct defect.

65. An 8 year old is receiving Digoxin (Lanoxin) for congestive heart failure. The nurse provides the caregiver with the following education, indicating that the medication is effective when: 1. The child is happy and active. 2. The child is pink and breathing easily. 3. The childs urine output increases. 4. The child has an improvement in his/her sleeping at night.

ANS: 3 Feedback 1. This is not an indicator of effective drug mechanisms. 2. This is not an indicator of effective drug mechanisms. 3. This indicates improved cardiac output, resulting in improved urine output. 4. This is not an indicator of effective drug mechanisms.

17. Treatment for otitis externa (OE) is usually: 1. No treatment because it resolves on its own. 2. Antibiotic therapy. 3. Corticosteroid therapy. 4. Applying a warm pack to the area for comfort

ANS: 3 Feedback 1. Treatment is recommended because long-term or frequent infections can cause hearing loss. 2. The concern is the fluid and inflammation. Antibiotics will not help remove the fluid and inflammation. 3. Corticosteroids will help reduce the inflammation and fluid in the ear. 4. The warm pack can be a comfort measure, but the fluid and inflammation need to be addressed.

35. The nurse has received hemoglobin and hematocrit levels for a 7-year-old patient. The results are HGB 9.0 and HCT 28 percent. These results indicate: 1. A normal HGB and HCT. 2. A high HGB and low HCT. 3. A low HGB and low HCT. 4. A low HGB and normal HCT.

ANS: 3 Feedback1.The HGB and HCT are low for a child this age. Intervention is needed.2.The HGB is low, and intervention is needed.3.The values are low. The family requires education on how to increase the hemoglobin and hematocrit levels.4.The HCT is low, and an intervention is needed.

3. The nurse is providing care for an infant who is 2 months old. Which assessment finding will cause the nurse to suspect an upper respiratory infection? 1. A raspy cry and occasional cough 2. Adventitious lung sounds bilaterally 3. A stuffy nose and reddened eardrums 4. A fever, lethargy, and skin pallor

ANS: 3 The upper respiratory tract is a passageway that includes the nasopharynx and oropharynx and is connected to the ears by the eustachian tubes. Because of the stuffy nose and reddened eardrums, the nurse suspects an upper respiratory infection. The lungs are part of the lower respiratory system due to the presence of the terminal bronchioles, which end in sacs called alveoli. This finding is indicative of a lower respiratory infection. A raspy cry results from inflammation of the larynx; however, an occasional cough is more indicative of trachea irritation. The manifestations do not necessarily indicate an upper respiratory infection because structures of both the upper and lower respiratory tract are involved. Fever, lethargy, and pallor can be seen in either an upper or lower respiratory infection.

75. In educating an adolescent and his/her caregivers on the modifiable risk factors related to the hypertension, the nurse would include information related to: (Select all that apply.) 1. Age. 2. Race or ethnicity. 3. Hyperlipidemia. 4. Exercise levels. 5. Weight management.

ANS: 3, 4, 5 Feedback 1. Age is not a modifiable risk factor. 2. Race and ethnicity are not modifiable risk factors. 3. Hyperlipidemia through diet education is a modifiable risk factor. 4. Exercise levels are modifiable risk factors that can reduce hypertension. 5. Weight management is a modifiable risk factor that can reduce hypertension.

68. An infant is tachypneic, retracting, and tachycardic with a temp of 39.0 C and a pulse oximetry of 92 percent. You place the infant on 1L nasal cannula oxygen and raise the head of the bed. What intervention would the nurse expect the physician to order next? 1. MRI 2. CT 3. Bronchoscopy 4. Chest x-ray

ANS: 4 Feedback 1. A chest x-ray is the least invasive and can visualize the lung fields. The other answers require sedation, making the infant NPO prior to studies. 2. A chest x-ray is the least invasive and can visualize the lung fields. The other answers require sedation, making the infant NPO prior to studies. 3. A chest x-ray is the least invasive and can visualize the lung fields. The other answers require sedation, making the infant NPO prior to studies. 4. A chest x-ray is the least invasive and can visualize the lung fields. The other answers require sedation, making the infant NPO prior to studies.

35. A 12 year old has been admitted to the pediatric floor for cardiomyopathy. During the acute phase, the nurse should: 1. Have defibrillation equipment present for tachycardic situations. 2. Monitor for crackles in the lungs. 3. Provide string cheese for a snack. 4. All of the above would be appropriate nursing actions during the acute phase.

ANS: 4 Feedback 1. A defibrillator should be present if the child is having tachycardic instances. 2. Crackles in the lungs will indicate if excess fluid is present. 3. String cheese will provide the carnitine needed in the childs diet. 4. A defibrillator should be present if the child is having tachycardic instances. Crackles in the lungs will indicate if excess fluid is present. String cheese will provide the carnitine needed in the childs diet.

47. The nurse is aware that utilizing the proper size cuff is important for an accurate assessment of the blood pressure. For this reason, the nurse chooses a cuff that: 1. Is somewhat smaller than the circumference of the childs left arm. 2. Is somewhat larger than the circumference of the childs right arm. 3. Fits snugly, but not too tight around either arm. 4. The cuff should fit 40 percent of the upper arm between the acromion process and the elbow.

ANS: 4 Feedback 1. A smaller size cuff will result in a higher blood pressure. 2. A larger size cuff will result in a lower blood pressure. 3. This fit will not give an accurate measurement for blood pressure. 4. This is the correct method of measuring the size of a blood pressure cuff.

14. The most common heart defect is: 1. ASD. 2. Patent Foramen Ovale (PFO). 3. Hypertrophic left heart syndrome. 4. Ventricular Septal Defect (VSD)

ANS: 4 Feedback 1. An ASD is not a common heart defect. Usually see closure within a few hours after birth. 2. The PFO is common in premature neonates. Not common in full-term neonates thus is not the most common defect. 3. A rare heart congenital heart defect. 4. The most common defect. The defect can be medically managed with minimal intervention.

21. The child states, I hate losing my hair during chemotherapy. Which of the following interventions is important to the care of the child? 1. A wig similar to the usual hair style can be purchased. 2. Playing with others should include cancer survivors with hair loss. 3. Discuss the benefits of therapy with the child and caregivers. 4. Discuss feelings of the change and concerns with the child.

ANS: 4 Feedback 1. Attempting to cover the head with a wig needs to be discussed with the child before attempting to purchase it. 2. The child should be playing with children that he/she feels most comfortable with. This does not address the concerns of the patient at this time. 3. This does not address the childs concerns and needs to focus on the body image issue at this time. 4. The child needs an opportunity to discuss feelings about the changes in body image.

What is the major contributing factor for the development of BPD? 1. Immature lungs have a decreased number of alveoli for gas exchange 2. Premature birth with decreased number of functional alveoli, leading to lung injury 3. Chronic respiratory infections, leading to pulmonary hypertension and lung scarring 4. Ventilator assistance with high oxygen flow rate at birth, causing inflammation and scarring in lungs

ANS: 4 Feedback 1. BPD occurs because of the increased resistance and amount of damaged alveoli, decreasing the amount of oxygen exchange. 2. Scarring occurs on the alveoli that are present. The preemie baby has the same amount of alveoli, but less surface area to ventilate. 3. Neonates do not commonly have respiratory infections to cause an increased risk for BPD. 4. Ventilator assistance with high oxygen flow rate at birth, causing inflammation and scarring in lungs

18. Important discharge teaching for a 4-year-old boy who had a tympanostomy procedure done would include: 1. The tubes usually fall out spontaneously within a year. 2. Draining of purulent fluid after two days, then return for a follow-up. 3. Placing waterproof ear plugs in the ears when swimming. 4. All of the above should be included in the discharge teaching.

ANS: 4 Feedback 1. Because of the rapid growth of children, the tubes usually last approximately one year. 2. Purulent fluid is a sign of infection. 3. Preventing water from entering the tubes will help decrease the chance of infection. 4. Because of the rapid growth of children, the tubes usually last approximately one year. Purulent fluid is a sign of infection. Preventing water from entering the tubes will help decrease the chance for infection.

23. When assessing a child with coarctation of the aorta, the nurse should perform assessments to all of the follow areas except: 1. Blood pressure in all of the extremities. 2. Monitoring the perfusion to the extremities. 3. Pre-assessment for Digoxin before giving the prescribed doses. 4. Assessing the narrowing pulse pressures.

ANS: 4 Feedback 1. Blood pressure will greatly differ in the upper extremities versus the lower extremities. 2. Perfusion to the lower extremities will be worse than in the upper extremities. 3. Digoxin should always have a pre-assessment of an apical pulse for one minute. 4. Pulse pressures do not give adequate data for a child with coarctation of the aorta.

63. Which statement is most accurate regarding chest physiotherapy (CP)? 1. CP includes postural drainage, chest percussion, vibration, and daily chest x-rays. 2. CP is used to mechanically loosen secretions to prevent or manage atelectasis and gastritis. 3. CP should only be performed in the absence of respiratory distress. 4. CP is contraindicated when chest rib fractures, lung contusions, or hemothorax are present.

ANS: 4 Feedback 1. CP does not require daily X-rays. 2. CP is not used for gastritis. 3. CP should only be done with patients with an increase in respiratory secretions. 4. Chest physiotherapy is contraindicated when rib fractures, lung contusions, or hemothorax are present because further damage can occur.

62. A child with congenital heart disease is more prone to develop which complication? 1. Urinary disturbances 2. Bleeding tendencies 3. Repeated abdominal distention 4. Repeated respiratory infections

ANS: 4 Feedback 1. Congenital heart disease does not cause urinary disturbances. 2. Congenital heart disease does not result in bleeding tendencies. 3. Congenital heart disease does not result in repeated abdominal distention. 4. Congenital heart disease does predispose the child to repeated respiratory infections due to pulmonary congestion.

42. The nurse assessing a child with known right-sided heart failure will expect to find which of the following symptoms? 1. Crackles in the lungs 2. Increased edema in the face 3. A galloping rhythm 4. All of the above are symptoms of right-sided heart failure.

ANS: 4 Feedback 1. Crackles may be present because of the pooling of blood in the lungs. 2. The edema occurs because of the lack of venous return. 3. A galloping rhythm will be present. 4. Crackles may be present because of the pooling of blood in the lungs. The edema occurs because of the lack of venous return. A galloping rhythm will be present.

25. Identify a common characteristic of pulmonary atresia. 1. Acrocyanosis at birth 2. Weight gain is similar to that of well newborns. 3. A murmur will be noted with an ASD or a PDA. 4. Severe cyanosis will be present at birth.

ANS: 4 Feedback 1. Cyanosis will be noted at birth. 2. Weight gain will be slower than peers. 3. A murmur will not be present with the ASD and the PDA. 4. Cyanosis will be noted at birth because of the fistula not allowing blood to go to the lungs in order to oxygenate.

26. When assessing a child with epiglotitus, the nurse should assess for all of the following except: 1. Drooling. 2. Dysphonia. 3. Stridor. 4. Crackles in the upper lungs.

ANS: 4 Feedback 1. Drooling can indicate swelling of the epiglottitis because the secretions are not able to go to the stomach. 2. Dysphonia can occur because of the swelling. 3. Stridor is common because of the swelling of the epiglottitis. 4. Crackles are heard in lower respiratory illnesses, not the upper respiratory illnesses in children.

64. A 6 year old who exhibits a moist, productive cough has a history of bronchitis several times every year and eating everything in sight. She appears thin for her age and has a sweat chloride test that is 67 mEQ/L. Her mother states, I just want to get this eating disorder treated so my baby can have a normal life. What is the nurses best response? 1. We will consult the dietician for a behavior management and eating plan, focusing on appropriate portion size. 2. We will need to do another sweat chloride test next week. Have your child take supplemental water-soluble vitamins, such as A, D, K and iron. 3. You should incorporate tofu and mayonnaise in your meal preparation to promote feeling full for a longer period of time. 4. Cystic fibrosis can cause an increase in appetite because of the lack of nutrients and calories absorbed. This affects children across the life span.

ANS: 4 Feedback 1. Food choices that contain the needed vitamins and minerals should be discussed. 2. The child already has the diagnosis and another test will not indicate which vitamins to give. 3. This diet will not easily be digested by a person with CF. The menu should be reconsidered. 4. Increased appetite is a physiologic response to decreased fat-soluble nutrients and calories absorbed in the CF digestive track. This requires fat-soluble (A,D, E, K) vitamins and pancreatic enzyme supplements.

13. Jordan is 10 years old and has hemophilia. The discharge instructions should include teaching about participation in which of the following activities? 1. Football 2. Soccer 3. Baseball 4. Swimming

ANS: 4 Feedback 1. Football has an increased risk for internal bleeding and should be avoided. 2. Soccer places the child at an increased risk for internal bleeding and should be avoided. 3. Baseball puts the child at increased risk for injury and should be avoided. 4. To lower the risk for internal bleeding, the child can participate in noncontact sports, such as swimming.

56. The nurse knows that one of the most likely symptoms of congestive heart failure (CHF) in infants is: 1. Jugular vein distention (JVD). 2. Decreased blood pressure. 3. Periorbital edema. 4. Diaphoresis with feedings

ANS: 4 Feedback 1. JVD is not a symptom of CHF in infants. 2. Increased, not decreased, blood pressure is an indicator of CHF. 3. Although periorbital edema occurs in infants with prolonged CHF, it is not a likely symptom of CHF. 4. This is the most common symptom due to sympathetic stimulation.

38. The nurse is providing education about giving iron supplements to a 2-year-old child. The nurse should include none of the following except: 1. Giving the supplements one hour prior to letting the child drink milk. 2. Noting that diarrhea may be present at first. 3. Giving the supplements with vitamin D to increase absorption. 4. Stools may be black

ANS: 4 Feedback 1. Milk does not influence the absorption of iron. 2. Constipation is more common than diarrhea when taking iron supplements. 3. Vitamin D will not increase absorption. 4. Iron supplementation can cause black stools in toddlers.

61. A newborn has a scaphoid-shaped abdomen, irregular chest wall movements, and decreased breath sounds on the left side of chest. What other symptoms would you expect to find? 1. Central cyanosis and pink nailbeds with brisk capillary refill 2. Protruding abdomen and fullness with palpation 3. Increased breath sounds over trachea, tachypnea, and stidor 4. Tachypnea, nasal flaring, and retractions

ANS: 4 Feedback 1. Nailbeds will be cyanotic and exhibit slow capillary refill. 2. The abdomen will be full and stiff because of excessive air. 3. Grunting may be present, and there will be decreased breath sounds. 4. Tachypnea, nasal flaring, and retractions are the correct symptoms.

33. When assessing a newborn with a known diaphragmatic hernia, the nurse would anticipate hearing bowel sounds: 1. In the upper abdomen. 2. In the lower abdomen. 3. To not exist. 4. In the chest.

ANS: 4 Feedback 1. Normal bowel sounds can be heard in the upper abdomen. 2. Normal bowel sounds can be heard in the lower abdomen. 3. Bowel sounds do exist, just in a different area of the body. 4. Because of the lack of diaphragm, the gastrointestinal tract is shifted into the chest cavity.

24. A newborn has had a repair of a trancheoesophageal fistula one hour ago. When the newborn is taken to the neonatal intensive care unit, the nurse should: 1. Monitor the oxygen saturations of the newborn. 2. Assess for respiratory distress. 3. Provide oral suctioning as needed. 4. All of the above should be done for the newborn.

ANS: 4 Feedback 1. Oxygen saturations will indicate the respiratory status of the newborn. 2. Assessment for respiratory distress is needed because the surgery requires some trauma to the trachea. 3. Suctioning is needed so the secretions do not cause blockage in the airway. 4. Oxygen saturations will indicate the respiratory status of the newborn. Assessment for respiratory distress is needed because the surgery requires some trauma to the trachea. Suctioning is needed so the secretions do not cause blockage in the airway.

27. The nurse is assessing a baby with a known diagnosis of Tetralogy of Fallot with pulmonary atresia. The nurse should expect which of the following in her assessment of the baby? 1. A VSD murmur 2. Normal growth and development 3. Decreased peripheral pulses 4. Profound cyanosis

ANS: 4 Feedback 1. PDA murmur is common, not a VSD. 2. Growth and development will be delayed. 3. Peripheral pulses will be bounding. 4. Cyanosis will be present due to where the holes in the heart are located.

29. A 4-year-old boy comes into the emergency department with multiple bruises on his body, excessive nausea, headaches that lead to vomiting, persistent localized pain, and is very pale. Identify the one symptom that is not a cardinal sign of cancer. 1. Paleness 2. Bruises 3. Headaches 4. Excessive nausea

ANS: 4 Feedback 1. Pale skin tone is common. 2. Bruises appear when no injury has occurred in children with cancer. 3. Excessive headaches can indicate tumor growth in children with cancer. 4. Excessive nausea is not common in children with cancer.

49. A mother has reported that her 18 month old has a lump on the abdominal area. She has noticed the lump while changing her daughters diaper. A nurse should assess by using which measure(s)? 1. An abdominal circumference 2. Palpation and an abdominal circumference 3. Assessing the childs pain when the area is palpated 4. Assessing pain, palpation, and an abdominal circumference

ANS: 4 Feedback 1. Palpation and pain assessment should also be conducted for a thorough exam. 2. A pain assessment is needed for the child. 3. An abdominal circumference should be performed. 4. Assessing pain, palpating, and measuring the abdominal circumference will aid in the nursing process for the child.

6. A child with respiratory distress can experience dehydration because: 1. The child is not drinking enough fluids. 2. The body requires an increased amount of fluids when sick. 3. The child is retaining water in the kidneys since the body is using all the oxygen in the lungs. 4. Mouth breathing occurs when in distress, so the child is losing hydration.

ANS: 4 Feedback 1. Respiratory distress causes dehydration issues. 2. Fluids are required to keep mucous membranes and secretions moist, but are not the reason for dehydration. 3. Water is not retained in the kidneys with respiratory difficulties. 4. Children are known to be mouth breathers during respiratory distress situations, thus increasing their risk for dehydration due to the lack of moist mucous membranes

67. If a nurse suspects that a 2-month-old infants death was related to SIDS, what statement made by the mother reflects an accurate understanding of SIDS? 1. I knew that I should not have given our baby the antibiotics for the ear infection. 2. Being a twin with low birth weight, he didnt have a chance. 3. I should not have fed him that eight-ounce bottle before laying him down. 4. I am having a hard time not knowing what happened. I had just checked on him 20 minutes earlier in the crib, and he was sleeping on his back.

ANS: 4 Feedback 1. SIDS is a diagnosis of exclusion. Antibiotics are not known to cause SIDS. 2. A lower birth weight child is at more risk, but is not the only reason SIDS can occur. 3. The amount of feeding does not influence the occurrence of SIDS. 4. SIDS is a diagnosis of exclusion. It is difficult to know what exactly causes the death in SIDS cases.

11. A 6 month old has a known diagnosis of an Atrial Septal Defect (ASD). The nurse would anticipate all except which of the following during an assessment? 1. Shortness of breath 2. Enlarged liver 3. Poor feeding 4. A diastolic murmur

ANS: 4 Feedback 1. Shortness of breath is expected since more blood flows to the pulmonary area because of the hole. 2. The liver enlarges because of the increase in blood flow. 3. Poor feedings are expected as a result of shortness of breath because of the pulmonary hypertension issues. 4. A systolic murmur is expected due to the blood being forced through the pulmonary valve.

21. A child is scheduled to have a tonsillectomy in two hours. The nurses assessment should include: 1. A question to see if the child snores or has difficulty breathing at times. 2. Assessing for halitosis. 3. The size of the tonsils. 4. All of the above

ANS: 4 Feedback 1. Snoring and difficulty breathing are an indication of obstruction of the tonsils. 2. Halitosis is common in children with enlarged tonsils because of the bacterial content. 3. Tonsil size should be documented because removal of the entire tissue will need to occur during surgery. 4. Snoring and difficulty breathing are an indication of obstruction of the tonsils. Halitosis is common in children with enlarged tonsils because of the bacterial content. Tonsil size should be documented because removal of the entire tissue will need to occur during surgery.

12. A 12 year old is admitted with a Sickle Cell crisis. Which of the following is the priority nursing diagnosis? 1. High risk for dehydration 2. Impaired airway 3. Inappropriate grieving 4. Pain related to tissue ischemia

ANS: 4 Feedback 1. The admitting diagnosis does not indicate if the child is dehydrated, thus this is not the proper diagnosis at this time. 2. The airway is not usually compromised in a child with a Sickle Cell crisis. 3. The Sickle Cell crisis may cause the child to grieve, but this is not the priority at this time. 4. The pain from the decreased oxygen carrying capacity of the cells causes distress. Comfort measures, including medication and warm, moist heat, must be considered.

37. A nurse is preparing to administer Digoxin to a 4 year old. The nurse should: 1. Administer the medication and check the blood pressure one hour later. 2. Give the medication with food. 3. Take the apical pulse for 30 seconds prior to giving the medication. 4. Note the rate, rhythm, and quality of the heart prior to giving the medication.

ANS: 4 Feedback 1. The baseline blood pressure is needed prior to the administration of the medication. 2. The medication should be taken on an empty stomach. 3. The apical pulse should be taken for 60 seconds. 4. A baseline of the rate, rhythm, and quality is needed.

64. A 7-year-old child is discharged following a cardiac catheterization yesterday. The nurse should instruct the mother to: 1. Allow the child to take a tub bath today. 2. Allow the child to resume normal physical activities, including sports. 3. Limit diet within the first few days to prevent straining to stool. 4. Observe for signs and symptoms of infection for the first few days.

ANS: 4 Feedback 1. The child can take showers, not baths, for the first several days. 2. The child should not lift anything heavy and should not resume physical activity for two weeks. 3. There is no limit on the diet of the child. 4. The child should be monitored for signs and symptoms of infection.

22. The laboratory results of a child with AML indicate a white blood cell count of 500 with two percent bands. Which of the following responses is appropriate? 1. Administer the Hepatitis B vaccine as ordered. 2. Prepare for hemolytic reactions. 3. Visitors can bring flowers and gifts of fruit to the child. 4. Have people wash their hands prior to contact with the child.

ANS: 4 Feedback 1. The child is at high risk for infection and vaccinations are not recommended. 2. The child is at risk for infection. 3. Flowers and fruit can bring in bacteria and should be avoided. 4. The child is at high risk of infection because he/she does not have an adequate white blood cell count of greater than 500, and the bands are immature white cells.

24. A 7-year-old child has been hospitalized for treatment for leukemia. Which nursing action is most appropriate for the childs nutrition? 1. Offer only foods that the child likes. 2. Turn on the television for distraction while eating. 3. Offer juice or popsicles every two hours. 4. Have caregivers visit at mealtime.

ANS: 4 Feedback 1. The child may not want the foods that are the most nutritious, thus requiring the offer of other foods. 2. The television should be off in order to encourage concentration on eating. 3. Juice and popsicles provide empty calories. A concentration on nutritious foods should be made. 4. The caregivers can offer emotional, social, and psychological support to enhance nutritional intake.

5. When caring for a child with Wilms tumor, which of the following nursing interventions would be most important? 1. Place child on neutropenic precautions. 2. Monitor bowel sounds in order to detect ileus. 3. Position in the high fowlers position in order to increase lung capacity. 4. Avoid palpation of the abdomen.

ANS: 4 Feedback 1. The child will need neutropenic precautions if radiation or chemotherapy are provided. 2. A Wilms tumor should not cause an ileus. 3. The lungs are not affected in a child with a Wilms tumor, so it is not the most important intervention at this time. 4. The tumor is located in the abdomen and may be harmful to the child with palpation of the abdomen.

58. Which statement regarding the pathophysiology of TB is accurate? 1. The settling of the bacillus in the alveoli triggers the clotting response. 2. Macrophages form hard tubercules around bacilli that always remain dormant in the lungs. 3. TB can affect the lungs, spinal cord, bone formation and the brain. 4. Tubercles in the lungs can remain dormant or progress to active tuberculosis, but are not as prevalent in children.

ANS: 4 Feedback 1. The clotting response is not triggered by the bacillus. 2. The tubercules are rare in children. 3. TB affects the lungs only. 4. Tubercles in the lungs can remain dormant or progress to active tuberculosis, but are not as prevalent in children.

40. A nurse is attempting to educate a 10-year-old girl in the use of a peak flow meter. Identify the best way to explain the test to the child. 1. The purpose of the test is to see how hard you breathe. 2. The purpose of the test is for you to monitor what is normal and abnormal for you. Then your parents can help with your medication on days when you are not measuring in your normal ranges. 3: We are measuring how well you can blow birthday candles out. 4. The meter will help monitor when you are healthy and when you are becoming ill.

ANS: 4 Feedback 1. The description is not accurate, and a 10 year old is able to comprehend the reason for use of a peak flow meter. 2. The description of normal and abnormal can cause concern for the child. It is important to explain that the peak flow meter is a measurement of health. 3. This description can be used for a younger child. A 10 year old is able to comprehend the use of the peak flow meter. 4. The peak flow meter is a monitor used to indicate when the child is breathing easily and when illness may be starting.

9. When assessing a 12-year-old girl with the diagnosis of pneumonia, the nurse performs percussion. The lower left lobe is noted to have a dull sound. What should the nurse do next? 1. Call the doctor with the assessment. 2. Check the orders and start chest physiotherapy. 3. Palpate the chest to check for tactile fremitus. 4. Place the child on oxygen

ANS: 4 Feedback 1. The doctor will need to be called after oxygen is applied because the first priority is to maintain oxygen saturation in order to prevent further respiratory distress. 2. The child needs immediate intervention. 3. Tactile fremitus will be increased due to the pneumonia. 4. The assessment indicates that the child has a lower lobe that is not expanding and needs oxygen supplementation in order to maintain saturation levels.

29. When a child with transposition of the greater vessels is assessed, the nurse should anticipate that: 1. The lower extremities will have bounding pulses. 2. Cyanosis will be noted when the child is sleeping on his/her back. 3. An ASD murmur will be present. 4. The oxygen saturations in the upper extremities will be lower than the oxygen saturations in the lower extremities.

ANS: 4 Feedback 1. The extremities will have weak pulses and low oxygen saturations. 2. Cyanosis will be present when crying. 3. The child will have a PDA. 4. The difference in oxygenation is caused by the aorta receiving deoxygenated blood.

39. A nurse is discussing discharge instructions with parents of a child who received a cardiac transplant five weeks ago. The discharge instructions should include all of the following except: 1. Discussion of signs and symptoms of rejection. 2. The child should not participate in moderate to high physical activity. 3. Instructions to wear a medical alert bracelet. 4. Take the anti-rejection medications when signs of rejection arise.

ANS: 4 Feedback 1. The family should be informed about possible signs and symptoms for early detection. 2. Increased activity causes an increased workload on the heart, so this should be avoided until the doctor gives permission for increasing activity. 3. A medical alert bracelet is recommended so others are aware of needs of the patient. 4. The anti-rejection medications will need to be taken for the remainder of the childs life every day.

45. The nurse caring for a child with a suspected congenital heart defect performs the hyper-oxygenation test appropriately when he/she applies the pulse oximeter to the childs: 1. Left arm. 2. Right leg. 3. Left leg. 4. Right arm.

ANS: 4 Feedback 1. The left arm is postductal and not an accurate reflection of the true oxygen saturation. 2. The right leg, although preductal, is not the best choice for obtaining an accurate oxygen concentration that is close to the heart. 3. The left leg is postductal and not an accurate reflection of the true oxygen saturation. 4. The right arm is preductal and the most accurate assessment of the oxygen saturation prior to most congenital heart defects.

59. The nurse is doing discharge teaching with the mother of a 10 year old, who has been newly diagnosed with TB. Which statement is not accurate regarding the spread of TB? 1. The patient should take anti-tubercular medicine for two weeks before being exposed to any non-infected people. 2. Everyone should wash their hands or use sanitizer after exposure to respiratory secretions. 3. It is transmitted through inhaled droplets from a close contact that is infected. 4. About 460,000 new cases of multi-drug sensitive TB are reported every year because of incomplete treatment regimes.

ANS: 4 Feedback 1. The medication will be needed for this length of time before being exposed to others. 2. Washing of hands should occur with every patient. 3. Close contact with those who have the disease increases the risk. 4. This statement is not accurate.

50. An 8 month old was admitted to the hospital last night with cold symptoms and respiratory distress. She is on a simple mask with a flow rate of 10 L and on a cardiorespiratory monitor. The nurse goes into the infants room to find her tachypneic, retracting, and slightly cyanotic with a pulse oximetry of 90%. What would be the oxygen delivery system that may help the infant? 1. A venturi mask with an oxygen flow of 1 liter per minute. 2. A nasal cannula with an oxygen flow of 4 liters per minute. 3. An oxygen tent with an oxygen flow rate of 10 liters per minute. 4. A partial rebreather mask with an oxygen flow rate of 8 liters per minute.

ANS: 4 Feedback 1. The pressure is not adequate to oxygenate the infant. 2. A nasal cannula does not deliver enough pure oxygen to raise the oxygen saturation of the infant. 3. The oxygen tent will not allow for enough pressure for the infant to raise the oxygen saturation. 4. A partial rebreather mask with an oxygen flow rate of 8 liters per minute will raise the oxygen saturation of the infant.

17. Jaycob, a 24-month-old child with a diagnosis of RSV and Tetrology of Fallot, is being cared for by a new nurse. Jaycob is agitated and is crying when care is provided. He begins to drop his oxygen saturations below an acceptable range. The nurse should: 1. Have the parent console the child. 2. Feed the child. 3. Call the doctor for an order for a sedative. 4. Cluster the care and allow the child time to rest.

ANS: 4 Feedback 1. The question does not state that the parent is available. 2. Feeding the child may cause oxygen saturations to drop lower. 3. All attempts at consoling the child should first be provided before asking for a sedative. 4. Clustering cares will allow for time to rest and result in less stress for the child.

57. You suspect a 14 year old with persistent cough, anorexia, low-grade fever, and night sweats has tuberculosis. What is the most accurate statement about the treatment of this patient? 1. A nurse needs to collect serial sputum cultures in the a.m. and do serial AFB tests. 2. Latent TB would be treated with antituberculin medication combinations in higher doses for nine months. 3. Anti-tubercular medications given in higher doses in combination for six months are only effective after BCG vaccine is given. 4. Active TB is treated with combinations of rifampicin, isoniazid, ethambutol, and streptomycin in higher doses for six months.

ANS: 4 Feedback 1. The time of day does not influence when the sample should be taken. 2. Active TB is treated with combinations of rifampicin, isoniazid, ethambutol, and streptomycin in higher doses for six months. 3. Active TB is treated with combinations of rifampicin, isoniazid, ethambutol, and streptomycin in higher doses for six months. 4. Active TB is treated with combinations of rifampicin, isoniazid, ethambutol, and streptomycin in higher doses for six months.

12. A nurse should question which of the following orders for a child with a known ASD? 1. A transesophogeal ultrasound 2. Digoxin 3. EKG 4. All are appropriate orders for a child with an ASD.

ANS: 4 Feedback 1. The ultrasound allows for the entire heart to be viewed in order to find the exact location of the ASD. 2. Digoxin will help with the cardiac output. 3. An EKG will help identify heart function. 4. All the orders would be appropriate for the child because each aids in gathering all the information needed for proper treatment of the defect

19. An outbreak of influenza has occurred at the middle school. The school nurse is preparing to send home information about influenza. Her flyer should include all of the following except: 1. The virus is contagious one to two days prior to the appearance of symptoms. 2. Do not send your child to school if he/she has the chills or a erythematous rash. 3. Hydration is important. 4. If your child vomits, take them to the emergency room immediately

ANS: 4 Feedback 1. The virus is most contagious one to two days prior to the appearance of symptoms. 2. Chills and a erythematous rash indicate fever and can cause the spread of the virus. 3. Hydration will help keep mucous membranes moist to remove secretions. 4. Vomiting may occur and is not a medical emergency.

49. The prevalent incidence of rheumatic heart disease occurs in: 1. Late spring and winter. 2. Summer and early spring. 3. Early spring and fall. 4. Early spring and winter.

ANS: 4 Feedback 1. There is a two-fold increase in the early spring, not late spring, and winter for the development of rheumatic fever. 2. The increase is not in the summer. 3. The increase is not in the fall. 4. There is a two-fold increase in the early spring and winter for the development of rheumatic fever.

54. Children who have defects with decreased pulmonary blood flow exhibit which of the following common symptoms? 1. Nausea and vomiting 2. Weight gain 3. Tachypnea, bradycardia, and diaphoresis 4. Cyanosis, tachypnea, and polycythemia

ANS: 4 Feedback 1. This is not a symptom of decreased pulmonary blood flow. 2. This is more a symptom of congestive heart failure. 3. Tachypnea and diaphoresis are correct, but bradycardia, or slow heart rate, is not a symptom of decreased pulmonary blood flow. 4. Cyanosis and tachypnea are symptoms of decreased pulmonary blood flow, which decreases the bodys ability to oxygenate the blood. Polycythemia occurs due to a chronic decrease in oxygenation. The body attempts to produce more red blood cells to carry additional oxygen.

34. A nurse is repositioning an infant with a known diaphragmatic hernia. The nurse should place the infant in which position? 1. With the head of bed elevated 20 degrees 2. Supine 3. Prone 4. In a semi-fowlers position

ANS: 4 Feedback 1. This position does not take enough pressure off of the respiratory muscles. 2. Supine can cause the collapsing of the chest cavity and increase difficulty breathing. 3. Prone can cause too much pressure on the respiratory muscles and not allow for expansion. 4. Semi-fowlers will allow for pressure to be taken off of the diaphragm and decrease difficulty breathing.

38. Teaching a child with a chronic respiratory illness to forcefully exhale can be done by: 1. Pretending to blow candles out. 2. Blowing bubbles. 3. Pretending to blow out a flashlight. 4. All of the above are techniques for teaching a child to forcefully exhale.

ANS: 4 Feedback 1. This requires a large volume for inhalation and expiration, thus being an effective treatment. 2. This requires pursed-lip breathing and helps force air, thus being an effective treatment. 3. This requires a large volume for inhalation and expiration, thus being effective treatment. 4. Pretending to blow out candles or a flashlight require a large volume for inhalation and expiration, thus being effective treatment. Blowing bubbles requires pursed-lip breathing and helps force air, thus being an effective treatment.

53. In caring for a patient with Tetralogy of Fallot, the nurse understands that during a tet spell, the following nursing intervention will reduce the symptoms of this disorder. 1. Increase the oxygen level of the child 2. Set the infant in an upright position 3. Bounce or pat the child until they are consoled. 4. Place infant in a knee-chest position

ANS: 4 Feedback 1. While this may improve oxygenation, it will not alleviate the symptoms of the disorder. 2. This will have no effect on the clamping down of the vasculature. 3. Although there is an emotional component to the tet spell, this will not alleviate the symptoms of the disorder. 4. Placing the child in a knee-chest position will aid in returning the blood flow back to the heart and improving the cyanosis. Flexing the legs decreases venous flow from the lower extremities and decreases shunting through the ventricular septal defect. It increases vascular resistance and increases pressure in the left ventricle.

75. A nurse is discussing the process in which tuberculosis can infect a child. Place the following in the correct order. __ Sputum specimen is obtained __ Tubercles are dormant __ Bacillus triggers the immune response __ Bacilli spread to the lymphatic system __ Macrophages form tubercles around bacilli

ANS: 5, 4, 1, 3, 2

The nurse is caring for a child diagnosed with Kawasaki's disease. Which interventions should the nurse anticipate as part of the treatment protocol? Select all that apply. Administration of IV gamma globulin Aspirin administration, high dose for weight Emergent echocardiogram Administer varicella (chickenpox) vaccine prior to discharge Help parents cope with the irritability of the child

Administration of IV gamma globulin Rationale: IV gamma globulin is given for the first 10 days after the onset of symptoms to prevent coronary aneurysm Aspirin administration, high dose for weight Rationale: Anti-inflammatory medications, such as high-dose salicylate therapy, will be maintained. Help parents cope with the irritability of the child.

what position to place a child after a tonsillectomy?

After tonsillectomy, place in side-lying or prone position to drain secretion until child awakens Avoid coughing, using a straw, or blowing nose to prevent the risk for bleeding

Which congenital heart defect results in pulmonary hypertension and right atrial enlargement from an increase in pulmonary blood? Atrial septal defect Tetralogy of Fallot Tricuspid atresia Aortic stenosis

Atrial septal defect Rationale: ASD results in increased pulmonary blood flow through a hole in the atria. This may result in pulmonary hypertension with right atrial enlargement that can cause right ventricular hypertrophy.

A client has just undergone cardiac catheterization. The nurse is preparing discharge instructions. What should the nurse include in the instructions? Select all that apply. Avoid strenuous activity such as lifting, sports, or physical education, although school is appropriate. A regular diet can be consumed. Follow-up appointments are essential. Fever is common following catheterization and should last longer than 24 hours and go above 100°F (37.7°C). Keep lower extremities slightly flexed for 4 to 6 hours following the catheterization.

Avoid strenuous activity such as lifting, sports, or physical education, although school is appropriate. A regular diet can be consumed. Follow-up appointments are essential.

The nurse is providing discharge teaching to parents of a child receiving digoxin (Lanoxin). Which items should the nurse include as signs of digoxin toxicity? Select all that apply. Bradycardia Nausea Diarrhea Increase in appetite Flushing of the skin

Bradycardia Nausea Diarrhea

The nurse is developing a teaching plan about medication management for a client who is taking a loop diuretic. Which drug should be included in the plan? Bumetanide (Bumex) Hydrochlorothiazide (Diuril) Amoxicillin (Amoxil) Spironolactone (Aldactone)

Bumetanide (Bumex) rationale: Bumetanide is a loop diuretic

Complications of tetralogy of Fallot?

Cerebral thrombosis is caused by polycythemia, especially if dehydration occurs. -Iron deficiency anemia -Bacterial endocarditis can occur

What position can help a child breathe easier in TOF condition?

Children may squat during a tet spell to improve blood flow from the legs back to the brain and vital organs, increasing systemic vascular resistance. **This help by increasing systemic pressure to help push blood through the pulmonary valve stenosis

when to closely monitor children who are at risk for hyperlipidemia?

Children with 2 consecutive blood cholesterol levels exceeding 170 mg/dL should be monitored.

The nurse is measuring blood pressure on the leg of a sleeping toddler and receives an unusually high reading. What is the best action for the nurse? Confirm the appropriate size of the cuff and obtain a larger cuff if needed to recheck pressures. Confirm the appropriate size of the cuff and obtain a smaller cuff if needed to recheck pressures. Allow the child to rest and recheck with the same cuff in approximately 10 minutes. Document the finding and notify the practitioner.

Confirm the appropriate size of the cuff and obtain a larger cuff if needed to recheck pressures. rationale: undersized BP cuffs will give the nurse a falsely HIGH reading.

What is the longer-acting corticosteroid that is given to treat variable coup syndromes?

Corticosteroids (dexamethasone): longer acting, 36- to 54-hour half-life • May lead to hypertension and elevated glucose levels

what are the side effects of corticosteroids, oral (dexamethasone) in treating RSV and laryngotracheobronchitis?

Corticosteroids, oral (dexamethasone): May lead to hypertension and elevated glucose levels

Croup vs Epiglottitis

Croup: VIRAL, fever, HUMIDITY for treatment Epiglottitis: bacterial; antibiotics needed.

While auscultating a client's chest, the nurse observes a shift in the point of maximal impulse (PMI) and magnified heart sounds (hyperactive precordium). The nurse understands this may indicate what? Normal finding Dextrocardia Acrocyanosis Bradycardia

Dextrocardia Rationale: hyperactive precordium (during auscultation, the heart sounds are magnified) or a shift in the PMI indicates consideration of dextrocardia or pneumothorax.

Why we shouldn't give cough suppression Dextromethorphan to children under 6 years?

Dextromethorphan has alcohol in it

A nurse is teaching a caregiver of a child with Wilms' tumor. Which statement regarding care of the child is true? Select all that apply. Side effects of cancer treatment are usually not evident after 6 months. Do not palpate the tumor. Avoid pushing or lifting in tumor area. Genetic counseling may be ordered. Chemotherapy and radiation typically eradicate the tumor.

Do not palpate the tumor. Avoid pushing or lifting in tumor area. Genetic counseling may be ordered.

what are the safety alert for CHF?

Early signs of CHF in infants: Tachycardia at rest Fatigue during feedings sweating around scalp & forehead sudden weight gain

A nurse is assessing a child with a hematologic disorder. Children with a hematologic disorder may present with: (Select all that apply.) Edema Bradycardia Delayed learning Brittle and concave nails Increased serum concentration of the proteins albumin, gamma globulin, and transferrin

Edema Delayed learning Brittle and concave nails

The nurse is teaching a caregiver of a child with human immunodeficiency virus (HIV). Which statement regarding caring for a child with HIV are true? Select all that apply. Educate day-care and school staff on current HIV information. Encourage the use of pharmacologic pain interventions. Dependence on opioids may require decreased dosing over time. Emphasize time of administration of highly active antiviral therapy (HAART). Foster positive self-concept.

Educate day-care and school staff on current HIV information. Emphasize time of administration of highly active antiviral therapy (HAART). Foster positive self-concept.

A nurse is caring for a neonate with a suspected tracheoesophageal fistula (TEF). What nursing intervention would appropriate for this client? Elevate the head for feedings. Avoid suctioning the client. Elevate the head, but give nothing by mouth. Feed the baby glucose water only.

Elevate the head for feedings. Explain: elevate the head up to 30°, especially while feeding NG TUBE

A nurse is planning administration of fresh frozen plasma (FFP). Which statement is true? It is a replacement for deficiency of hemoglobin and hematocrit. FFP product must be used within eight hours of thawing Fibrinogen levels must be below 150 mg/dL. Warfarin anticoagulant administration indicates FFP.

FFP product must be used within eight hours of thawing

59. A child with a known diagnosis of HIV should not receive immunizations against common childhood illnesses. True or False

False The child should receive immunization on schedule to prevent further complications of the illness.

When assessing pulses of a child with coarctation of the aorta, the nurse should use the right subclavian artery. True or False

False The right brachial artery should be used because the subclavian can give a false reading.

The nurse is charting a care plan for a client who has undergone a heart transplant. Which common signs and symptoms of heart transplant rejection should be listed on the plan of care? Select all that apply. Fever Oliguria Weight gain Bradycardia Tachycardia

Fever Oliguria Weight gain Tachycardia

When can you begin giving flu vaccine to infant?

Flu vaccine can be first seasonal vaccine for children 6 months to 8 years of age • Not protected until 2 weeks after second vaccine

A nurse is planning care for a human immunodeficiency virus (HIV)-positive child. Which interventions will the nurse include? Select all that apply. Follow standard precautions when providing care. Administer highly active antiretroviral therapy drugs (HAART) combination therapy. Strictly adhere to a low sodium, low calorie diet. Immunization is not recommended against common childhood illnesses. Prevent and manage opportunistic infections.

Follow standard precautions when providing care. Administer highly active antiretroviral therapy drugs (HAART) combination therapy. Prevent and manage opportunistic infections.

A nurse is planning care for a child with suspected lead poisoning. Which diagnostic tests are utilized? Select all that apply. Free erythrocyte protoporphyrin (FEP) Liver function test Urinalysis Bone radiography Blood glucose

Free erythrocyte protoporphyrin (FEP) Urinalysis Bone radiography

The nurse is caring for an adolescent post cardiac transplant and notes episodes of tachycardia, lightheadedness, and irritability. Which additional assessment would support the possibility of rejection? Temperature of 98.4°F (36.8°C) orally Intake of 90% of meals for the last 24 hours Gain of 4 pounds in the last 2 days Respiratory rate varying between 20 and 24

Gain of 4 pounds in the last 2 days

The nurse working in the pediatric cardiac care unit (CCU) notices that a client who is 8 years old has a rapid rhythm on the electrocardiogram (EKG) tracing. The nurse recognizes this dysrhythmia as supraventricular tachycardia (SVT) when the heart rate reaches what? Greater than 220 beats/minute Less than 200 beats/minute 180 to 200 beats/minute 70 to 90 beats/minute

Greater than 220 beats/minute

what vaccine can help prevent epiglottis?

H. influenzae type B, S. pneumoniae vaccines there is decreased incidence of epiglottis

What are some s/s of COA?

HEADACHE leg & stomach cramping Cool extremities decrease LE pulses

What are some medications treatments for Sickle-cell? list 3

Hydroxyurea Rationale: It increases fetal hemoglobin, which prevents sickling. Morphine Rationale: it prescribes to help control the pain experienced in a vaso-oclusive crisis. Penicillin VK Rationale: prescribed as a prophylaxis.

what is the second most common congenital heart defect?

Hypoplastic left heart is the second most common congenital heart defect, caused by underdevelopment of the left side of the heart, aorta, aortic valve, left ventricle, and mitral valve.

What is a "TET" spell?

Hypoxemia and cyanosis that can occur during stressful circumstances like exercise, crying, defecation, IV placement in the awake child, or during induction

Medication that can help open the constricted ductus arteriosus?

IV prostaglandin E therapy can open a constricted ductus arteriosus and allow for oxygenation of the body until surgery is performed on patient with TOF

What are the medications treatment for PDA?

Ibuprofen Indomethacin aspirin IV acetaminophen

A nurse is assessing for indications to administer packed red blood cells. Select indications for administration. Select all that apply. Impending heart failure Recurrent priapism Hemoglobin >8 grams in a stable patient with chronic anemia Hypervolemia Splenic sequestration

Impending heart failure Recurrent priapism Splenic sequestration

A nurse is planning care for a human immunodeficiency virus (HIV) positive child. Which interventions will the nurse include? Select all that apply. HIV enzyme-linked immunosorbent assay (ELISA) is recommended for children 6 months and older. Virologic diagnostic testing is recommended for infants 6 months and older. In children 18 months or older, use HIV antibody assays. Western blot immunoassay is used for children 18 months and older. Rapid HIV testing may include oral specimen collection.

In children 18 months or older, use HIV antibody assays. Western blot immunoassay is used for children 18 months and older. Rapid HIV testing may include oral specimen collection.

A nurse is conducting in-service over mixed disorders of the heart. Which statements are true of mixed disorders? Select all that apply. In mixed disorders, blood from the pulmonary circulation mixes with blood from systemic circulation in the chambers of the heart. Due to mixing of the blood, there is increased volume in the ventricles, resulting in an increase in cardiac output. Due to mixing of the blood, there is an increased volume in the ventricles, resulting in a decrease in cardiac output. Examples of mixed disorders include Ebstein's anomaly and hypoplastic left heart. Children with mixed disorders will have normal oxygen saturations.

In mixed disorders, blood from the pulmonary circulation mixes with blood from systemic circulation in the chambers of the heart. Rationale: this is the definition of mixed disorder Due to mixing of the blood, there is an increased volume in the ventricles, resulting in a decrease in cardiac output. Rationale: CO decreases because of increased blood volume load on ventricles. Examples of mixed disorders include Ebstein's anomaly and hypoplastic left heart. Rationale: examples of mixed disorders include hypoplastic left heart and Ebstein's anomaly, as well as TGV, truncus arteriosus (TA), and TAPVR

A client is in the neonatal intensive care unit (NICU) and has a patent ductus arteriosus (PDA). The nurse explains to the mother that the conventional drug of choice for treating a PDA is what? Amoxicillin Indomethacin Ibuprofen Paracetamol

Indomethacin Rationale: indomethacin is the usual drug of choice, but some clients have more adverse side effects than with ibuprofen

what are symptoms of digoxin toxicity in infants and children?

Instruct parents that changes in heart rate, especially bradycardia, are one of the first signs of digoxin toxicity in infants and children Monitor baseline and periodic ongoing potassium, magnesium, and calcium levels. **** anorexia, nausea, vomiting, diarrhea, and visual disturbances. Do not breastfeed without contacting your physician if you are taking this medication

A nurse is teaching the caregiver of a child with lead poisoning. Caregiver education will include which of the following? Select all that apply. Instructing about lead hazards and sources Relocating the child from the environment until lead is removed Cleaning the environment with a disinfectant and discard in a puncture resistant container Referring to a physical therapist to evaluate the child as needed Referring to a speech and child developmental specialist to evaluate child as needed

Instructing about lead hazards and sources Relocating the child from the environment until lead is removed Referring to a speech and child developmental specialist to evaluate child as needed

The mother of a premature infant asks the nurse to describe the patent ductus arteriosus. Which is the best description of this anomaly? It is a defect that allows blood to flow from the right to the left side of the heart, resulting in decreased oxygen flow to the body. It is not unusual for preterm infants to have a patent ductus arteriosus. The severity depends upon the age of the neonate and size of the opening. The defect is due to low pulmonary pressures in the vessels in utero. Substances produced in the lungs called prostaglandins help to keep this open. It will eventually close due to increased oxygenation.

It is not unusual for preterm infants to have a patent ductus arteriosus. The severity depends upon the age of the neonate and size of the opening. Rationale: the incidence of the ductus staying open increases as the gestational age of the infant dereases. There are variations in severity.

Jones Criteria

Joints: polyarthritis O: imagine the heart is an O=> carditis Nodules (subcutaneous) Erythema marginatum Sydenham's chorea

what is the current recommendation for lipid screening?

LDL over 230 mg/dL **** age 9-11 year old need to Fast for lab then 17-21 year regardless of family history

Jones criteria for rheumatic fever

Major: Carditis, Polyarthritis, Chorea, Erythema marginatum, Subcutaneous nodules Minor: Arthralgia, Fever, Elevated Acute Phase Reactants, Prolonged PR, Previous hx of GAS or RF 2 Major or 1 major 2 Minor

A nurse is providing care to a child with an oncologic disorder. Please identify nursing interventions for the child. Select all that apply. Monitor for seizures and changes in posturing. Assess oral cavity for stomatitis. Place the child in Trendelenburg position to facilitate drainage. Monitor level of consciousness and sleep patterns. Allow the child to lie with head sideways on the pillow.

Monitor for seizures and changes in posturing. Assess oral cavity for stomatitis. Monitor level of consciousness and sleep patterns.

What does the care of a child following cardiac catheterization include? Select all that apply. Monitoring pulses above and below the site monitoring intake and output placing the child side-lying position with the affected extremity down Instructing family members to keep the child quiet for 8 hours following the procedure Placing a dry, occlusive dressing over the site 24 hours after the procedure

Monitoring pulses above and below the site monitoring intake and output Placing a dry, occlusive dressing over the site 24 hours after the procedure

A nurse is planning care for a patient with thrombocytopenia. Which nursing intervention will be included? Select all that apply. Monitoring stools for blood Avoiding injections Avoiding use of salicylates (aspirin) Avoiding steroids Monitoring activities

Monitoring stools for blood Avoiding injections Avoiding use of salicylates (aspirin) Monitoring activities

A nurse is assessing a child with Acquired Immunodeficiency Syndrome (AIDS). Which assessment findings are present? Select all that apply. Chronic or recurrent urinary tract infection Mucocutaneous eruptions Epiglottitis Parotitis Chronic or recurrent diarrhea

Mucocutaneous eruptions Parotitis Chronic or recurrent diarrhea

What is the adventitious sound for atrial septal defect?

Murmur can be heard during routine physical exam for ASD

The nurse is caring for a toddler diagnosed with Kawasaki's disease when the child states, "My heart hurts." What should the nurse assess for related to these symptoms? Elevated blood pressure Myocardial infarction Renal impairment and fluid overload Need to educate the child on truth telling Next

Myocardial infarction Rationale: MI occurs in 73% of children with Kawasaki's disease.

70. The most accurate physiologic reason for respiratory distress in respiratory distress syndrome (RDS) is: 1. Altered surface tension causes fluid and protein leak, preventing atelectasis and ground glass appearance on CXR. 2. Infants with RDS are premature and incidence of RDS increases with increased gestational age. 3. Infants with RDS have a decreased number of alveoli, increased surface tension, and decreased AP diameter, limiting lung development. 4. Infants with RDS have altered surface tension, which produces hyaline membrane, atelectasis, and hypoventilation.

NS: 4 Feedback 1. The hypoventilation occurring in RDS causes an increased risk. 2. RDS can occur in any gestational age neonate. 3. The neonates have damage to the alveoli, not a decreased number. 4. Infants with RDS have altered surface tension, which produces, hyaline membrane, atelectasis and hypoventilation.

An infant presents to the emergency department with a monitored heart rate of 254. Which treatment would be appropriate for the nurse to institute? Nasal suctioning Prepare for unsynchronized defibrillation. Insert an IV for a fluid bolus. Obtain a weight in order to prepare medications.

Nasal suctioning Rationale: nasal suctioning will stimulate a vagal response and slow heart rate in SVT

What is the initial dosage of prostaglandin E1?

Neonatal initial dose: 0.05-0.1 mcg/kg/min IV Maintenance dosage: 0.01-0.4 mcg/kg/min IV

A nurse is teaching about various cancers in children. Which type of cancer is usually diagnosed <1 year of age? Wilms' tumor Neuroblastoma Eye cancer Hodgkin's cancer

Neuroblastoma

Medication regimen for pulmonary artery hypertension

Oxygen CCB Prostacyclin sildenafil

What the heart sound of patent Ductus arteriosus?

PDA patient's heart sounds like a washing machine

What is the prostaglandin E1 mechanism of action?

POTENT VASODILATOR Relaxes smooth muscle of the ductus arteriosus, leading to increased pulmonary blood flow with increased blood oxygenation and lower body perfusion; increases SpO2 and PaO2

What is the only prophylactic medication for RSV?

Palivizumab (Synagis) is the only prophylactic medication for RSV

A nurse is monitoring a child with suspected thrombocytopenia. The nurse will observe for which clinical manifestation? History of epistaxis Petechiae, often over bony prominences History of sickle cell Recent exposure to bacterial pneumonia

Petechiae, often over bony prominences

A nurse is assessing clinical indications for administration of platelets. Select indications for administration. Select all that apply. Platelet count <20,000 Hypervolemia Thrombocytopenia Pallor Active bleeding

Platelet count <20,000 Thrombocytopenia Active bleeding

A 12-year-old male child is diagnosed with acquired thrombocytopenia (ATP) after an infection with Rocky Mountain spotted fever. The nurse understands that which diagnostic examination will identify ATP? Partial thromboplastin time: 28 seconds Total iron-binding capacity values: 280 mg/dL Platelet count: 80 × 103/mcL Hemoglobin: 13.5 g/dL

Platelet count: 80 × 103/mcL Rationale: normal range is 195-365.

A nurse is teaching a caregiver for the child undergoing hematopoietic stem cell transplantation (HSCT). Which statement is true for the child? Discourage sibling participation in child's care until stabilized. Care will be provided only by trained personnel. Prepare the child for isolation to undergo intensive ablative therapy. Adherence to standard precautions is warranted.

Prepare the child for isolation to undergo intensive ablative therapy. Rationale: ablative therapy is a prefer treatment for HSCT

B-T shunt procedure

Procedure connecting the aorta to the pulmonary artery to push deoxygenated oxygen over to the lungs => left atria => left ventricle. Help improved O2

A nurse is teaching a caregiver of a child with cancer. Which statement regarding caregiver education is true? Select all that apply. Discourage discussions centered around caregiver guilt. Provide education to the caregiver and child. Explain the plan of care for the child. Avoid discussions on survival rate. Encourage emotional support.

Provide education to the caregiver and child. Explain the plan of care for the child. Encourage emotional support.

How do you test BP for COA?

Pulse ox check in right hand and left foot.

A nurse is assessing a child for leukemia. Which assessment finding confirms diagnosis? Purpura Auditory hallucinations Bulimia Bone and joint infection

Purpura

what are the side effects of Beta-adrenergic (racemic epinephrine) in treating RSV and laryngotracheobronchitis?

Racemic epinephrine can lead to tachycardia hypertension headache anxiety

Nursing intervention for congestive heart failure (CHF)

Raise HOB to improve O2 minimizing stimulation/decrease work of breathing digoxin (Lanoxin) therapy diuretic (furosemide) inotropic agent.

The nurse is preparing to administer digoxin to a child in the hospital who routinely takes the medication at home. Which is the appropriate action for the nurse to take prior to administration? Request a second nurse to verify the dosage and amount prior to administration. Make sure the child's tray is present so they can take the dosage with food. Ask the mother if this is the same dosage administered at home. Obtain vital signs immediately after administration to note changes in heart rate.

Request a second nurse to verify the dosage and amount prior to administration. Rationale: digoxin has a narrow therapeutic range.

The nurse is assessing a 3-day-old infant who has received prostaglandin E1 intravenously for hypoplastic left heart syndrome. Which side effect related to the medication requires an immediate nursing intervention? Respiratory rate of 12 breaths per minute Heart rate is 160 beats per minute Constipation Jaundice

Respiratory rate of 12 breaths per minute Rationale: respiratory depression is a potential side effect of prostaglandin E1.

A nurse is teaching a caregiver of a child receiving radiation therapy about possible side effects. Which would the nurse identify? Select all that apply Retinopathy Alopecia Bulimia Cardiomegaly Sterility

Retinopathy Alopecia Sterility

What is the adverse effect of Ribavirin?

Ribavirin can cause birth defect

What is the only drug that is licensed to treat patients with RSV?

Ribavirin is the only drug licensed to treat patients with RSV (with varying success)

An infant is diagnosed with central apnea. What symptoms might the nurse expect to see for this infant? Select all that apply. Observed periods of cessation of breathing for 20 seconds or more Enlarged tonsils Seizure activity Desaturations on the pulse oximeter Primarily nasal breathing and nasal flaring

Seizure activity Desaturations on the pulse oximeter explain: central apnea has a period of severe episodic breathing, particularly if there are more than 10 episodes per night. Seizure activity can occur, especially if the infant has a diagnosis of intraventricular hemorrhage or meningitis The saturations are common with central apnea, and oxygen may be required to assist with periods of hypoxia.

A nurse is planning care for a child with suspected sickle cell disease. Which diagnostic test will yield results within minutes to determine if Hgb S present? Hematocrit Sickle cell turbidity test Hemoglobin electrophoresis Liver function test

Sickle cell turbidity test Rationale: Sickle cell turbidity test with fingerstick blood yields results in 3 minutes to determine if Hgb S present.

side effects of indomethacin?

Side effects include renal insufficiency, necrotizing enterocolitis, gastrointestinal bleeding, gastrointestinal perforation, or bleeding disorders *indomethacin

A nurse is assessing a child with a brain tumor. Identify assessment findings associated with this diagnosis. Select all that apply. Decrease in head size Sluggish pupils Unequal and weak hand grasp Headache at bedtime Nausea and vomiting

Sluggish pupils Unequal and weak hand grasp Nausea and vomiting

What are the treatments for "TET" spell?

Supplemental oxygen sedation volume expansion for the hemodilution of polycythemia knee-to-chest position

The nurse understands appropriate drug therapy for infants with bronchopulmonary dysplasia (BPD) includes which therapy? Surfactant vitamin D methylphenidate (Ritalin) sodium chloride

Surfactant explain: infants with BPD have decreased amount of surfactant.

A nurse is planning care for a child with acute lymphocytic leukemia (ALL). Which statement is true? Select all that apply. Swollen glands are a clinical manifestation of ALL. ALL is the most common type of cancer in children. Philadelphia chromosome abnormality is a possible cause of ALL. Previous chemotherapy is a possible cause of ALL. The cause is unknown.

Swollen glands are a clinical manifestation of ALL. ALL is the most common type of cancer in children. Previous chemotherapy is a possible cause of ALL.

Syndrome X (metabolic syndrome)

Syndrome X is the triad of insulin resistance, hypertension, and hyperlipidemia. resulted in a dramatic increase in obesity in young children.

what is the normal systolic BP in infants?

Systolic BP in infants is between 70 and 90 mm Hg; adolescents reach adult levels.

What is transposition of great arteries or vessels?>

TGA is when the aorta and the pulmonary artery switch with one another's ventricle.

What are the treatments for TGA?

TGA treatments include: surgery and prostaglandin E1 drip to maintain fetal circulation through PDA. • Monitor children prescribed captopril/enalapril to relax the coronary arteries.

An infant with total anomalous pulmonary venous return (TAPVR) suddenly develops tachypnea, cyanosis, tachycardia, and a change in level of consciousness. What is the correct interpretation for the nurse to make regarding these findings? The infant had a patent ductus arteriosus (PDA), which closed. The infant is showing incompetence of the tricuspid valve and blood flow to the lungs. The infant is showing signs of septic shock. The child has developed a respiratory infection.

The infant had a patent ductus arteriosus (PDA), which closed. Rationale: while the PDA is open, the child has few symptoms. When the PDA closes, profound cyanosis, severe shock, and congestive heart failure result.

Which blood flow dynamics are involved in transposition of the great vessels (TGV)? Select all that apply. The systemic and pulmonary circulation are always mixed. The left ventricle sends oxygenated blood to the lungs. The aorta receives oxygenated blood from the right ventricle for systemic circulation. The aorta receives unoxygenated blood from the right ventricle for systemic circulation. An arterial or ventricular septal defect is a lifesaving defect in an infant with TGV.

The left ventricle sends oxygenated blood to the lungs. Rationale: The pulmonary artery receives oxygenated blood from the left ventricle that then goes back to the lungs. The aorta receives unoxygenated blood from the right ventricle for systemic circulation. An arterial or ventricular septal defect is a lifesaving defect in an infant with TGV. Rationale: ASD and VSD, it helps with mixing unoxygenated and oxygenated blood through the septal.

A nurse reviewing the prenatal history of an expectant mother notes the use of lithium for bipolar disorder. How should the nurse interpret this finding? There is no risk to the unborn child. There is a demonstrated link between lithium and certain acquired diseases. The mother's diet will need to be closely monitored during pregnancy to avoid potential harm from the medication. Lithium only poses a threat to the unborn child if certain genetic factors are present.

There is a demonstrated link between lithium and certain acquired diseases. Rationale: Chemical agents, including lithium, are risk factors for acquired heart disease.

A nurse is planning prioritized care for children with complex hematologic and oncologic disorders. Which statements are true related to prioritized care? Select all that apply. Triage clients quickly and place in a treatment room as soon as possible. Avoid invasive procedures. Use strict standard precautions. Start antibiotics if febrile within 24 hours of assessment. Dedicate stethoscopes and equipment to the room.

Triage clients quickly and place in a treatment room as soon as possible. Avoid invasive procedures. Dedicate stethoscopes and equipment to the room.

What is tricuspid Atresia?

Tricuspid atresia is a heart condition in which the tricuspid valve is either defective or missing. defect blocks the blood flow from the right atrium to the right ventricle, diminishing the blood flow to the lungs

In fetal circulation, which structure is responsible for carrying oxygenated blood to the fetus? Umbilical vein Pulmonary artery Patent ductus arteriosus Aorta

Umbilical vein rationale: Oxygenated blood is returned to the fetus via the umbilical vein flowing into the inferior vena cava.

which medication is contraindicated for cystic fibrosis?

Use of antihistamines is contraindicated for patients with CF because these drugs have a drying effect, making expectoration of mucus more difficult.

What is the number one most common congenital heart defect?

VSD is the most common overall congenital heart disease

The nurse is auscultating the heart of a newborn infant and notes a holosystolic murmur and thrill in the left lower sternal border. Which defect should the nurse suspect? Ventricular septal defect (VSD) Atrial septal defect (ASD) Patent ductus arteriosus (PDA) Ebstein's anomaly

Ventricular septal defect (VSD) rationale: these are signs of VSD

what is the diagnostic tests for PDA?

Wide pulse pressures—low diastolic pressures • Increased vascular markings on the chest x-ray are a late sign of an enlarged heart • Hyperactive precordium • An echocardiogram will show increased enlargement of left heart chambers

A nurse is planning care for a child with a suspected oncologic disorder. Which statement is true relating to oncologic disorders with children? Possible cancer causes are heredity and genetics. Causes of most cancers are known. Hodgkin's lymphoma is typically untreatable. Wilms' tumor of the kidney is typically discovered by caregivers.

Wilms' tumor of the kidney is typically discovered by caregivers. Rationale: this is discovery usually during bathing and dressing

A child is suspected of having rheumatic heart disease. Which history question is a priority for the nurse in helping to establish this diagnosis? a. "Did your child complete antibiotics for strep throat in the last month?" b. "Has your child had low-grade fever in the last week?" c. "Has your child demonstrated a rash on the legs recently?" d. "Has your child been swimming in rivers or lakes?"

a. "Did your child complete antibiotics for strep throat in the last month?"

An adolescent client being seen in the clinic demonstrates a blood pressure of 145/92. The mother asks if the child has hypertension. How should the nurse respond? a. "Hypertension is not diagnosed based on a single reading." b. "I cannot diagnose, but your child will need to be referred to a cardiologist." c. "We don't routinely monitor blood pressure until adult age, so I would not worry." d. "Blood pressure in adolescents is very different than in adults, so those numbers are fine."

a. "Hypertension is not diagnosed based on a single reading."

The parents of a neonate with a patent ductus arteriosus ask why their child is receiving ibuprofen if the child is not in pain. What is the nurse's best response? a. "Ibuprofen will help facilitate closure of the patent ductus arteriosus." b. "Ibuprofen is used prophylactically to prevent infection in children with cardiac disorders." c. "Ibuprofen will help keep the ductus open in order to provide additional blood flow to the body." d. "Routine pain medication is given to children with cardiac disorders to keep them calm."

a. "Ibuprofen will help facilitate closure of the patent ductus arteriosus."

A nurse is explaining what a tracheoesophageal (TE) fistula is to a parent. Which statement is correct? a. "It is an abnormal opening between the trachea and the esophagus." b. "The trachea and esophagus do not form in utero." c. "Both the esophagus and trachea end in a pouch." d. "The trachea connects to the stomach and the esophagus to the lungs."

a. "It is an abnormal opening between the trachea and the esophagus."

In educating a mother of a child with laryngomalacia, which statement is correct? a. "Symptoms usually resolve by age 2." b. "Your child will most likely need a tracheostomy." c. "Your child will definitely have feeding problems." d. "Administration of albuterol is most important in this condition."

a. "Symptoms usually resolve by age 2."

In educating a mother of a child with laryngomalacia, which would be a correct statement for the nurse to make? a. "Symptoms usually resolve by age 2." b. "Your child will most likely need a tracheostomy." c. "Your child will definitely have feeding problems." d. "Administration of Albuterol is most important in this condition."

a. "Symptoms usually resolve by age 2."

The nurse is counseling a client undergoing an electrocardiogram (EKG) about the electrical conduction of the heart. Which statement is correct regarding the anatomical structure that is found in the right atrium and is needed for the contraction of the aorta to get blood flow to the ventricles? a. "The sinoatrial (SA) node acts as the heart's pacemaker, helping the heart to set and maintain a steady heart rate." b. "The patent foramen ovale (PFO) acts as the heart's pacemaker, helping the heart set and maintain a steady heart rate." c. "The PQ interval acts as the heart's pacemaker, helping the heart set and maintain a steady heart rate." d. "The atrioventricular (AV) node acts as the heart's pacemaker, helping the heart set and maintain a steady heart rate."

a. "The sinoatrial (SA) node acts as the heart's pacemaker, helping the heart to set and maintain a steady heart rate."

A student nurse asks which classification of congenital heart defects results in the combining of blood between systemic and pulmonary circulation in the heart chambers, which results in a desaturation of blood oxygen levels and a decrease in cardiac output related to an increased volume load on ventricles. What is the appropriate nursing response? a. "This type of congenital heart defect is classified as a mixed defect." b. "You are describing a defect that is related to a decrease in pulmonary blood flow." c. "The defect that you are describing is brought on by an increased pulmonary blood flow." d. "This defect is referred to as an obstructive disorder."

a. "This type of congenital heart defect is classified as a mixed defect."

The nurse is taking a blood pressure on a 4-year-old cardiac client. To ensure a proper reading, a blood pressure cuff should be what? a. 25% greater than the width of the extremity b. 25% smaller than the width of the extremity c. 35% greater than the width of the extremity d. 42% the width of the extremity

a. 25% greater than the width of the extremity

Which child does the nurse anticipate to be most at risk for being hospitalized for respiratory syncytial virus (RSV)? a. A 3-month-old who was born at 30 weeks gestation b. A 18-month-old with a tracheostomy c. A 4-year-old with a ventricular septal defect (VSD) d. A 5-year-old who was term but has never received any immunizations

a. A 3-month-old who was born at 30 weeks gestation

Which statement does the nurse understand to be true about clients admitted with pertussis (whooping cough)? a. A paroxysmal cough frequently at night might be present. b. The client should be maintained on contact precautions/isolation. c. Peak occurrence is in the fall and winter. d. It is most often caused by Haemophilus influenzae.

a. A paroxysmal cough frequently at night might be present.

Which abnormal finding indicates to the nurse that a child likely has cystic fibrosis (CF)? a. A positive sweat chloride test b. A positive blood culture c. Excessive mucus production d. Edema and weight gain

a. A positive sweat chloride test

The nurse is caring for a child diagnosed with Kawasaki's disease. Which interventions should the nurse anticipate as part of the treatment protocol? Select all that apply. a. Administration of IV gamma globulin b. Aspirin administration, high dose for weight c. Emergent echocardiogram d. Administer varicella (chickenpox) vaccine prior to discharge e. Help parents cope with the irritability of the child

a. Administration of IV gamma globulin b. Aspirin administration, high dose for weight e. Help parents cope with the irritability of the child

The nurse is assessing a 2-day-old infant and notes blood pressure in the arm of 69/45, with a blood pressure in the leg of 45/32. The nurse understands this discrepancy is associated with which disorder? a. Aortic arch abnormality b. Ventricular septal defect c. Atrial septal defect d. Patent ductus arteriosus

a. Aortic arch abnormality

Which congenital heart defect results in pulmonary hypertension and right atrial enlargement from an increase in pulmonary blood? a. Atrial septal defect b. Tetralogy of Fallot c. Tricuspid atresia d. Aortic stenosis

a. Atrial septal defect

A client has just undergone cardiac catheterization. The nurse is preparing discharge instructions. What should the nurse include in the instructions? Select all that apply. a. Avoid strenuous activity such as lifting, sports, or physical education, although school is appropriate. b. A regular diet can be consumed. c. Follow-up appointments are essential. d. Fever is common following catheterization and should last longer than 24 hours and go above 100°F (37.7°C). e. Keep lower extremities slightly flexed for 4 to 6 hours following the catheterization.

a. Avoid strenuous activity such as lifting, sports, or physical education, although school is appropriate. b. A regular diet can be consumed. c. Follow-up appointments are essential.

The nurse is providing discharge teaching to parents of a child receiving digoxin (Lanoxin). Which items should the nurse include as signs of digoxin toxicity? Select all that apply. a. Bradycardia b. Nausea c. Diarrhea d. Increase in appetite e. Flushing of the skin

a. Bradycardia b. Nausea c. Diarrhea

The nurse is developing a teaching plan about medication management for a client who is taking a loop diuretic. Which drug should be included in the plan? a. Bumetanide (Bumex) b. Hydrochlorothiazide (Diuril) c. Amoxicillin (Amoxil) d. Spironolactone (Aldactone)

a. Bumetanide (Bumex)

The nurse is measuring blood pressure on the leg of a sleeping toddler and receives an unusually high reading. What is the best action for the nurse? a. Confirm the appropriate size of the cuff and obtain a larger cuff if needed to recheck pressures. b. Confirm the appropriate size of the cuff and obtain a smaller cuff if needed to recheck pressures. c. Allow the child to rest and recheck with the same cuff in approximately 10 minutes. d. Document the finding and notify the practitioner.

a. Confirm the appropriate size of the cuff and obtain a larger cuff if needed to recheck pressures.

A 3-year-old is noted to speak in a muffled tone and frequently complains of a sore throat. The mother has noted that lately the child is very noisy when he breathes. She states that he seems to "stop breathing." A sleep study was done to confirm obstructive sleep apnea. Which treatments would the nurse expect to occur for this child? Select all that apply. a. Continuous positive airway pressure (CPAP) b. Tonsillectomy c. Administration of Caffeine Citrate d. High caloric diet e. Waking the child at frequent intervals

a. Continuous positive airway pressure (CPAP) b. Tonsillectomy d. High caloric diet

Which techniques would a nurse implement when caring for a child diagnosed with respiratory syncytial virus (RSV)? Select all that apply. a. Corticosteroids b. IV and/or oral fluid therapy c. IV antibiotics for infection d. Cardiorespiratory monitoring e. Immediate intubation to maintain the airway

a. Corticosteroids b. IV and/or oral fluid therapy d. Cardiorespiratory monitoring

A nurse is caring for a neonate with a suspected tracheoesophageal fistula (TEF). What nursing intervention would appropriate for this client? a. Elevate the head for feedings. b. Avoid suctioning the client. c. Elevate the head, but give nothing by mouth. d. Feed the baby glucose water only.

a. Elevate the head for feedings.

The nurse is charting a care plan for a client who has undergone a heart transplant. Which common signs and symptoms of heart transplant rejection should be listed on the plan of care? Select all that apply. a. Fever b. Oliguria c. Weight gain d. Bradycardia e. Tachycardia

a. Fever b. Oliguria c. Weight gain e. Tachycardia

Which indicates the earliest sign of hemorrhage in a child who has just had a tonsillectomy? a. Frequent swallowing b. Labored respirations c. Tachypneic stridor d. Dark brown emesis

a. Frequent swallowing

The nurse working in the pediatric cardiac care unit (CCU) notices that a client who is 8 years old has a rapid rhythm on the electrocardiogram (EKG) tracing. The nurse recognizes this dysrhythmia as supraventricular tachycardia (SVT) when the heart rate reaches what? a. Greater than 220 beats/minute b. Less than 200 beats/minute c. 180 to 200 beats/minute d. 70 to 90 beats/minute

a. Greater than 220 beats/minute

A nurse is conducting in-service over mixed disorders of the heart. Which statements are true of mixed disorders? Select all that apply. a. In mixed disorders, blood from the pulmonary circulation mixes with blood from systemic circulation in the chambers of the heart. b. Due to mixing of the blood, there is increased volume in the ventricles, resulting in an increase in cardiac output. c. Due to mixing of the blood, there is an increased volume in the ventricles, resulting in a decrease in cardiac output. d. Examples of mixed disorders include Ebstein's anomaly and hypoplastic left heart. e. Children with mixed disorders will have normal oxygen saturations.

a. In mixed disorders, blood from the pulmonary circulation mixes with blood from systemic circulation in the chambers of the heart. c. Due to mixing of the blood, there is an increased volume in the ventricles, resulting in a decrease in cardiac output. d. Examples of mixed disorders include Ebstein's anomaly and hypoplastic left heart.

Which statements are true about apnea? Select all that apply. a. It occurs in most infants at less than 34 weeks gestation. b. Apnea is cessation of breathing for longer than 20 seconds. c. It can be a sign of patent ductus arteriosus (PDA). d. It is not affected by maternal drug use. e. It usually resolves by 36 weeks postconceptual age.

a. It occurs in most infants at less than 34 weeks gestation. b. Apnea is cessation of breathing for longer than 20 seconds. c. It can be a sign of patent ductus arteriosus (PDA). e. It usually resolves by 36 weeks postconceptual age.

You are teaching a family with a child who has cystic fibrosis (CF) about bronchial hygiene. Which of the following teaching points are correct to include? Select all that apply. a. It should be performed three to four times a day. b. It may cause bronchospasm. c. It is all right to percuss over the spine or internal organs. d. When manually percussing, you should use a cupped hand. Perform on bare skin for better results.

a. It should be performed three to four times a day. b. It may cause bronchospasm. d. When manually percussing, you should use a cupped hand.

A 2-week-old who was born at 35 weeks is in respiratory distress. The parents state that the infant has always been a "noisy breather that sounds like a crow." Upon initial assessment, severe suprasternal retractions and stridor are present that worsen when the infant is lying down. What illness might the nurse expect the child to have? a. Laryngomalacia b. Apnea of prematurity c. Bronchiolitis d. Severe gastroesophageal (GE) reflux

a. Laryngomalacia

What does the care of a child following cardiac catheterization include? Select all that apply. a. Monitoring pulses above and below the site b. Monitoring intake and output c. Placing the child in a side-lying position with the affected extremity down d. Instructing family members to keep the child quiet for 8 hours following the procedure e. Placing a dry, occlusive dressing over the site 24 hours after the procedure

a. Monitoring pulses above and below the site b. Monitoring intake and output e. Placing a dry, occlusive dressing over the site 24 hours after the procedure

Which are signs and symptoms of respiratory distress syndrome in a 2-month-old? Select all that apply. a. Nasal flaring b. Intercostal retractions c. Coughing d. Bronchovesicular lung sounds e. Grunting

a. Nasal flaring b. Intercostal retractions e. Grunting

An infant presents to the emergency department with a monitored heart rate of 254. Which treatment would be appropriate for the nurse to institute? a. Nasal suctioning b. Prepare for unsynchronized defibrillation. c. Insert an IV for a fluid bolus. d. Obtain a weight in order to prepare medications.

a. Nasal suctioning

An infant is diagnosed with central apnea. What symptoms might the nurse expect to see for this infant? Select all that apply. a. Observed periods of cessation of breathing for 20 seconds or more b. Enlarged tonsils c. Seizure activity d. Desaturations on the pulse oximeter e. Primarily nasal breathing and nasal flaring

a. Observed periods of cessation of breathing for 20 seconds or more c. Seizure activity d. Desaturations on the pulse oximeter

The nurse is educating a group of new nurses and explaining congenital heart defects classified as having increased pulmonary blood flow. Which defects should be included in this teaching session? Select all that apply. a. Patent ductus arteriosus (PDA) b. Atrial septal defects (ASDs) c. Ventricular septal defects (VSDs) d. Tetralogy of Fallot (TOF) e. Tricuspid atresia

a. Patent ductus arteriosus (PDA) b. Atrial septal defects (ASDs) c. Ventricular septal defects (VSDs)

Which assessment findings by the nurse indicate possible indicators of congenital heart disease (CHD) in children? Select all that apply. a. Poor weight gain b. Dysmorphic features c. LGA d. Clubbing & erythema in fingers & toes e. Anemia

a. Poor weight gain b. Dysmorphic features d. Clubbing & erythema in fingers & toes

A child just returned from the cardiac catheterization lab. What should the nurse include to provide safe and effective care of this client? Select all that apply. a. Provide developmentally appropriate postoperative instructions. b. Instruct that baths should be performed, not showers. c. Keep extremities straight for 4 to 6 hours with no movement. d. Monitor for temperature changes or color changes in the arm or leg that is used for the catheterization. e. Keep the client on a liquid diet.

a. Provide developmentally appropriate postoperative instructions. c. Keep extremities straight for 4 to 6 hours with no movement. d. Monitor for temperature changes or color changes in the arm or leg that is used for the catheterization.

The nurse is caring for a child with a congenital heart defect and notes an increased work of breathing with tachypnea, grunting, and nasal flaring. The nurse should suspect which complication has developed? a. Pulmonary edema b. Polycythemia c. Clubbing d. Hyperoxia

a. Pulmonary edema

A 15-month-old is admitted to the pediatric unit with a history of a recent upper respiratory infection. Which symptom is consistent with the diagnosis of laryngo-tracheobronchitis (croup)? Select all that apply. a. Reported inspiratory stridor that is worse at night. b. Suprasternal retractions are present upon examination. c. The toddler has a barking, seal-like, harsh cough. d. Lung sounds have inspiratory wheezing. e. Lung sounds with crackles in the bases bilaterally.

a. Reported inspiratory stridor that is worse at night. b. Suprasternal retractions are present upon examination. c. The toddler has a barking, seal-like, harsh cough.

The nurse is preparing to administer digoxin to a child in the hospital who routinely takes the medication at home. Which is the appropriate action for the nurse to take prior to administration? a. Request a second nurse to verify the dosage and amount prior to administration. b. Make sure the child's tray is present so they can take the dosage with food. c. Ask the mother if this is the same dosage administered at home. d. Obtain vital signs immediately after administration to note changes in heart rate.

a. Request a second nurse to verify the dosage and amount prior to administration.

The nurse is assessing a 3-day-old infant who has received prostaglandin E1 intravenously for hypoplastic left heart syndrome. Which side effect related to the medication requires an immediate nursing intervention? a. Respiratory rate of 12 breaths per minute b. Heart rate is 160 beats per minute c. Constipation d. Jaundice

a. Respiratory rate of 12 breaths per minute

Nursing education provided to caregivers of a child diagnosed with bronchopulmonary dysplasia (BPD) should include which interventions? Select all that apply. a. Safe oxygen administration b. Signs and symptoms of respiratory distress c. Strict hand washing d. Ventilator management e. Low-calorie feedings

a. Safe oxygen administration b. Signs and symptoms of respiratory distress c. Strict hand washing d. Ventilator management

The nurse understands appropriate drug therapy for infants with bronchopulmonary dysplasia (BPD) includes which therapy? a. Surfactant b. Vitamin D c. Methylphenidate (Ritalin) d. Sodium Chloride (NaCl)

a. Surfactant

The nurse should instruct the parent whose child is diagnosed with respiratory syncytial virus (RSV) to notify the health-care provider for which issue? Select all that apply. a. The child is not eating. b. There is a decrease in wet diapers. c. There is increased work of breathing. d. The child develops yellow drainage from the nose. e. Only when the child wheezes.

a. The child is not eating. b. There is a decrease in wet diapers. c. There is increased work of breathing.

An infant with total anomalous pulmonary venous return (TAPVR) suddenly develops tachypnea, cyanosis, tachycardia, and a change in level of consciousness. What is the correct interpretation for the nurse to make regarding these findings? a. The infant had a patent ductus arteriosus (PDA), which closed. b. The infant is showing incompetence of the tricuspid valve and blood flow to the lungs. c. The infant is showing signs of septic shock. d. The child has developed a respiratory infection.

a. The infant had a patent ductus arteriosus (PDA), which closed.

The nurse is discussing with the parents of a child just diagnosed with a cardiac defect an upcoming noninvasive test that indicates structure, size, flow patterns, function, and the blood vessels attached to the heart. The parents ask the nurse what the test is called, as they would like to do more research. What is the appropriate nursing response? a. This test is called an echocardiogram b. An EKG is the test used to look at flow pattern functions c. This is called angiography d. This is referred to as cardiac catheterization

a. This test is called an echocardiogram

The nurse is preparing a teaching session for new nurses on congenital heart defects classified as having decreased pulmonary blood flood flow. Which defects should be included in the teaching? Select all that apply. a. Tricuspid atresia b. Eisenmenger syndrome c. Ventricular septal defects (VSDs) d. Tetralogy of Fallot (TOF) e. Patent ductus arteriosus (PDA)

a. Tricuspid atresia b. Eisenmenger syndrome d. Tetralogy of Fallot (TOF)

In fetal circulation, which structure is responsible for carrying oxygenated blood to the fetus? a. Umbilical vein b. Pulmonary artery c. Patent ductus arteriosus d. Aorta

a. Umbilical vein

The nurse is auscultating the heart of a newborn infant and notes a holosystolic murmur and thrill in the left lower sternal border. Which defect should the nurse suspect? a. Ventricular septal defect (VSD) b. Atrial septal defect (ASD) c. Patent ductus arteriosus (PDA) d. Ebstein's anomaly

a. Ventricular septal defect (VSD)

treatment for RF

antimicrobial therapy initially Chemoprophylxis monthly for a minimum of 5 years rest pain: anti-inflammatory, steroids, and aspirin

The nurse is educating parents of a toddler about ways to avoid foreign body aspiration. Which statement made by the parents indicates their understanding of this concept? a. "I will use back blows and abdominal thrusts for my toddler if he begins to choke." b. "I will cut table foods into small pieces before feeding my child." c. "I will allow my 2-year-old to play with his older brother's Legos." d. "I will make sure he chews his food properly."

b. "I will cut table foods into small pieces before feeding my child."

The mother of a child recently diagnosed with ventricular septal defect (VSD) wants to know how this disorder affects the lungs. What is the best response from the nurse? a. "VSD causes an increase in blood flow to the lungs because blood is moving from the right side of the heart to the left." b. "The excess blood flowing into the lungs makes the lungs stiffer and more difficult to move air in and out." c. "Infants with VSD also often have abnormalities in the lungs, which make it difficult for them to take in enough oxygen." d. "The left side of the heart is not formed well, making the right side of the heart more full, which creates pressure on the lungs."

b. "The excess blood flowing into the lungs makes the lungs stiffer and more difficult to move air in and out."

The nurse educator is educating a group of parents about fetal circulation. Which statements made by a parent regarding fetal circulation indicate the need for further education? Select all that apply. a. "The fetal placenta acts like the lungs during pregnancy." b. "The fetus's lungs function normally during pregnancy." c. "Fetal circulation is dependent upon the electrical impulses of the heart." d. "The fetal heart acts like the lungs during pregnancy." e. "The patent ductus arteriosus (PDA) acts like the lungs during pregnancy."

b. "The fetus's lungs function normally during pregnancy." d. "The fetal heart acts like the lungs during pregnancy." e. "The patent ductus arteriosus (PDA) acts like the lungs during pregnancy."

The nurse educator has just completed a lecture on congenital heart defects and its symptoms. The educator knows that teaching was effective when a student defines central cyanosis as what? a. A reddish discoloration of the mucous membranes, tongue, circumoral area, or core body b. A bluish discoloration of the mucous membranes, tongue, circumoral area, or core body c. A bluish discoloration of the hands or feet d. Decreased hemoglobin levels

b. A bluish discoloration of the mucous membranes, tongue, circumoral area, or core body

The nurse preceptor is preparing a teaching session for a group of nurses that will cover the etiology of acquired heart defects that may occur in childhood. What information should be included in the session? Select all that apply. a. Excessive sweating b. Autoimmune factors c. Genetic factors d. Teratogens e. Pain

b. Autoimmune factors c. Genetic factors d. Teratogens

The nurse is explaining cardiac conditions that result in the obstruction of blood flow in the heart. Which defects should be included in the teaching? Select all that apply. a. Tricuspid atresia b. Coarctation of the aorta c. Aortic stenosis d. Pulmonary atresia e. Ventricular septal defect (VSD)

b. Coarctation of the aorta c. Aortic stenosis d. Pulmonary atresia

The parents of a school-age child with a congenital heart defect state they are having some dental work done. What recommendation should the nurse make? a. Inform the dentist of the condition so they can monitor during the procedure. b. Contact the provider for a prescription of prophylactic antibiotics. c. Inform the parent the child is at no increased risk during the procedure. d. Praise the parents for providing the appropriate health maintenance for their child.

b. Contact the provider for a prescription of prophylactic antibiotics.

Which assessment finding would indicate to the nurse that the plan of care for an 8-year-old hospitalized for an acute asthma attack is effective? a. Diminished breath sounds on auscultation b. Decreased tachypnea c. Prolonged expiratory phase of breathing d. Lung sounds have expiratory wheezes

b. Decreased tachypnea

While auscultating a client's chest, the nurse observes a shift in the point of maximal impulse (PMI) and magnified heart sounds (hyperactive precordium). The nurse understands this may indicate what? a. Normal finding b. Dextrocardia c. Acrocyanosis d. Bradycardia

b. Dextrocardia

A 2-year-old was found playing with coins in his mother's purse. He has no significant past medical history. Later that day, the mother noticed he was blue and having difficulty breathing. She called 911 and the child was brought to the emergency department. What does the nurse determine as the most likely cause of his symptoms? a. Acute asthma attack b. Foreign body aspiration c. Reactive airway disease d. Acute bronchiolitis

b. Foreign body aspiration

A client is in the neonatal intensive care unit (NICU) and has a patent ductus arteriosus (PDA). The nurse explains to the mother that the conventional drug of choice for treating a PDA is what? a. Amoxicillin b. Indomethacin c. Ibuprofen d. Paracetamol

b. Indomethacin

The mother of a premature infant asks the nurse to describe the patent ductus arteriosus. Which is the best description of this anomaly? a. It is a defect that allows blood to flow from the right to the left side of the heart, resulting in decreased oxygen flow to the body. b. It is not unusual for preterm infants to have a patent ductus arteriosus. The severity depends upon the age of the neonate and size of the opening. c. The defect is due to low pulmonary pressures in the vessels in utero. d. Substances produced in the lungs called prostaglandins help to keep this open. It will eventually close due to increased oxygenation.

b. It is not unusual for preterm infants to have a patent ductus arteriosus. The severity depends upon the age of the neonate and size of the opening.

Which are risk factors for the development of sudden infant death syndrome (SIDS)? Select all that apply. a. Mothers over 40 b. Low birth weight neonates c. A multiple pregnancy like triplets d. Maternal smoking e. Mothers who breastfeed

b. Low birth weight neonates c. A multiple pregnancy like triplets d. Maternal smoking

The nurse is caring for an infant who has bronchopulmonary dysplasia (BPD). What issues would the nurse associate with this diagnosis? Select all that apply a. Term infants can develop BPD. b. Mild forms of BPD can heal as the lungs grow and remodel. c. Children with BPD often are poor feeders. d. All clients with BPD will develop heart failure. e. Pulmonary hypertension frequently occurs with BPD.

b. Mild forms of BPD can heal as the lungs grow and remodel. c. Children with BPD often are poor feeders. e. Pulmonary hypertension frequently occurs with BPD.

The nurse is caring for a toddler diagnosed with Kawasaki's disease when the child states, "My heart hurts." What should the nurse assess for related to these symptoms? a. Elevated blood pressure b. Myocardial infarction c. Renal impairment and fluid overload d. Need to educate the child on truth telling

b. Myocardial infarction

Which blood flow dynamics are involved in transposition of the great vessels (TGV)? Select all that apply. a. The systemic and pulmonary circulation are always mixed. b. The left ventricle sends oxygenated blood to the lungs. c. The aorta receives oxygenated blood from the right ventricle for systemic circulation. d. The aorta receives unoxygenated blood from the right ventricle for systemic circulation. e. An arterial or ventricular septal defect is a lifesaving defect in an infant with TGV.

b. The left ventricle sends oxygenated blood to the lungs. d. The aorta receives unoxygenated blood from the right ventricle for systemic circulation. e. An arterial or ventricular septal defect is a lifesaving defect in an infant with TGV.

A nurse reviewing the prenatal history of an expectant mother notes the use of lithium for bipolar disorder. How should the nurse interpret this finding? a. There is no risk to the unborn child. b. There is a demonstrated link between lithium and certain acquired heart diseases. c. The mother's diet will need to be closely monitored during pregnancy to avoid potential harm from the medication. d. Lithium only poses a threat to the unborn child if certain genetic factors are present.

b. There is a demonstrated link between lithium and certain acquired heart diseases.

The nurse is teaching the parents of a child using inhaled aerosolized steroids to use a spacer device with the child's metered dose inhaler (MDI). The parents want to know the purpose of the spacer. What is the appropriate nursing response? a. To minimize the risk of oral candidiasis b. To ensure proper dosing c. To minimize the adverse effects of the drug d. To cleanse the oral cavity

b. To ensure proper dosing

The nurse is taking care of a 12-year-old client with cystic fibrosis (CF). The breakfast tray has arrived. The client is ordered pancreatic enzymes. When should the nurse administer this medication for maximal effect? a. One hour before breakfast b. With breakfast c. Two hours after breakfast d. With antibiotics

b. With breakfast

The nursing student asks the nurse about genetic implications related to cystic fibrosis (CF). How should the nurse respond? a. "It is inherited as an autosomal dominant trait." b. "It is a genetic defect found primarily in non-Caucasian people." c. "If it is present in a child, both parents are carriers of the defective gene." d. "There is a 50% chance the siblings of an affected child will also be affected."

c. "If it is present in a child, both parents are carriers of the defective gene."

The nurse is speaking with a parent of a premature infant with apnea of prematurity. Which statement made by the parent indicates to the nurse that they understand the diagnosis? a. "It can be cured if my baby takes a lot of caffeine." b. "It is caused by my baby being tired." c. "It is a pause in breathing for 20 seconds or more." d. "It is caused by infants sensing carbon dioxide in their brain to breathe."

c. "It is a pause in breathing for 20 seconds or more."

A 9-month-old is admitted with influenza. Which statement made by the nurse would be the best response when caring for this infant? a. "This infection could have been prevented if the parents washed their hands at home." b. "Antibiotics should be given as soon as a diagnosis is made to prevent further infection." c. "Supportive care such as encouraging fluids to liquefy secretions will help prevent dehydration." d. "Antiviral medications such as Oseltamivir (Tamiflu) can be given at any time during the illness.

c. "Supportive care such as encouraging fluids to liquefy secretions will help prevent dehydration."

A mother is breastfeeding her infant and looks down to notice her baby's lips are blue and the baby isn't breathing. She takes the baby off the breast and rubs the baby's sternum. The baby then coughs and starts to breathe. The mother calls 911 and the child is admitted for an acute life-threatening event (ALTE). The mother is crying and wants to know what happened. She is afraid to breastfeed the baby again. What would be the correct statement to tell the mother? a. "Don't worry; the baby won't die if you breastfeed again." b. "Does your baby have reflux? This could explain it." c. "We need to be sure the baby's nose is not obstructed." d. "When your baby naps, be extra vigilant because he might have sudden infant death syndrome (SIDS)."

c. "We need to be sure the baby's nose is not obstructed."

The nurse is educating the parents of a child with asthma about environmental control. The nurse knows teaching has been understood if the parent states: a. "We will allow the cat to sleep in her room just on the floor." b. "We will be sure to keep the house cool throughout the year." c. "We will be sure to use an exterminator to decrease the likelihood of cockroaches or other bugs in the house." d. "We will use the wood fireplace often in the winter to keep our house warm."

c. "We will be sure to use an exterminator to decrease the likelihood of cockroaches or other bugs in the house."

While assessing an infant diagnosed with a congenital heart defect (CHD) in the newborn intensive care unit, the nurse notes the results from the arterial blood gas indicate a Pao2 level of 30. After bagging the infant with 100% oxygen, the Pao2 level is 34. What does the nurse suspect related to the assessment findings and diagnosis? a. Normal newborn assessment findings, no abnormalities b. Pulmonary disease c. Continued mixing of oxygenated and nonoxygenated blood d. Metabolic disease

c. Continued mixing of oxygenated and nonoxygenated blood

The nurse is planning a primary prevention initiative to prevent bacterial endocarditis in children with prosthetic valvular repairs. Which intervention should be included in the initiative? a. Avoid dental procedures. b. Avoid surgical procedures. c. Counsel parents on the importance of antibiotics for all dental and surgical procedures. c. Antibiotics are only needed for surgical procedures if fever arises.

c. Counsel parents on the importance of antibiotics for all dental and surgical procedures.

The nurse instructs the parent of a 5 -year-old with asthma to use the AeroChamber because it: a. Can help identify asthma triggers b. Monitors daily changes in airway reactivity c. Delivers more medication d. Indicates whether allergen exposure has occurred

c. Delivers more medication

The nurse is caring for an adolescent post cardiac transplant and notes episodes of tachycardia, lightheadedness, and irritability. Which additional assessment would support the possibility of rejection? a. Temperature of 98.4°F (36.8°C) orally b. Intake of 90% of meals for the last 24 hours c. Gain of 4 pounds in the last 2 days d. Respiratory rate varying between 20 and 24

c. Gain of 4 pounds in the last 2 days

A nurse is assessing a preterm infant. Which assessment items might indicate that the infant's respiratory status is worsening? a. Arterial CO2 of 43 b. Acrocyanosis c. Grunting and nasal flaring d. Respiratory rate of 50

c. Grunting and nasal flaring

The nurse is providing education to expectant mothers regarding modifiable risk factors for acquired heart disease. Which risk factors can be minimized through lifestyle changes? Select all that apply. a. Autoimmune factors b. Genetic factors c. Hyperthermia d. Cocaine and alcohol use e. Exposure to infectious agents

c. Hyperthermia d. Cocaine and alcohol use

The mother of a child who is 72 hours post-cardiac catheterization calls the clinic to report a temperature of 102°F (38.3°C). What is the best action for the nurse to take? a. Inform the mother it is normal to have some fever following a cardiac catheterization. b. Ask the mother to administer acetaminophen and recheck in 1 hour. c. Instruct the mother to bring the child in for evaluation. d. Instruct the mother to reduce activity and report any signs of changes to the affected extremity.

c. Instruct the mother to bring the child in for evaluation.

A 12-year-old is brought into the emergency department in severe respiratory distress. His respiratory rate is 45 breaths per minute. Oxygen saturations are 84% on room air. His mouth is open and he is gasping for air. Which oxygen delivery device would me most effective in delivering the highest oxygen concentration quickly? a. Nasal cannula b. Simple mask c. Nonrebreather mask d. An oxygen tent

c. Nonrebreather mask

A child is admitted to the hospital with pneumonia. The child's pulse oximetry reading is 85% on room air. What would the priority nursing activity for this child be? a. Obtain an arterial blood gas (ABG). b. Start an intravenous (IV) line and begin fluids. c. Place the child on oxygen at 1 liter via nasal cannula. d. Obtain a respiratory viral panel to determine the cause of illness.

c. Place the child on oxygen at 1 liter via nasal cannula.

A 6-week-old is admitted for respiratory syncytial virus (RSV). The infant has expiratory wheezes, increased work of breathing, and substernal and subcostal retractions. Oxygen saturations is 87% on room air. An intravenous line (IV) is placed, and the infant is placed on 2 liters via nasal cannula. Nursing care for this infant should include: a. Administering Acyclovir intravenously b. Giving morphine as needed for pain c. Providing nebulized hypertonic saline d. Giving Palivizumab (Synagis) prior to discharge

c. Providing nebulized hypertonic saline

Which statement does the nurse understand to be true about giving palivizumab, the immunization for respiratory syncytial virus (RSV), to an infant? a. Preterm infants are not able to receive the vaccine. b. It is 100% effective in preventing RSV. c. The immunization must be given monthly during RSV season. d. Palivizumab is one of several prevention vaccines for RSV.

c. The immunization must be given monthly during RSV season.

The nurse is caring for a child who swallowed a quarter. A bronchoscopy with retrieval is planned for later that day. Which assessment finding would be most concerning? a. The child develops a fever of 101°F (38.3°C). b. Intermittent audible stridor is noted. c. The respiratory rate is 45 with the increased work of breathing. d. Oxygen saturations drop to 96% on room air.

c. The respiratory rate is 45 with the increased work of breathing.

The parent of a 7-year-old girl who has just received her second influenza vaccine asks when she will be protected from the flu. What answer should the nurse give? a. Immediately after vaccination b. 5 days c. Two weeks d. 1 month

c. Two weeks

ductus arteriosus

connects the pulmonary artery to the aorta, bypassing the lungs

foramen ovale

connects the two atria in the fetal heart

ductus venosus

connects the umbilical vein to the inferior vena cava, bypassing the liver

The nurse is providing discharge teaching to the mother of a 4-year-old who had a tonsillectomy. Which statement made by the mother indicates she has a good understanding of what to feed her child when they arrive home? a. "I will give her whatever she wants so she eats something." b. "She likes to eat pretzels and tortilla chips for a snack. I will give that to her if she asks." c. "I will give her cool apple juice and orange juice to drink to help her throat." d. "I will give her Italian ice and yogurt initially when she gets hungry."

d. "I will give her Italian ice and yogurt initially when she gets hungry."

A 4-year-old is presented to the urgent care center with a history of a sudden onset of a severe sore throat. He began drooling and has difficulty swallowing. The temperature is 102.2°F (39.0°C). Lung sounds are clear and there is no cough. The child is very anxious and flushed, and is leaning forward in a tripod position. Based on these symptoms, the nurse anticipates a diagnosis of: a. Acute asthma attack b. Laryngotracheomalacia c. Acute laryngotracheobronchitis (Croup) d. Epiglottitis

d. Epiglottitis Sudden onset of high fever, sore throat with the 4 Ds (drooling, dysphagia, dysphonia, distressed air movement/stridor), and the tripod position are classic signs of acute epiglottitis

At what age should annual influenza vaccines first be given to children? a. Newborn b. Two weeks c. Three months d. Six months

d. Six months

The nurse is assigned to take care of a five-month-old with Respiratory Syncytial Virus (RSV). The baby presents with a temperature of 102.2°F (39°C) rectally, HR 165, RR, 72, and O2 saturation of 96% on room air. The best initial intervention for the nurse to do would be to: a. Make sure the baby eats to maintain hydration. b. Place an intravenous catheter (IV) promptly and hang IV antibiotics ASAP. c. Put the infant on 4 liters of oxygen via nasal cannula. d. Suction the nares bilaterally frequently.

d. Suction the nares bilaterally frequently.

A 6-year-old is admitted to the emergency department after being a restrained front seat passenger in a motor vehicle crash. The airbags deployed. Vital signs on admission are T-98F, HR 110, RR 56, BP 108/96. The child is having great difficulty breathing. The trachea is deviated to the left. There are no lung sounds on the right side. What problem might you suspect the child has? a. Cardiac contusion b. Fractured ribs c. Pneumonia d. Tension pneumothorax

d. Tension pneumothorax

What is tetralogy of fallot?

defect is associated with 22 deletion chromosome disorders such as DiGeorge's and Down's syndromes. "Tet" spells—sudden, marked increase in cyanosis; syncope; can lead to hypoxic brain injury and death Boot-shaped heart, right ventricular hypertrophy, and small pulmonary artery

what are the 4 Ds and an S in Epiglottis?

drooling dysphagia, dysphonia, distressed inspiratory air movement stridor

What are ACE inhibitor adverse effect?

dry cough angioedema reversible decline in renal function hyperkalemia

What are the characteristics of hypoplastic left ventricular heart defect?

hypoplasia of the left ventricle and encompasses atresia or critical stenosis of the aortic and/or mitral valves hypoplasia or the ascending aorta and aortic arch.

at what age can an infants use ibuprofen?

ibuprofen should not be used until after 6 months of age.

What is the adventitious sound for VENTRICULAR septal defect?

loud, harsh murmur with systolic thrill can be heard in patient with VSD *** enlarged right ventricular hypertrophy

What is coarctation of the aorta?

narrowing of the aorta between the upper body and the lower extremities results in increased BP in the upper extremities and head and reduction of BP in the lower extremities.

Nursing intervention for tricuspid atresia?

prostaglandin E1 administration to maintain blood flow to the lungs, and then a balloon septostomy is performed

In assessing an infant with right-sided heart failure, the nurse understands that the underlying condition prevents the right ventricle from pumping enough blood into the _____________, resulting in increased pressure in the right atrium and systemic venous system.

pulmonary artery

What is myringotomy?

surgical incision into the eardrum, to relieve pressure or drain fluid.

rheumatic heart disease (RHD)

systemic inflammatory disease that occurs in response to a group A beta-hemolytic streptococcal infection, such as strep throat, rheumatic fever, or scarlet fever, that starts in the throat.

Which heart defects that needs prostaglandin E1 treatment?

tetralogy of fallot Aortic stenosis Pulmonary Atresia Transposition of the Great Vessels Transposition of the Great Vessels Tricuspid Atresia Ebstein's Anomaly

76. A child with a VSD will have cardiomegaly of the left side of the heart. true or false

true The left side of the heart will be increased because of the increased pulmonary vascularity. 2. The increase in pulmonary vascularity causes cardiomegaly of the left side of the heart.


संबंधित स्टडी सेट्स

LearningCurve 6b. Operant Conditioning; Biology, Cognition, and Learning

View Set

Chapter 5 - Arousal, Stress and Anxiety

View Set

6. Cultivating Microorganisms Pt. 2

View Set

Graphing Lines By Finding Ordered Pairs #2

View Set

Developmental Psychology - Piaget, Kohlberg, Erikson and Bandura

View Set